Vous êtes sur la page 1sur 200

• Constrained, NLM, Friction

• Circular, WPE
Copyright © reserved with Motion Edu. Pvt. Ltd. and Publications

All rights reserved. No part of this work herein should be reproduced or used either
graphically, electronically, mechanically or by recording, photocopying, taping, web
distributing or by storing in any form and retrieving without the prior written permission of
the publisher. Anybody violating this is liable to be legally prosecuted.

Corporate Head Office


394 - Rajeev Gandhi Nagar Kota, (Raj.)
Ph. No. : 08003899588, 0744-2209671
IVRS No : 0744-2439051, 52, 53,
www. motioniitjee.com , info@motioniitjee.com
CONTENTS

CONSTRAINED, NLM, FRICTION


THEORY AND EXERCISE BOOKLET
S.NO. TOPIC ......................... PAGE NO.

 THEORY WITH SOLVED EXAMPLES ................................................... 05 – 56


 EXERCISE - 1 (JEE MAIN) ............................................... 57 – 70
 EXERCISE - 2 (JEE ADVANCED-OBJECTIVE) ........................ 71 – 91
 EXERCISE - 3 (JEE ADVANCED) ........................................................ 92 – 100
 EXERCISE - 4 (PRIVIOUS YEAR QUESTIONS) ................................ 101 – 108
 ANSWER KEY ....................................................................................... 109 – 111

CIRCULAR, WPE
THEORY AND EXERCISE BOOKLET
S.NO. TOPIC .............. PAGE NO.

 THEORY WITH SOLVED EXAMPLES ................................................... 112 – 157


 EXERCISE - 1 (JEE MAIN) ............................................... 158 – 169
 EXERCISE - 2 (JEE ADVANCED-OBJECTIVE) ........................ 170 – 181
 EXERCISE - 3 (JEE ADVANCED) ........................................ 182 – 190
 EXERCISE - 4 (PRIVIOUS YEAR QUESTIONS) ....................... 191 – 198
 ANSWER KEY ....................................................................................... 199 – 200
Page # 4 CONSTRAINED MOTION

Syllabus
Newton's law of motion; inertial frame of reference; Uniformly
accelerated frames of reference, Static and dynamic friction.

Circular Motion (uniform and non-uniform), Work, Power, Kinetic


Energy, Potential Energy, Conservation of Mechanical Energy.

Corporate Head Office : Motion Education Pvt. Ltd., 394 - Rajeev Gandhi Nagar, Kota-5 (Raj.)
CONSTRAINED MOTION Page # 5

CONSTRAINED MOTION
1. CONSTRAINED MOTION :

1.1 String constraint :


When the two object are connected through a string and if the string have the following properties :
• The length of the string remains constant i.e., it is inextensible string
• Always remains taut i.e., does not slacks.
Then the parameters of the motion of the objects along the length of the string have a definite relation
between them.
Ist format : - (when string is fixed)

A s B v

The block B moves with velocity v. i.e. each particle of block B moves with velocity v.
If string remain attached to block B it is necessary that velocity of each particle of string is same = v
(vs = v)
Now we can say that Block A also moves with velocity v.
v v
A B

vA = vB = v

: If pulley is fixed then the velocity of all the particles of string is same along the string.

v
B

Ex.1
A vA=?

Sol. In the above situation block B is moving with velocity v. Then speed of each point of the string is v
along the string.
 speed of the block A is also v
v
B

A vA=v

Ex.2

A
VA = 8 m/s

37°
vB=? B
A
Sol.  Block A is moving with velocity 8 ms–1. 8 m/s
 velocity of every point on the string must be 8m/s along the string. 8 m/s
The real velocity of B is vB. Then the string will not break only 8 m/s
when the compoent of vB along string is 8 m/s.
37°
8 vB B
 vB cos 37° = 8  vB = = 10 m/sec
cos 37

: 0744-2209671, 08003899588 | url : www.motioniitjee.com, : info@motioniitjee.com


Page # 6 CONSTRAINED MOTION
Ex.3 Find out the velocity of block B in a pulley block system as shown in figure.

53° 37°
10 m/s A B

Sol. In a given pulley block system the velocity of all the particle of string is let us assume v then.

v v
53° 37°
10m/s A B
53°
10cos53°
10 m/s is the real velocity of block A then its component along string is v.
 10 cos 53° = v ...(1)
v
If vB is the real velocity of block B then it component along string is v then 37°
vBcos37° = v ...(2)
vB B
from (1) & (2) vB cos37° = 10 cos53°
10  3 / 5 30 15
 vB = =  m / sec
4/5 4 2

50/3 m/s

53°

Ex.4

What is the velocity of block A in the figure as shown above.


Sol. The component of velocity of ring along string = velocity of A
50
= cos 53 = vA  vA = 10 m/s
3

: In the first format only two points of string are attached or touched to moving bodies.
IInd format (when pulley is also moving)

To understand this format we consider the following example vP


in which pulley is moving with velocity vp and both block
have velocity vA & vB respectively as shwon in figure.
If we observe the motion of A and B with respect to pulley.
Then the pulley is at rest. Then from first format.
vA vB
vAP = – vBP
A B

Corporate Head Office : Motion Education Pvt. Ltd., 394 - Rajeev Gandhi Nagar, Kota-5 (Raj.)
CONSTRAINED MOTION Page # 7

(–ve sign indicate the direction of each block is opposite with respect to Pulley)
v A – vp = – vB + v P
v A  vB
 vP =
2

:- To solve the problem put the values of vA, vB, & vP with sign.

10 m/s
vP

Ex.5

A v=?
A

B
v A  vB
Sol. vP =
2
Putting vp = 10 ms–1, vB = 0,
we get
vA = 20 ms–1 (upward direction)

vP= 10m/s

Ex.6

5m/s
A B v=?
B

Sol. If we take upward direction as +ve then


–5  vB
10 =
2
vB = 25 m/sec (in upward direction)

7 m/s

E F
8m/s

Ex.7
2m/s A B C D
5m/s

Find out the velocity of Block D


Sol. From 2nd format of constrained motion
v A  vB
vE =
2
2–5
vE = = –3/2 (If upward direction is taken to be +ve)
2
vE = –3/2 m/s

: 0744-2209671, 08003899588 | url : www.motioniitjee.com, : info@motioniitjee.com


Page # 8 CONSTRAINED MOTION
vE  vF –3 / 2  vF 31
Now = 7 m/s  7 =  14 + 3/2 = vF  vF =
2 2 2
v C  vD 8  vD 31
Now = vF  =  vD = 31 – 8
2 2 2
vD = 23 m/s (upward direction)

B C

E
Ex.8 G
m
A H F

10 m/sec

Find the velocity of point G.


Sol. In string ABCD from first format of constrain
VD = 10 m/s
v H  vE
Now vD =
2
vH = 10 m/s  if upward direction is taken to be positive
10  vE
+ 10 =  vE = 30 m/s 
2
vF  v G –10  v G
Now = vE  30 =
2 2
60 + 10 = vG
vG = 70 m/s

: In IInd format three or four Points of the string is attached to the moving bodies.

III format :

SOLVING STRATEGY :
1. First choose the longest string in the given problem which contains the point of which velocity/
acceleration to be find out.
2. Now mark a point on the string wherever it comes in contact or leaves the contact of real bodies.

3. If due to motion of a point, length of the part of a string with point is related, increases then its speed
will be taken +ve otherwise –ve.

A
D E H I

J
C vC=?
Ex.9 B C F G
5m/s
A B 2m/s

Sol.

Corporate Head Office : Motion Education Pvt. Ltd., 394 - Rajeev Gandhi Nagar, Kota-5 (Raj.)
CONSTRAINED MOTION Page # 9
Step 1. We choose a longest string ABCDEFGHIJ in which we have to find out velocity of point J (vc)
Step 2. Mark all the point A, B ................
Step 3. Write equation
vA + vB + vC + vD + vE + vF + vG + vH + vI + vJ = 0
vA = vD = vE = vH = vI = 0
(No movement of that point because attached to fixed objects)
 vB + vc + vF + vG + vJ = 0 ...(1)
vB = vC = 5 m/s (increases the length)
vF = vG = 2m/s (It also increases the length)
Let us assume C is moving upward with velocity vc so vc negative because it decreasing the length
 5 + 5 + 2 + 2 – vc = 0
vC = 14 m/sec (upward)

Ex.10
4m/s
F

c  2m / sec

E
8 m / s A B  2m / s
1 m / s D
Find out the velocity of block E as shown in figure.
Sol.
Step-1 We first choose the longest string in which point j (block E) lie. (abcdefghij)
4m/s
c d F
b e
x
a h y i
c  2m / s
k
j
f g E
8m / s  A B  2m / s
1m / s  D z
Step 2 : Now write equation according to the velocity of each point (either increase or decrease the length)
va + vb + vc + vd + ve + vf + vg + vh + vi + vj= 0 ...(1)
Now find value of va, vb ..... in a following way
v A  vB
vk = (from second format)
2
8–2
= = 3 m/sec. (upward)
2

: 0744-2209671, 08003899588 | url : www.motioniitjee.com, : info@motioniitjee.com


Page # 10 CONSTRAINED MOTION

vK  vC
va = (from 2nd format)
2
32
= = 5/2 m/sec. (upward)
2
vx = 4m/s (from first format of constrain)
vy  vz
from 2nd format of constrain vx =  vz = 0 (fixed)
2
 vy = 2 vx = 8 m/s (upward)
 Now va = – 5/2 m/s (decreases the length)
vb = vc = vd = ve = 0 (attached to fixed object)
vf = vg = 1m/s (increases the length)
vh = vi = vy = 8 m/s (increase the length)
Let us assume block E move upward then vj = – vE (decrease the length)
Puting the above values in eq. (1)
 –5/2 + 1 + 1 + 8 + 8 – vE = 0
vE = 31/2 m/s (upward)

: In the following figure pulley is moving with velocity v at an angle  with the horizontal.

v sin 
v
v
A B A
 B

C C v cos 

D D
Only v cos  is responsible to increase or decrease the length AB and v sin  is responsible to either
decrease or increase the length CD.
* Further solving strategy is same as 3rd format
Ex.11 Find out the relation between acceleration a and b as shown in following figure.

b
A a
B

Sol.
Step 1. Mark the points on the string which is attached to the real object (e.f,g,h)

b cos 
f b f
e
g
b  g
bc

h
b
os

A a

B  h
 b
a
Step 2. Acceleration of each point which are responsible to effect the length of string
ae = 0 (because it is attached to fixed object)
af = –b (attach to pulley which is moving with wedge's acceleration & –ve because it decreases the
length)

Corporate Head Office : Motion Education Pvt. Ltd., 394 - Rajeev Gandhi Nagar, Kota-5 (Raj.)
CONSTRAINED MOTION Page # 11
ag = b cos  (only this component is responsible to effect the length of string)
ah = (a – b cos ) (resultant velocity at point h along the string)
So now from 3rd format
ae + af + ag + ah = 0
 0 + (–b) + b cos  + (a – b cos ) = 0
a–b=0
 a=b

2. WEDGE CONSTRAINT :

Conditions :
(i) Contact must not be lost between two bodies.
(ii) Bodies are rigid.
The relative velocity / acceleration perpendicular to the contact surface of the two rigid object is
always zero. Wedge constraint is applicable for each contact.

v3

v3 v1
v 1 sin 
v2

Contact
Plane
v 3  v 1 sin 
In other words,
Components of velocity and acceleration perpendicular to the contact surface of the two objects is
always equal if there is no deformation and they remain in contact.
Ex.12 Find the relation between velocity of rod and that of the wedge at any instant in the figure
shown.

v
 u

Sol. Using wedge constraint.


Component of velocity of rod along perpendicular to inclined surface is equal to velocity of wedge
along that direction.
u cos  = v sin 
u
 tan
v
u = v tan 

: 0744-2209671, 08003899588 | url : www.motioniitjee.com, : info@motioniitjee.com


Page # 12 NEWTON’S LAW OF MOTION

NEWTON’S LAW OF MOTION


1. FORCE

A pull or push which changes or tends to change the state of rest or of uniform motion or direction of
motion of any object is called force. Force is the interaction between the object and the source
(providing the pull or push). It is a vector quantity.
Effect of resultant force :
• may change only speed
• may change only direction of motion.
• may change both the speed and direction of motion.
• may change size and shape of a body
kg.m
unit of force : newton and (MKS System)
s2
g.cm
dyne and (CGS System)
s2
1 newton = 105 dyne
Kilogram force (kgf)
The force with which earth attracts a 1 kg body towards its centre is called kilogram force, thus
Force in newton
kgf =
g
Dimensional Formula of force : [MLT–2]
• For full information of force we require
 Magnitude of force
 direction of force
 point of application of the force

Force

Electromagnetic Gravitational Contact Nuclear


force force force force

Normal Tension friction


reaction

1.1 Electromagnetic Force


Force exerted by one particle on the other because of the electric charge on the particles is called
electromagnetic force.
Following are the main characteristics of electromagnetic force
• These can be attractive or repulsive
• These are long range forces
• These depend on the nature of medium between the charged particles.
• All macroscopic force (except gravitational) which we experience as push or pull or by contact are
electromagnetic, i.e., tension in a rope, the force of friction, normal reaction, muscular force, and
force experienced by a deformed spring are electromagnetic forces. These are manifestations of the
electromagnetic attractions are repulsions between atoms/molecules.

Corporate Head Office : Motion Education Pvt. Ltd., 394 - Rajeev Gandhi Nagar, Kota-5 (Raj.)
NEWTON’S LAW OF MOTION Page # 13
1.2 Gravitational force :
It acts between any two masses kept anywhere in the universe. It follows inverse square rule (F 
1
) and is attractive in nature.
dis tan ce2
GM1M 2
F=
R2
The force mg, which Earth applies on the bodies, is gravitational force.

1.3 Nuclear force :


It is the strongest force. It keeps nucleons (neutrons and protons) together inside the nucleus inspite
of large electric repulsion between protons. Radioactivity, fission, and fusion, etc. result because of
unbalancing of nuclear forces. It acts within the nucleus that too upto a very small distance.

1.4 Contact force :


Forces which are transmitted between bodies by short range atomic molecular interactions are called
contact forces. When two objects come in contact they exert contact forces on each other.

1.4.1 Normal force (N) :

It is the component of contact force perpendicular to the surface.


It measures how strongly the surfaces in contact are pressed
against each other. It is the 2
electromagnetic force. A table is placed on Earth
as shown in figure. 1
3 4
• Here table presses the earth so normal force exerted by four legs of table on earth are as shown in
figure.

N1 N2
ground
N3 N4
• Now a boy pushes a block kept on a frictionless surface.

Block

Here, force exerted by boy on block is electromagnetic interaction which arises due to similar charges
appearing on finger and contact surface of block, it is normal force.

(by boy) N
Block

• A block is kept on inclined surface. Component of its weight presses the surface perpendicularly due
to which contact force acts between surface and block.


Normal force exerted by block on the surface of inclined plane is shown in figure.

N

: 0744-2209671, 08003899588 | url : www.motioniitjee.com, : info@motioniitjee.com


Page # 14 NEWTON’S LAW OF MOTION
Force acts perpendicular to the surface

:
• Normal force acts in such a fashion that it tries to compress the body
• Normal is a dependent force, it comes in role when one surface presses the other.

Ex.1 Two blocks are kept in contact on a smooth surface as shown in figure. Draw normal force
exerted by A on B.

A B

Sol. In above problem, block A does not push block B, so there is no molecular interaction between A and B.
Hence normal force exerted by A on B is zero.

Ex.2 Draw normal forces on the massive rod at point 1 and 2 as shown in figure.

Sol. Normal force acts perpendicular to extended surface at point of contact.


N2

N1

50N
30°
Ex.3 Two blocks are kept in contact as shown in figure. Find
(a) forces exerted by surfaces (floor and wall) on blocks 100N
10kg 20kg
(b) contact force between two blcoks.

N1
Sol. F.B.D. of 10 kg block
100 N
N1 = 10 g = 100 N ...(1) N2
N2 = 100 N ...(2)
10 g

N4 50 N
F.B.D. of 20 kg block
30°
N2 = 50 sin 30° + N3
 N3 = 100 – 25 = 75 N ...(3)
N2 N3
and N4 = 50 cos30° + 20 g
N4 = 243.30 N
20 g

Corporate Head Office : Motion Education Pvt. Ltd., 394 - Rajeev Gandhi Nagar, Kota-5 (Raj.)
NEWTON’S LAW OF MOTION Page # 15

R=5m
B 3m
A 1m
Ex.4

Find out the normal reaction at point A and B if the mass of sphere is 10 kg.

N2 N1
y
N2 N1
O

5m3m 37° B 37° 53°


Sol. A 53°
1m 4m Now F.B.D. x' O x
10 g
y'

Now resolve the forces along x & y direction

3N2
N2sin37° 
5
y
N1sin53° = 4N1/5
N2 N1

37° 53°
3N1
4N2 O N1cos53° 
N2cos37°= 5
5
100

 The body is in equilibrium so equate the force in x & y direction


3N1 4N2
In x-direction = ...(1)
5 5
3N2 4N1
In y-direction + = 100 ...(2)
5 5
after solving above equation
N1 = 80 N, N2 = 60 N

1.4.2 Tension :
Tension in a string is an electromagnetic force. It arises when a string is pulled. If a massless string is
not pulled, tension in it is zero. A string suspended by rigid support is pulled by a force ‘F’ as shown in
figure, for calculating the tension at point ‘A’ we draw F.B.D. of marked portion of the string; Here string
is massless.

F.B.D of marked portion


A T
A

F F

: 0744-2209671, 08003899588 | url : www.motioniitjee.com, : info@motioniitjee.com


Page # 16 NEWTON’S LAW OF MOTION
 T=F
String is considered to be made of a number of small segments which attracts each other due to
electromagnetic nature. The attraction force between two segments is equal and opposite due to
newton’s third law.
Conclusion :
T = mg
(i) Tension always acts along the string and in such a direction that it tries
to reduce T
the length of string
(ii) If the string is massless then the m
tension will be same along the string but if the string have some mass
then the tension will continuously change along the string. mg

Ex.5 The system shown in figure is in equilibrium. Find the magnitude of tension in each string ;
T1, T2, T3 and T4. (g = 10 m/s–2)

60°
T4
T3 30° B
T
T1 A 2
10 kg

Sol. F.B.D. of block 10 kg F.B.D. of point ‘A’

y
T0 T2
30°
T0=10 g T1 x
T0=100N A

10g T0

 Fy = 0
T2 cos30° = T0 = 100 N
200
T2 = N
3
 Fx = 0
200 1 100
T1 = T2 sin 30° = . = N
3 2 3
y

F.B.D of point of ‘B’ T4


60°
 Fy = 0  T4 cos60° = T2 cos 30°
T3 x
and  Fx = 0  T3 + T2 sin 30° = T4 sin 60° B
3 0°
200 T2
 T3 = N , T = 200 N
4
3
1.4.3 Frictional force :
It is the component of contact force tangential to the surface. It opposes the relative motion (or
attempted relative motion) of the two surfaces in contact. (which is explained later)

Corporate Head Office : Motion Education Pvt. Ltd., 394 - Rajeev Gandhi Nagar, Kota-5 (Raj.)
NEWTON’S LAW OF MOTION Page # 17

2. NEWTON’S FIRST LAW OF MOTION :


According to this law “A system will remain in its state of rest or of uniform motion unless a net external
force act on it.
1st law can also be stated as “If the net external force acting on a body is zero, only then the body
remains at rest.”
The word external means external to the system (object under observation), interactions within the
system has not to be considered.
The word net means the resultant of all the forces acting on the system.
Newton’s first law is nothing but Galileo’s law of inertia.
Inertia means inability of a body to change its state of motion or rest by itself.
The property of a body that determines its resistance to a change in its motion is its mass (inertia).
Greater the mass, greater the inertia.
An external force is needed to set the system into motion, but no external force is needed to keep a
body moving with constant velocity in its uniform motion.
Newton’s laws of motion are valid only in a set of frame of references, these frames of reference are
known as inertial frames of reference.
Generally, we take earth as an inertial frame of reference, but strictly speaking it is not an inertial
frame.
All frames moving uniformly with respect to an inertial frame are themselves inertial.
We take all frames at rest or moving uniformly with respect to earth, as inertial frames.

3. NEWTON’S SECOND LAW OF MOTION :


Newton’s second law states, “The rate of change of a momentum of a body is directly proportional to
the applied force and takes place in the direction in which the force acts”
  
 dp
i.e., F  dp or F  k
dt dt
where k is a constant of proportionality.
   
p  mv, So F  k (dmv)
dt
For a body having constant mass,
 
dv 
 F  km  kma
dt
From experiments, the value of k is found to be 1.
 
So, Fnet  ma
Force can’t change the momentum along a direction normal to it, i.e., the component of velocity
normal to the force doesn’t change.
Newton’s 2nd law is strictly applicable to a single point particle. In case of rigid bodies or system of
 
particles or system of rigid bodies, F refers to total external force acting on system and a refers to
acceleration of centre of mass of the system. The internal forces, if any, in the system are not to be

included in F .
Acceleration of a particle at any instant and at a particular location is determined by the force (net)
acting on the particle at the same instant and at same location and is not in any way depending on the
history of the motion of the particle.
PROBLEM SOLVING STRATEGY :
Newton’s laws refer to a particle and relate the forces acting on the particle to its mass and to its
acceleration. But before writing any equation from Newton’s law, you should be careful about which
particle you are considering. The laws are applicable to an extended body too which is nothing but
collection of a large number of particles.
Follow the steps given below in writing the equations :

: 0744-2209671, 08003899588 | url : www.motioniitjee.com, : info@motioniitjee.com


Page # 18 NEWTON’S LAW OF MOTION
Step 1 : Select the body
The first step is to decide the body on which the laws of motion are to be applied. The body may be a
single particle, an extended body like a block, a combination of two blocks-one kept over another or
connected by a string. The only condition is that all the parts of the body or system must have the
same acceleration.
Step 2 : Identify the forces
Once the system is decided, list down all the force acting on the system due to all the objects in the
environment such as inclined planes, strings, springs etc. However, any force applied by the system
shouldn’t be included in the list. You should also be clear about the nature and direction of these
forces.
Step 3 : Make a Free-body diagram (FBD)
Make a separate diagram representing the body by a point and draw vectors representing the forces
acting on the body with this point as the common origin.
This is called a free-body diagram of the body.

Tb Ts
R
B
5
C R
m/sec2

Wp Wm
100kg platform man
F.B.D of Diagram
50 kg
Look at the adjoining free-body diagrams for the platform and the man. Note that the force applied by
the man on the rope hasn’t been included in the FBD.
Once you get enough practice, you’d be able to identify and draw forces in the main diagram itself
instead of making a separate one
Step 4 : Select axes and Write equations
When the body is in equillibrium then choose the axis in such a fashion that maximum number of force
lie along the axis.
If the body is moving with some acceleration then first find out the direction of real acceleration and
choose the axis one is along the real acceleration direction and other perpendicular to it.
Write the equations according to the newton’s second law (Fnet = ma) in the corresponding axis.
4. APPLICATIONS :
4.1 Motion of a Block on a Horizontal Smooth Surface.
Case (i) : When subjected to a horizontal pull :
The distribution of forces on the body are shown. As there is no motion along vertical direction, hence,
R = mg
For horizontal motion F = ma or a = F/m R
a
m F

mg
Case (ii) : When subjected to a pull acting at an angle () to the horizontal :
Now F has to be resolved into two components, F cos along the horizontal and F sin  along the
vertical direction.
Fsin
R
F

m Fcos

mg
For no motion along the vertical direction.

Corporate Head Office : Motion Education Pvt. Ltd., 394 - Rajeev Gandhi Nagar, Kota-5 (Raj.)
NEWTON’S LAW OF MOTION Page # 19

we have R + F sin  = mg
or R = mg – F sin 

: Hence R  mg. R < mg

For horizontal motion

F cos 
F cos  = ma, a =
m

Case (iii) : When the block is subjected to a push acting at an


angle  to the horizontal : (down ward) R
The force equation in this case 
F
R = mg + F sin 


F cos 

: R  mg, R > mg
mg F
For horizontal motion

F cos  F sin 
F cos = ma, a =
m

4.2 Motion of bodies in contact.


Case (i) : Two body system :
Let a force F be applied on mass m1

B
A
F
f f
m1 m2

Free body diagrams :


(vertical force do not cause motion, hence they have not been shown in diagram)

F m 2F
 a = m m and f = m  m
1 2 1 2

(i) Here f is known as force of contact. f


m1 m2
(ii) Acceleration of system can be found simply by F f

force
a=
total mass

: If force F be applied on m2, the acceleration will remain the same, but the force of contact will be

different

m1F
i.e., f’ = m  m
1 2

: 0744-2209671, 08003899588 | url : www.motioniitjee.com, : info@motioniitjee.com


Page # 20 NEWTON’S LAW OF MOTION
Ex.6 Find the contact force between the 3 kg and 2kg block as shown in figure.
B
A F2 = 25N
F1 = 100N 3kg 2kg

Sol. Considering both blocks as a system to find the common acceleration


Fnet = F1 – F2 = 100 – 25 = 75 N
common acceleration Fnet = 75 N
5kg a
Fnet 75 2
a   15 m / s
5kg 5
To find the contact force R
between A & B we draw a
F.B.D of 2 kg block N 2kg 25 N
from (Fnet)x = max
 N – 25 = (2) (15)
 N = 55 N 2g

Case (ii) : Three body system :


C
A B

F m1 m2 m3

Free body diagrams :


For A For B For C
F
 a = m m m
1 2 3

(m2  m 3 ) F m1 m2 m3
F f1 f1 f2 f2
and f1 = (m  m  m )
1 2 3

m 3F F – f1 = m1a f1 – f2 = m2a f2 = m3a


f2 = (m  m  m )
1 2 3
f1 = contact force between masses m1 and m2
f2 = contact force between masses m2 and m3
Remember : Contact forces will be different if force F will be applied on mass C
Ex.7 Find the contact force between the block and acceleration of the blocks as shown in figure.
C
B
A
F1 = 50N 3kg F2 = 30N
2kg 5kg

Sol. Considering all the three block as a system to find the common acceleration
Fnet = 50 – 30 = 20 N
20 Fnet=20N
a  2m / s2 10kg a
10
To find the contact force
R
between B & C we draw F.B.D.
of 3 kg block.
N1 3kg 30N
 Fnet
x
 ma
a
 N1 – 30 = 3(2)  N1 = 36 N mg
To find contact force between A & B we draw a
F.B.D. of 5 kg block
N2 5kg N1
 N2 – N1 = 5a
N2 = 5 × 2 + 36  N2 = 46 N

Corporate Head Office : Motion Education Pvt. Ltd., 394 - Rajeev Gandhi Nagar, Kota-5 (Raj.)
NEWTON’S LAW OF MOTION Page # 21
4.3 Motion of connected Bodies
Case (i) For Two Bodies :
F is the pull on body A of mass m1. The pull of A on B is exercised as tension through the string
connecting A and B. The value of tension throughout the string is T only.
B A
T T
m2 m1 F

Free body diagrams :

For body A For body B

R1 a R2 a

T A F B T

m 1g m2 g

R1 = m1g R2 = m2g
F – T = m1a T = m2a

F
 a = m m
1 2

Case (ii) : For Three bodies :


a
A B C 
T1 T2 F
m1 m2 m3

Free body diagrams :

For A For B For C

R1 R2 R3

A T1 B C
T1 T2 T2 F

m1g m2g m3g

R2 = m2g R3 = m3g
R1 = m1g
T2 – T1 = m2 a F – T2 = m3a
T1 = m1a
 T2 = m2a + T1  F = m3a + T2
T2 = (m2 + m1 )a =m 3 a+(m 1+m2)a
F=(m1 +m2+m3)a

F
 a = m m m
1 2 3

: 0744-2209671, 08003899588 | url : www.motioniitjee.com, : info@motioniitjee.com


Page # 22 NEWTON’S LAW OF MOTION
Ex.8 A 5 kg block has a rope of mass 2 kg attached to its underside and a 3 kg block is suspended
from the other end of the rope. The whole system is accelerated upward is 2 m/s2 by an external
force F0.
F0
5 kg
2 kg
3 kg
(a) What is F0?
(b) What is the force on rope?
(c) What is the tension at middle point of the rope?
(g = 10 m/s2)
Sol. For calculating the value of F0, consider two blocks with the rope as a system.
F.B.D. of whole system

F0

(a)
2m/s2
10 g = 100N

F0 – 100 = 10 × 2
F = 120 N ...(1)
(b) According to Newton’s second law, net force on rope.
F = ma = (2) (2) = 4 N ...(2)
(c) For calculating tension at the middle point we draw T
F.B.D. of 3 kg block with half of the rope (mass 1 kg) as
shown.
T – 4g = 4.(2) = 48 N 4g

4.4 Motion of a body on a smooth inclined plane :

Natural acceleration down the plane = g sin 


Driving force for acceleration a up the plane, F=m(a+ gsin)
and for an acceleration a down the plane, F=m(a – gsin)

mg sin mgcos
mg

Ex.9 Find out the contact force between the 2kg & 4kg block as shown in figure.
g
4k
g
2k

37º

Sol. On an incline plane acceleration of the block is independent of mass. So both the blocks will move with
the same acceleration (gsin 37º) so the contact force between them is zero.

Corporate Head Office : Motion Education Pvt. Ltd., 394 - Rajeev Gandhi Nagar, Kota-5 (Raj.)
NEWTON’S LAW OF MOTION Page # 23

Ex.10 Find out the contact force between 2kg & 3kg block placed on the incline plane as shown in
figure.

g
3k
g
2k
N
20
37º
Sol. Considering both the block as a 5kg system because both will move the same acceleration.

g
5k
N
20
37º
Now show forces on the 5 kg block N

g
5k
N
20

37º 5g

 Acceleration of 5kg block is down the incline.

N
So choose one axis down the incline and other

g
5k
a
N
perpendicular to it
From Newton’s second Law 20
º
37 5gcos 37º
s in
37º 5g
N = 5g cos 37º ...(i)
5g

5gsin 37º – 20 = 5a ..(ii)


30 – 20 = 5a
a = 2m/s2 (down the incline)
For contact force (N1) between 2kg & 3kg block
1
N

we draw F.B.D. of 3kg block


From
g
2

3k
/s

Fnet = ma
2m

º
37

 3gsin 37º – N1 = 3 × 2
s in
3g

18 – N1 = 6
N1= 12 N

4.5 Pulley block system :

Ex.11 One end of string which passes through pulley and connected to 10 kg mass at other end is
pulled by 100 N force. Find out the acceleration of 10 kg mass. (g = 9.8 m/s2)

Sol. Since string is pulled by 100N force.


100 N
So tension in the string is 100 N.
F.B.D. of 10 kg block
100 – 10 g = 10 a
100 N
100 – 10 × 9.8 = 10 a
10 g
a = 0.2 m/s2 10 kg

: 0744-2209671, 08003899588 | url : www.motioniitjee.com, : info@motioniitjee.com


Page # 24 NEWTON’S LAW OF MOTION

Ex.12 In the figure shown, find out acceleration of each block.

10kg

2kg 4kg

Sol. Now F.B.D. of each block and apply Newton’s


second law on each F.B.D
10kg
a1
2T

(1) 10 kg a1  10g –2 T = 10a1 ...(i) a2 2kg 4kg a3

10g

(2) 2kg a2  T – 2g = 2a2 ...(2)

2g

(3) 4kg a3  T – 4g = 4a3 ...(3)

4g

from constrain relation 2a1 = a2 + a3 ...(4)


Solving equations (1), (2), (3) and (4) we get
800
T= N
23
a1 = 70/23 m/s2 (downward), a2 = 170/23 m/s2 (upward), a3 = 30/23 m/s2 (downward)
Ex.13 Find the acceleration of each block in the figure shown below; in terms of their masses m1,
m2 and g. Neglect any friction.
m1

m2

Corporate Head Office : Motion Education Pvt. Ltd., 394 - Rajeev Gandhi Nagar, Kota-5 (Raj.)
NEWTON’S LAW OF MOTION Page # 25

Sol. Let T be the tension in the string that is assumed to be massless. N1


For mass m1, the FBD shows that
N1 = m1g T
Where N1 is the force applied upward by plane on the mass m1.
If acceleration of m1 along horizontal is a1. then
m1g
T = m1a1 ...(i)
For mass m2, the FBD shows that
m2g – 2T = m2a2 ...(ii)
Where a2 is vertical acceleration of mass m2.
Note that upward tension on m2 is 2T applied
by both sides of the string. T T
from constrain relation 2T
a
a2 = 1
2
Thus, the acceleration of m1 its twice that of m2. m2g
with this input, solving (i) and (ii) we find
a2
2m2 g
a1 = 4m  m m2g
1 2
m2g
a2 = 4m  m
1 2

Ex.14 Two blocks A and B each having a mass of 20 kg, rest on frictionless surfaces as shown in the
figure below. Assuming the pulleys to be light and frictionless, compute :
(a) the time required for block A, to move down by 2m on the plane, starting from rest,
(b) tension in the string, connecting the blocks.
A

B
37º
Sol.
Step 1. Draw the FBDs for both the blocks. If tension in the string is T, then we have

NA T
NB
T
and
mAg mBg
Note that mAg, should better be resolved along and perpendicular to the plane, as the block A is moving
along the plane.
T
NA

mAg sin mAg cos

Step 2. From FBDs, we write the force equations ‘


for block A where
NA = mA g cos  = 20 × 10 × 0.8 = 160 N
and mAg sin  – T = mA a ... (i)
Where ‘a’ is acceleration of masses of blocks A and B.
Similarly, force equations for block B are
NB = mBg = 20 × 10 = 200 N
and T = mBa ...(ii)
From (i) and (ii), we obtain

: 0744-2209671, 08003899588 | url : www.motioniitjee.com, : info@motioniitjee.com


Page # 26 NEWTON’S LAW OF MOTION

m A g sin  20  10  0.6
a = m m = = 3 ms–2
A B 40
T = mBa = 20 × 3 = 60 N

Step 3. With constant acceleration a = 3 ms–2, the block A moves down the inclined plane a distance S =
2 m in time t given by
1 2 2S 2
S= at or t =  sec onds.
2 a 3

Ex.15 Two blocks m1 and m2 are placed on a smooth inclined


plane as shown in figure. If they are released from rest.
Find : m1
(i) acceleration of mass m1 and m2 3kg m2 1kg
(ii) tension in the string
(iii) net force on pulley exerted by string 30° 60°
Sol. F.B.D of m1 : N1 T
m1g sin  – T = m1a
a
m1
3
g – T = 3a ...(i)
2 =30°
m1g
F.B.D. of m2 :
a
T – m2g sin  = m2a T N2

3 m2
T – 1. g = 1.a ...(ii)
2 
Adding eq. (i) and (ii) we get a = 0
m2g
Putting this value in eq. (i) we get

3g
T= ,
2

F.B.D. of pulley
FR = 2T
T T
3
FR = g FR
2

5. NEWTONS’ 3RD LAW OF MOTION :

Statement : “To every action there is equal and opposite reaction”.


But what is the meaning of action and reaction and which force is action and which force is reaction?
Every force that acts on body is due to the other bodies in environment. Suppose that a body A
 
experiences a force FAB due to other body B. Also body B will experience a force FBA due to A.
According to Newton third law two forces are equal in magnitude and opposite in direction Mathematically
we write it as
 
FAB  –FBA
 
Here we can take either FAB or FBA as action force and other will be the reaction force.

Corporate Head Office : Motion Education Pvt. Ltd., 394 - Rajeev Gandhi Nagar, Kota-5 (Raj.)
NEWTON’S LAW OF MOTION Page # 27

: (i) Action-Reaction pair acts on two different bodies.

(ii) Magnitude of force is same.


(iii) Direction of forces are in opposite direction.
(iv) For action-reaction pair there is no need of contact

Ex.16 A block of mass ‘m’ is kept on the ground as shown in figure.


m
(i) Draw F.B.D. of block
(ii) Are forces acting on block action - reaction pair
(iii) If answer is no, draw action reaction pair.
Sol. (i) F.B.D. of block

N (Normal)

mg (field force)
(ii) ‘N’ and Mg are not action - reaction pair. Since pair act on different bodies, and they are of same
nature.
(iii) Pair of ‘mg’ of block acts on earth in opposite direction.
m

earth
mg
and pair of ‘N’ acts on surface as shown in figure.

5.1 Climbing on the Rope :

F.B.D of man
Rope
T

a a

mg

Now three condition arises.


if T > mg  man accelerates in upward direction
T < mg  man accelerates in downward direction
T = mg  man’s acceleration is zero
* Either climbing or decending on the rope man exerts force downward

: 0744-2209671, 08003899588 | url : www.motioniitjee.com, : info@motioniitjee.com


Page # 28 NEWTON’S LAW OF MOTION
Ex.17 If the breaking strength of string is 600N then find out
the maximum acceleration of the man with which he
can climb up the road
600N
(50 kg)
Sol. Maximum force that can be exerted on the man by the rope is 600 N.
F.B.D of man a
 600 – 50 g = 50 a
amax = 2 m/s2
50 g

Ex.18 A 60 kg painter on a 15 kg platform. A rope attached to the platform and passing over an
overhead pulley allows the painter to raise himself along with the platform.

400 N

(i) To get started, he pulls the rope down with a force of 400 N. Find the acceleration of the platform
as well as that of the painter.
(ii) What force must he exert on the rope so as to attain an upward speed of 1 m/s in 1 s ?
(iii) What force should apply now to maintain the constant speed of 1 m/s?
Sol. The free body diagram of the painter and the platform as a system can be drawn as shown in the
figure. Note that the tension in the string is equal to the force by which he pulles the rope.
(i) Applying Newton’s Second Law
2T – (M + m)g = (M + m)a T T
2T – (M  m)g
or a= a
Mm
Here M = 60 kg; m = 15 kg ; T = 400 N
g = 10 m/s2
2( 400) – (60  15)(10) (M+m) g
a= = 0.67 m/s2
60  15
(ii) To attain a speed of 1 m/s in one second the acceleration a must be 1 m/s2
Thus, the applied force is
1
F= (M + m) (g + a) = (60 + 15) (10 + 1) = 412.5 N
2
(iii) When the painter and the platform move (upward) together with a constant speed, it is in a state
of dynamic equilibrium
Thus, 2F – (M + m) g = 0
(M  m)g (60  15)(10)
or F =  = 375 N
2 2

6. SPRING FORCE :

Every spring resists any attempt to change its length; when it is compressed or extended, it exerts
force at its ends. The force exerted by a spring is given by F = –kx, where x is the change in length and
k is the stiffness constant or spring constant (unit Nm–1)
When spring is in its natural length, spring force is zero.

Corporate Head Office : Motion Education Pvt. Ltd., 394 - Rajeev Gandhi Nagar, Kota-5 (Raj.)
NEWTON’S LAW OF MOTION Page # 29

0 +x
0

F=0 Fext F = –kx


F

Graph between spring force v/s x

Ex.19 Two blocks are connected by a spring of natural length 2 m. The force constant of spring is 200
N/m. Find spring force in following situations.
2m

A B

(a) If block ‘A’ and ‘B’ both are displaced by 0.5 m in same direction.
(b) If block ‘A’ and ‘B’ both are displaced by 0.5 m in opposite direction.
Sol. (a) Since both blocks are displaced by 0.5 m in same direcetion, so change in length of spring is zero.
Hence, spring force is zero.
(b) In this case, change in length of spring is 1 m. So spring force is F = –Kx
= – (200). (1)
F = –200 N
2m

A B Natural length

3m 1m

A B When spring A B When spring


is extended is compressed
F F F F

Ex.20 Force constant of a spring is 100 N/m. If a 10 kg block attached


with the spring is at rest, then find extension in the spring
(g = 10 m/s2)
Sol. In this situation, spring is in extended state so spring force acts in
upward direction. Let x be the extension in the spring.
F.B.D. of 10 kg block :
Fs
Fs = 10 g 10 kg
 Kx = 100
 (100)x = (100)
 x=1m
10g

6.1 SPRING FORCE SYSTEM :


Initially the spring is in natural length at A with block m. But when the block displaced towards right
then the spring is elongated and now block is released at B then the block move towards left due to
spring force (kx).

: 0744-2209671, 08003899588 | url : www.motioniitjee.com, : info@motioniitjee.com


Page # 30 NEWTON’S LAW OF MOTION
Analysis of motion of block :

Natural Length
v a v a
v=0
m m Initial position

C v a Av a B

(i) From B to A speed of block increase and acceleration decreases. (due to decrease in spring force kx)

kx a
m
v
(ii) Due to inertia block crosses natural length at A.
From A to C speed of the block decreases and acceleration increases.(due to increase in spring force
kx)

m kx
a
v
(iii) At C the block stops momentarily at this instant and since the spring is compressed spring force is
towards right and the block starts to move towards right. From C to A speed of block increases and
acceleration decreases.(due to decrease in spring force kx)

m kx a
v
(iv) Again block crosses point A due to inertia then from A to B speed decreases and acceleration increases.

kx
a
m v

In this way block does SHM (to be expalined later) if no resistive force is acting on the block.
Note :
N.L.

Release
A
(i)

B
when the block A is released then it take some finite time to reach at B. i.e., spring force doesn’t
change instantaneously.

N.L.

Release
A
m
(2)

When point A of the spring is released in the above situation then the spring forces changes
instantaneously and becomes zero because one end of the spring is free.
(3) In string tension may change instantaneously.

Corporate Head Office : Motion Education Pvt. Ltd., 394 - Rajeev Gandhi Nagar, Kota-5 (Raj.)
NEWTON’S LAW OF MOTION Page # 31
Ex.21 Find out the acceleration of 2 kg block in the figures shown at the instant 1 kg block falls from
2 kg block. (at t = 0)

1kg 1kg

A 2kg 2kg B

Sol. F.B.D.s before fall of 1kg block

1kg 30N (kx) 30N (T)


1kg
2kg
2kg
30(mg)
30(mg)

after the fall of the 1 kg block tension will change instantaneously but spring force (kx) doesn’t
change instantaneously. F.B.D.s just after the fall of 1 kg block

30N (kx) 20
(A) 2kg B 2kg
20 20

30 – 20
aA = = 5 m/s2 (upward) aB = 0 m/s2
2

Ex.22 Two blocks ‘A’ and ‘B’ of same mass ‘m’ attached with a
light spring are suspended by a string as shown in
A m
figure. Find the acceleration of block ‘A’ and ‘B’ just after
the string is cut.

Sol. When block A and B are in equilibrium position


F.B.D of ‘B’ B m
T0

T0=mg
...(i)

mg

: 0744-2209671, 08003899588 | url : www.motioniitjee.com, : info@motioniitjee.com


Page # 32 NEWTON’S LAW OF MOTION
F.B.D of ‘A’

T = mg + T0 .....(ii)
T = 2 mg

mg T0

when string is cut, tension T becomes zero. But spring does not change its shape just after cutting. So
spring force acts on mass B, again draw F.B.D. of block A and B as shown in figure
F.B.D of ‘B’

T0=mg

T0 – mg = m.aB
aB = 0

mg
F.B.D. of ‘A’

mg + T0 = m. aA
2 mg = m. aA
aA = 2g (downwards)
mg T0=mg

Ex.23 Find out the acceleration of 1kg, 2kg and 3kg block and
tension in the string between 1 kg & 2 kg block just after A 1kg
cutting the string as shown in figure.

B 2kg
Sol. F.B.D before cutting of string

6gN
C 3kg
A 1kg
1gN (mg)
5gN
B 2kg
3gN 2gN

3gN(spring force)
C 3kg

3gN(mg)

Let us assume the Tension in the string connecting blocks A & B becomes zero just after cutting the
string then.

Corporate Head Office : Motion Education Pvt. Ltd., 394 - Rajeev Gandhi Nagar, Kota-5 (Raj.)
NEWTON’S LAW OF MOTION Page # 33

2 kg
1 kg
1g 5g
a1 = = g ms–2 a2 = = 2.5 g ms–2
1 2g + 3g 2
1g (weight) (spring force)

 a2 > a1 i.e.,  T0


If T  0 that means string is tight and Both block A & B will have same acceleration. So it will take as a
system of 3 kg mass.

1 kg
10N

T
3kg a
2 kg

30N 20N 60N

System T
Total force down ward = 10 + 30 + 20 = 60 N 20m/s
2

60 2kg
Total mass = 3 kg  a = = 20 m/s2 50 – T = 2 × 20
3
Now apply Fnet = ma at block B. T = 10 N
50

 the spring force does not change instantaneously the F.B.D of ‘C’
3g

2
3kg ac = 0 m/s

3g
Reference Frame :
A frame of reference is basically a coordinate system in which motion of object is analyzed. There are
two types of reference frames.
(a) Inertial reference frame : Frame of reference moving with constant velocity or stationary
(b) Non-inertial reference frame : A frame of reference moving with non-zero acceleration

: (i) Although earth is a non inertial frame (due to rotation) but we always consider it as an inertial
frame.
(ii) A body moving in circular path with constant speed is a non intertial frame (direction change cause
acceleration)

7. PSEUDO FORCE :
Consider the following
example to understand the pseudo force concept
support a
B
m A

The block m in the bus is moving with constant acceleration a with respect to man A at ground. Force

: 0744-2209671, 08003899588 | url : www.motioniitjee.com, : info@motioniitjee.com


Page # 34 NEWTON’S LAW OF MOTION
required for this acceleration is the normal reaction exerted by the support
So, N = ma ..(i)
This block m is at rest with respect to man B who is in the bus (a non intertial frame). So the
acceleration of the block with respect to man B is zero.
N = m(0) = 0 ..(ii)
But the normal force is exerted in a non-inertial frame also. So the equation (ii) is wrong therefore we
conclude that Newton’s law is not valid in non-inertial frame.
If we want to apply Newton’s law in non-inertial frame, then we can do so by using of the cencept
pseudo force.
Pseudo force is an imaginary force, which in actual is not acting on the body. But after applying it on
the body we can use Newton’s laws in non-inertial frames.
This imaginary force is acting on the body only when we are solving the problem in a non-inertial frame
of reference.
In the above example. The net force on the block m is zero with respect to man B after applying the
pesudo force.

ma N
m

N = ma

1. Direction of pseudo force is opposite to the acceleration of frame


2. Magnitude of pseudo force is equal to mass of the body which we are analyzing multiplied by acceleration
of frame
3. Point of application of pseudo force is the centre of mass of the body which we are analysing

Ex.24 A box is moving upward with retardation ‘a’ < g, find the direction and
magnitude of “pseudo force” acting on block of mass ‘m’ placed inside
the box. Also calculate normal force exerted by surface on block 'm'

'Ma' N
Sol. Pseudo force acts opposite to the direction of

acceleration of reference frame. N + ma = mg


(Pseudo
N = mg – ma
pseudo force = ma in upward direction force)

F.B.D of ‘m’ w.r.t box (non-inertial)


mg
Ex.25 Figure shows a pendulum suspended from the roof of a car that has a constant acceleration a
relative to the ground. Find the deflection of the pendulum from the vertical as observed from
the ground frame and from the frame attached with the car.

Corporate Head Office : Motion Education Pvt. Ltd., 394 - Rajeev Gandhi Nagar, Kota-5 (Raj.)
NEWTON’S LAW OF MOTION Page # 35

T

ax =a
 a
Sol. mg
ay = 0
mg

Figure represents free Body diagram of the bob w.r.t ground.

In an inertial frame the suspended bob has an acceleration a caused by the horizontal component of
tension T.
T sin  = ma ...(i)
T cos = mg ...(ii)
From equation (i) and (ii)
a  a
tan  =   = tan–1  
g  g
In a non-inertial frame
T
 a=0
x
ma a
 ma
mg
a=
y 0
mg
Figure represents free Body diagram of bob w.r.t car.
In the non-intertial frame of the car, the bob is in static equilibrium under the action of three froces, T,
mg and ma (pseudo force)
T sin  = ma ...(iii)
T cos  = mg ....(iv)
From equation (iii) and (iv)

a  a
tan  =   = tan–1  g 
g
Ex.26 A pulley with two blocks system is attached to the ceiling of a lift moving upward with an
acceleration a0. Find the deformation in the spring.

K
a0

m2
m1
Sol. Non-Inertial Frame

T T
K
a0 a a
2T

m2T T
m1 m2g m2a0 m g m a (pseudo)
1 1 0
(pseudo)

Let relative to the centre of pulley, m1 accelerates downward with a and m2 accelerates upwards with
a. Applying Newton’s 2nd law.
m1a + m1a0 – T = m1a ...(i)
T – m2g – m2a0 = m2a ...(ii)

: 0744-2209671, 08003899588 | url : www.motioniitjee.com, : info@motioniitjee.com


Page # 36 NEWTON’S LAW OF MOTION
On adding (iv) and (v) we get

 m1 – m2 
a =  m  m  (g + a0) ...(iii)
 1 2

Substituting a in equation (i)


2m1m 2 (g  a0 ) F 2T 4m1m2 ( g  a0 )
We get T = m1  m 2 x = = = (m  m ) k
k k 1 2

Ex.27 All the surfaces shown in figure are assumed to be frictionless. The block of mass m slides on
the prism which in turn slides backward on the horizontal surface. Find the acceleration of the
smaller block with respect to the prism.
A
m
a0


B C
Sol. Let the acceleration of the prism be a0 in the backward direction. Consider the motion of the smaller
block from the frame of the prism The forces on the block are (figure)

N'
a
a0 N

 ma0
N  

mg 
Mg

(i) N normal force


(ii) mg downward (gravity),
(iii) ma0 forward (Psuedo Force)
The block slides down the plane. Components of the forces parallel to the incline give
ma0 cos + mg sin  = ma
or, a = a0 cos  + g sin  ...(i)
Components of the forces perpendicular to the incline give
N + ma0 sin  = mg cos  ...(ii)
Now consider the motion of the prism from the ground frame. No pseudo force is needed as the frame
used is inertial. The forces are (figure)
(i) Mg downward
(ii) N normal to the incline (by the block)
(iii) N’ upward (by the horizontal surface)
Horizontal components give,
N sin  = Ma0 or N = Ma0 / sin , ...(iii)
Putting in (ii)
Ma0
+ ma0 sin  = mg cos 
sin
mg sin  cos 
or, a0 =
M  m sin2 
mg sin  cos 2  (M  m) g sin 
From (i) a = + g sin  =
2
M  m sin  M  m sin 2 

Corporate Head Office : Motion Education Pvt. Ltd., 394 - Rajeev Gandhi Nagar, Kota-5 (Raj.)
NEWTON’S LAW OF MOTION Page # 37
8. WEIGHING MACHING :
A weighing machine does not measure the weight but measures the force. exerted by object on its
upper surface or we can say weighing machine measure normal force on the man.

8.1 Motion in a lift :


(A) If the lift is unaccelerated (v = 0 or constant)
In this case no pseudo force act on the man
N
In this case the F.B.D. of the man
N = mg weighing
In this case machine read the machine
actual weight

(B) If the lift is accelerated upward.


(where a = constant) mg

a
N
weighing
machine

F.B.D of man with respect to lift


So weighing machine read
N = m(g + a)
Apparent weight
N > Actual weight (mg) mg ma(pseudo)

(c) If the lift is accelerated down ward.

a
N ma (pseudo)
weighing
machine

F.B.D of man with respect to lift

So weighing machine read


mg
N = m(g - a)
Apparent weight
N < Actual weight (mg)
Note :
(i) If a = g  N = 0
Thus in a freely falling lift, the man will experience a state of weightlessness
(ii) If the lift is accelerated downwards such that a > g : So the man will be accelerated upward and
will stay at the ceiling of the lift.
(iii) Apparent weight is greater than or less than actual weight only depends on the direction and
magnitude of acceleration. Magnitude and direction of velocity doesn’t play any roll in apparent weight.

: 0744-2209671, 08003899588 | url : www.motioniitjee.com, : info@motioniitjee.com


Page # 38 FRICTION

FRICTION
1. FRICTION :
Friction is a contact force that opposes the relative motion or tendency of relative motion of two
bodies.
Mg

F F

f
N
Consider a block on a horizontal table as shown in the figure. If we apply a force, acting to the right,
the block remains stationary if F is not too large. The force that counteracts F and keeps the block in
rest from moving is called frictional force. If we keep on increasing the force, the block will remain at
rest and for a particular value of applied force, the body comes to state of about to move. Now if we
slightly increase the force from this value, block starts its motion with a jerk and we observe that to
keep the block moving we need less effort than to start its motion.
So from this observation, we see that we have three states of block, first, block does not move,
second, block is about to move and third, block starts moving. The friction force acting in three states
are called static frictional force, limiting frictional force and kinetic frictional force respectively. If we
draw the graph between applied force and frictional force for this observation its nature is as shown in
figure.

1.1 Static frictional force


f
flim b
c d
fkin

a Static region Kinetic region


F
When there is no relative motion between the contact surfaces, frictional force is called static
frictional force. It is a self-adjusting force, it adjusts its value according to requirement (of no relative
motion). In the taken example static frictional force is equal to applied force. Hence one can say that
the portion of graph ab will have a slope of 45º.

The Direction of Static Friction


The direction of static friction on a body is such that the total force acting on it keeps it at rest with
respect to the body in contact.
The direction of static friction is as follows. For a moment consider the surfaces to be frictionless. In
absence of friction the bodies will start slipping against each other. One should then find the direction
of friction as opposite to the velocity with respect to the body applying the friction.

1.2 Limiting Frictional Force


This frictional force acts when body is about to move. This is the maximum frictional force that can
exist at the contact surface.
(i) The magnitude of limiting frictional force is proportional to the normal force at the contact surface.

flim  N  flim = sN

Here s is a constant the value of which depends on nature of surfaces in contact and is called as
‘coefficient of static firction’.

Corporate Head Office : Motion Education Pvt. Ltd., 394 - Rajeev Gandhi Nagar, Kota-5 (Raj.)
FRICTION Page # 39

1.3 Kinetic Frictional Force


Once relative motion starts between the surface in contact, the frictional force is called as kinetic
frictional force. The magnitude of kinetic frictional force is also proportional to normal force.
fk = kN
From the previous observation we can say that k < s
Although the coefficient of kinetic friction varies with speed, we shall neglect any variation
i.e., when relative motion starts a constant frictional force starts opposing its motion.

Direction of Kinetic Friction


The kinetic friction on a body A slipping against another body B is opposite to the velocity of A with
respect to B.
It should be carefully noted that the velocity coming into picture is with respect to the body applying
the force of friction.
f
f v

Suppose we have a long truck moving on a horizontal road. A small block is placed on the truck which
slips on the truck to fall from the rear end. As seen from the road, both the truck and the block are
moving towards right, of course the velocity of the block is smaller than that of the truck. What is the
direction of the kinetic friction acting on the block due to the truck ? The velocity of the block as seen
from the truck is towards left. Thus, the friction on the block is towards right. The friction acting on
the truck due to the block is towards left.

Ex.1 Find the direction of kinetic friction force


F=1N
(a) on the block, exerted by the ground. 1 kg V=5m/s
(b) on the ground, exerted by the block. ///////////////////////////////////
5m/s
F=1N w.r.t to block
Sol. (a) 1 kg 5 m/s (b)
f1 w.r.t to ground f2

where f1 and f2 are the friction forces on the block and ground respectively.

Ex.2 The correct relation between magnitude of f1 and f2 in above problem is :


(A) f1 > f2 (B) f2 > f1 (C) f1 = f2
(D) not possible to decide due to insufficient data.
Sol. By Newton’s third law the above friction forces are action-reaction pair and equal but opposite to each
other in direction. Hence (C)
Also note that the direction of kinetic friction has nothing to do with applied force F.

A 10 m/s
Ex.3 B 20m/s

All surfaces are rough. Draw the friction force on A & B


fkBA
A fkAB B
Sol.
fkBG
Kinetic friction acts to reduce relative motion.
Summary
We can summarise the laws of friction between two bodies in contact as follows:

: 0744-2209671, 08003899588 | url : www.motioniitjee.com, : info@motioniitjee.com


Page # 40 FRICTION
(i) If the bodies slip over each other, the force of friction is given by
fk = k N
where N is the normal contact force and k is the coefficient of kinetic friction between the surfaces.
(ii) The direction of kinetic friction on a body is opposite to the velocity of this body with respect to the
body applying the force of friction.
(iii) If the bodies do not slip over each other, the force of friction is given by
fs s N
where s is the coefficient of static friction between the bodies and N is the normal force between
them. The direction and magnitude of static friction are such that the condition of no slipping between
the bodies is ensured.
(iv) The frictional force fk or fs does not depend on the area of contact as long as the normal force N is
same.

Ex.4 A block of mass 5 kg is resting on a rough surface as shown in the figure. It is acted upon by a
force of F towards right. Find frictional force acting on block when (a) F = 5N (b) 25 N (c) 50 N (s =
0.6, k = 0.5) [g = 10 ms–2]

Sol. Maximum value of frictional force that the surface can offer is
fmax = flim = sN F Mg
= 0.6 × 5 × 10 = 30 newton
Therefore, it F  fmax body will be at rest and f = F
f
or F > fmax body will more and f = fk
(a) F = 5N < Fmax N
So body will not move hence static frictional force will act and ,
fs = f = 5N
(b) F = 25 N < Fmax  fs = 25 N
(c) F = 50 N > Fmax
So body will move and kinetic frictional force will act, its value will be
fk = k N = 0.5 × 5 × 10 = 25 newton

Ex.5 A block having a mass 3 kg is initially at rest on a horizontal surface. The coefficient of static
friction s = 0.3 between the block and the surface and k is 0.25. A constant force F of 50 N, acts
on the body at the angle  = 37º. What is the acceleration of the block ?
F

x
Sol. We have two possibilities here, the block may remain at rest, or it may accelerate towards the right.
The decision hinges on whether or not the x-component of the force F has magnitude, less than or
greaer than the maximum static friction force.
The x-component of F is
Fx = Fcos  = (50 N ) (0.8) = 40 N
To find fs, max, we first calculate the normal force N, whether or not the block accelerates horizontally,
the sum of the y-component of all the forces on the block is zero. N
N - F sin  – mg = 0
f x
or N = F sin  + mg=(50 N) (0.6) + (3 kg)(9.8ms–2) = 59.4 N 
The maximum static frictional force
mg F
fs,max = sN = (0.3) (59.4 N) = 17.8 N
This value is smaller than the x-component of F, hence the block moves. We now interpret the force f
in the figure as a kinetic frictional force. This value is obtained as
fK = k N = (0.25) (59.4 N) = 14.8 N

Corporate Head Office : Motion Education Pvt. Ltd., 394 - Rajeev Gandhi Nagar, Kota-5 (Raj.)
FRICTION Page # 41
Therefore resultant force in the x-direction is

F x  F cos  f = 40 N –14.8 N = 25.2 N


Then the acceleration ‘a’ of the block is
25.2 N
a  8.4 ms 2
3 kg
Think : What would happen if the magnitude of Fx happened to be less than fs.max but larger
than fk ?

Ex.6 In the previous example, suppose we move the block by pulling it with the help of a massless
string tied to the block as shown here. What is the force F required to produce the same
acceleration in the block as obtained in the last example ?
F

m a

Sol. We are given that,


N F sin
m = 3kg, s = 0.3, k = 0.25,  = 37º,
and a = 8.4 ms–2
F cos
In order to determine the force F, we first draw the FBD as shown below
The equations of motion therefore, are f
N + Fsin  = mg mg
N = mg – Fsin 
F cos  – f = ma
and where f = s N before the start of the motion, once motion is set, f = kN.
Hence, force F which produces a = 8.4 m/s2 is given by
Fcos – k (mg – F sin ) = ma

ma  kmg 3( 0.4  0 .25  9 .8 )


or F    34 .26 N
cos   k sin  0.8  0.25  0.6

: Fsin  works out to be less than mg. Otherwise we would lift the block up in the above analysis

COMMENT
It is easier to pull then to push. Only about 34 N force is required to pull than 50 N required during
pushing why ?
Because, when we pull at an angle, the effective normal force N by which block is pressing down on
surface is reduced and consequently friction is reduced. Just the contrary happens when you are
pushing.

2. MINIMUM FORCE REQUIRED TO MOVE THE PARTICLE :

A body of mass m rests on a horizontal floor with which it has a coefficient of static friction . It is
desired to make the body slide by applying the minimum possible force F.
F
m 

Fig. A
Let the applied force F be at angle  with the horizontal

: 0744-2209671, 08003899588 | url : www.motioniitjee.com, : info@motioniitjee.com


Page # 42 FRICTION

R F

R m
R = Normal force
mg

Fig. B
For vertical equilibrium,
R + F sin  = mg or, R = (mg – F sin) ...(i)
For horizontal equilibrium i.e. when the block is just about to slide,
F cos  = R ...(ii)
Substituting for R,
F cos =  (mg – F sin) or F = mg / (cos  +  sin)
for minimum F (cos + sin) is maximum,
 Let x = cos  +  sin
dx
  sin    cos 
d

for maximum of x, dx  0
d
tan  =  and at this value of 

mg
Fmin 
2
1 

3. FRICTION AS THE COMPONENT OF CONTACT FORCE :

When two bodies are kept in contact, electromagnetic forces act between the charged particles at
the surfaces of the bodies. As a result, each body exerts a contact force on other The magnitudes of
the contact forces acting on the two bodies are equal but their directions are opposite and hence the
contact forces obey Newton’s third law.
N=normal force
Fc=contact force

f=friction

The direction of the contact force acting on a particular body is not necessarily perpendicular to the
contact surface. We can resolve this contact force into two components, one perpendicular to the
contact surface and the other parallel to it. The perpendicular component is called the normal contact
force or normal force and parallel component is called friction.

Contact force = f 2  N2
Fc min = N {when fmin = 0}

Fc max =  2N2  N2 {when fmax = N}

N  Fc  (  2  1) N
0  tan–1

Corporate Head Office : Motion Education Pvt. Ltd., 394 - Rajeev Gandhi Nagar, Kota-5 (Raj.)
FRICTION Page # 43
Ex.7 A body of mass 400 g slides on a rough horizontal surface. If the frictional force is 3.0 N, find (a)
the angle made by the contact force on the body with the vertical and (b) the magnitude of the
contact force. Take g = 10 m/s2.
Sol. Let the contact force on the block by the surface be Fc which makes an angle  with the vertical
(shown figure)
Fc N

The component of Fc perpendicular to the contact surface is the normal force N and the component of
F parallel to the surface is the firction f. As the surface is horizontal, N is vertically upward. For vertical
equilirbrium,
N = Mg = (0.400 kg) (10 m/s2) = 4.0 N
The frictional force is f = 3.0 N
f 3
(a) tan   or,  = tan–1 (3/4) = 37º
N 4
(b) The magnituded of the contact force is

F  N2  f 2 = ( 4.0 N) 2  (3.0N) 2  5.0 N

4. MOTION ON A ROUGH INCLINED PLANE


Suppose a motion up the plane takes place under the action of pull P acting parallel to the plane
N = mg cos  P
Frictional force acting down the plane N
F = N =  mg cos 
Appling Newton’s second law for motion up the plane mg sin  mg cos 
P – (mg sin  + f) = ma f  mg
P – mg sin  –  mg cos  = ma
If P = 0 the block may slide downwards with an acceleration a. The frictional force would then act
up the plane
mg sin  – F = ma
or, mg sin  –  mg cos  = ma

Ex.8 A 20 kg box is gently placed on a rough inclined plane of inclination 30° with horizontal. The
coefficient of sliding friction between the box and the plane is 0.4. Find the acceleration of the
box down the incline.
N Y X
F  N
O
mgsin
mg mgcos
Y'
X'
Sol. In solving inclined plane problems, the X and Y directions along which the forces are to be considered,
may be taken as shown. The components of weight of the box are
(i) mg sin  acting down the plane and
(ii) mg cos  acting perpendicular to the plane.
N = mg cos 
mg sin  –  N = ma  mg sin  –  mg cos  = ma
a = g sin  – g cos  = g (sin  –  cos )
1 3
= 9.8 2 – 0.4  2  = 4.9 × 0.3072 = 1.505 m/s2
 
The box accelerates down the plane at 1.505 m/s2.

: 0744-2209671, 08003899588 | url : www.motioniitjee.com, : info@motioniitjee.com


Page # 44 FRICTION

Ex.9 A force of 400 N acting horizontal pushes up a 20 kg block placed on a rough inclined plane which
makes an anlge of 45° with the horizontal. The acceleration experienced by the block is 0.6 m/
s2. Find the coefficient of sliding friction between the box and incline.
Sol. The horizontally directed force 400 N and weight 20 kg of the block are resolved into two mutually
perpendicular components, parallel and perpendicular to the plane as shown.
N = 20 g cos 45° + 400 sin 45° = 421.4 N
The frictional force experienced by the block
2 400 cos45°
a=0.6 m/s
F = N =  × 421.4 = 421.4 N.
R
As the accelerated motion is taking placed up the plane.
400 N
400 cos 45° – 20 g sin 45° – f = 20a

400 20  9.8 400 sin45°


– – 421.4  = 20a = 20 × 0.6 = 12 20g sin45°
2 2 45° 20 g 20 cos 45°

 400 196  1 282.8 – 138.6 – 12


 – – 12  = = 0.3137
 2 2  4214. 4214.

The coefficient of sliding friction between the block and the incline = 0.3137

5. ANGLE OF REPOSE :

Consider a rough inclined plane whose angle of inclination  with ground can be changed. A block of
mass m is resting on the plane. Coefficient of (static) friction between the block and plane is .
For a given angle , the FBD (Free body diagram) of the block is

f
N

mg cos
mg sin

Where f is force of static friction on the block. For normal direction to the plane, we have N=mg cos
As  increases, the force of gravity down the plane, mg sin , increases. Friction force resists the slide
till it attains its maximum value.
fmax = N =  mg cos 
Which decreases with  (because cos  decreases as  increases)
Hence, beyond a critical value  = c, the blocks starts to slide down the plane. The critical angle is the
one when mg sin  is just equal of fmax, i.e., when
mg sin c =  mg cosc
or tan c = 
where C is called angle of repose
If  > c, block will slide down.For  < c the block stays at rest on the incline.

Corporate Head Office : Motion Education Pvt. Ltd., 394 - Rajeev Gandhi Nagar, Kota-5 (Raj.)
FRICTION Page # 45

6. TWO BLOCKS ON AN INCLINED PLANE :


Consider two blocks having masses m1 & m2 placed on a rough inclined plane. 1 & 2 are the friction
coefficient for m1 & m2 respectively. If N is the normal force between the contact surface of m1 & m2

2
m
N
1
m
N

Now three condition arises.
(i) If 1 = 2 =  then
N = 0 because, Both the blocks are in contact but does not press each other.
a1 = a2 = g sin  –  mg cos 
(a1,a2 are acceleration of block 1 & 2 respectively)
(ii) If 1 < 2 then
N = 0 because, there is no contact between the blocks.
a1 = g sin  – 1 g cos 
a2 = g sin  – 2 g cos 
 a1 > a2
(iii) If 1 > 2 then N0
a1 = a2

Ex.10 Mass m1 & m2 are placed on a rough inclined plane as shown in figure. Find out the acceleration
of the blocks and contact force in between these surface.

m2
2kg
m1
1kg
2 =0.2
1=0.5
37°

Sol. As we know if 1 > 2 both will travel together so


a1 = a2 = a
F.B.D
.2
=3

°
37
os
gc
m2
2
m2 =
f2
m1 f1 = 1m1gcos37° =4
°
37
s in
g °
m2 37
in
g1 s 37°
m
f1 + f2
g

which is equivalent to
3k

3g sin 37–( f1  f2 ) 18 – 7.2


a= a= = 3.6 m/sec2 °
3 2 n37
si 37°
Now F.B.D of 1 kg block is 3g

: 0744-2209671, 08003899588 | url : www.motioniitjee.com, : info@motioniitjee.com


Page # 46 FRICTION

N
4N 2
c
m/se
3. 6
a=

sin3
g

gsin37° + N – 4 = (1) a
N = 3.6 + 4 – 6 = 1.6 Newton

7. RANGE OF FORCE F FOR WHICH ACCELERATION OF BODY IS ZERO.


F
g
2k

Ex.11
37º

Find out the range of force in the above situation for which 2kg block does not move on the
incline.
Sol. F.B.D of 2 kg block

F
=8N
º
37
os

x
fma
gc
m

2kg
F

)
0 .5
=(
g

ax
2k

fm
3 7º
in

N
gs

37º
12
m

Now take different value of F


Force (F) F.B.D. Acceleration Direction & magnitude Friction Type
Fnet
a
m
8N

(i) F = 0N 2m/s2 8N Kinetic


N
12
4N
8N

(ii) F = 4N 0m/s2 8N Static


N
12
8N
4N

(iii) F = 8N 0m/s2 4N Static


N
12

Corporate Head Office : Motion Education Pvt. Ltd., 394 - Rajeev Gandhi Nagar, Kota-5 (Raj.)
FRICTION Page # 47

N
12
0
f=
(iv) F = 12N 0m/s2 0 Static
N
12

N
16
(v) F = 16N 0m/s2 4N Static

N
12
4N
f=

(in this condition friction change its direction to stop relative slipping)
N
20

(vi) F = 20N 0m/s2 8N Static

N
12
8N

N
24

(vii) F = 24N 2m/s2 8N Kinetic

N
12
8N

From the above table block doesn't move from F = 4N (mgsin – mgcos) to F = 20N (mgsin +
mgcos). So friction develope a range of force for which block doesn't move

: If Friction is not present then only for F = 12N the block will not move but friction develop a range of
force 4N to 20N to prevent slipping. So we can write the range of force F for which acceleration of the
body is zero.
mg sin  – mg cos   F  mg sin  +  mg cos.

Ex.12 In the following figure force F is gradually increased from zero. Draw the graph between applied
force F and tension T in the string. The coefficient of static friction between the block and the
ground is s.

F M

s
Sol. As the external force F is gradually increased from zero it is T
compensated by the friction and the string beares no tension. 45°
When limiting friction is achieved by increasing force F to a
value till s mg, the further increase in F is transferred to the F
string. smg

Ex.13 Fig. shows two blocks tied by a string. A variable force F = 5t is applied on the block. The coefficient
of friction for the blocks are 0.6 and 0.5 respectively. Find the frictional force between blocks
and ground as well as tension in the string at

: 0744-2209671, 08003899588 | url : www.motioniitjee.com, : info@motioniitjee.com


Page # 48 FRICTION
(A) t = 1 s (B) t = 2s (C) t = 3s
B A
1kg 2kg
F=5t
=0.6 =0.5

10N 20N

T
T 5t
Sol. fA
fB

10N 20N

(a) At t = 1s, F = 5 × 1 = 5 N
Maximum value of friction force
fA = N = 0.5 × 20 = 10 N
To keep the block stationary the magnitude of frictional force should be 5N. So
fA = 5 N
Now from the figure it becomes clear that if
fA = 5N & F = 5 N, Tension T = 0
Since tension is not in application so frictional force on block B is 0 i.e.,
fB = 0
(b) At t = 2s, F = 5 × 2 = 10 N
Maximum value of friction force
f = N = 0.5 × 20 = 10 N
To keep the block stationary the magnitude of friction force should be 10 N. So
fA = 10 N
From the figure it is clear that if
fA = 10 N and F = 10 N
Tension T = 0
Hence friction force on block B is fB = 0
(c) At t = 3s, F = 5 × 3 = 15 N
Maximum value of friction force
f = N = 0.5 × 20 = 10 Newton
Again applying the same analogy fA = 10 N
From the figure it is clear that if
fA = 10 N and F = 15 N
Tension T = 5 N
So frictional force on block B is fB = 5 Newton
Ex.14 Find the tension in the string in situation as shown in the figure below. Forces 120 N and 100 N
start acting when the system is at rest.

120 N 10 20 100N

fsmax=90 N
fsmax=60 N

Sol. (i) Let us assume that system moves towards left then as it is clear from FBD, net force in horizontal
direction is towards right. Therefore the assumption is not valid.

Corporate Head Office : Motion Education Pvt. Ltd., 394 - Rajeev Gandhi Nagar, Kota-5 (Raj.)
FRICTION Page # 49

120 N 10 20 100N

90 N 60 N
Above assumption is not possible as net force on system comes towards right. Hence system is not
moving towards left.
(ii) Similarly let us assume that system moves towards right.

120 N 10 20 100N

90 N 60 N
Above assumption is also not possible as net force on the system is towards left in this situation.
Hence assumption is again not valid.

Therefore it can be concluded that the system is stationary


T
120 N 10 20 100N
Fmax=90 N fmax=60 N
Assuming that the 10 kg block reaches limiting friction first then using FBD’s

10 T T 20
120 N 100N
90N
f
120 = T + 90  T = 30 N
Also T + f = 100
 30 + f = 100  f = 70 N
which is not possible as the limiting value is 60 N for this surface of block.
 Our assumption is wrong and now taking the 20 kg surface to be limiting we have

10 T T 20
120 N 100N
f 60 N
T + 60 = 100 N  T = 40 N
Also f + T = 120 N  f = 80 N
This is acceptable as static friction at this surface should be less than 90 N.
Hence the tension in the string is T = 40 N

8. PULLEY BLOCK SYSTEM INVOLVING FRICTION :


• If friction force is acting and value of acceleration of a particle is negative, then it means direction of
friction force is opposite to that what we assumed and acceleration would be having a different
numerical value.

Ex.15 Two blocks of masses 5 kg and 10 kg are attached with the help of light string and placed on a
rough incline as shown in the figure. Coefficients of friction are as marked in the figure. The
system is released from rest. Determine the acceleration of the two blocks.

: 0744-2209671, 08003899588 | url : www.motioniitjee.com, : info@motioniitjee.com


Page # 50 FRICTION

10kg 5kg

37° 50°
Fixed

Sol. Let 10 kg block is sliding down, then acceleration of both the blocks are given by,

10g sin 37–  1  10g cos 37–5g sin 53


–  2  5g cos 53
a  – ve
15
It means our assumed direction of motion is wrong and 5 kg block is going to slide down, if this would
be the case, the direction of friction force will reverse and acceleration of blocks would be given by :

5g sin 53 –  2  5g cos 53 –  1  10g cos 37


– 10g sin 37 = –ve
a1 
15
It means in this direction also there is no motion. So we can conclude that the system remains at rest
and friction force is static in nature.

9. TWO BLOCK SYSTEM :

Ex.16 B 2kg
A 4kg F

frictionless
Find out the maximum value of F for which both the blocks will move together
Sol. In the given situation 2kg block will move only due to friction force exerted by the 4 kg block
F.B.D.

B 2kg f
f A 4kg F

The maximum friction force exerted on the block B is


fmax = N
fmax = (0.5) (20) = 10 N
So the maximum acceleration of 2 kg block is
2kg f max = 10N
10
a 2 max   5m / s 2
2

amax is the maximum acceleration for which both the block will move together. i.e., for a  5 ms–2
acceleration of both blocks will be same and we can take both the blocks as a system.

Corporate Head Office : Motion Education Pvt. Ltd., 394 - Rajeev Gandhi Nagar, Kota-5 (Raj.)
FRICTION Page # 51
F.B.D

a 2
5m/s
6kg Fmax

Fmax = 6 × 5 = 30 N
for 0 < F < 30
Both the block move together.

Ex.17 In the above question find the acceleration of both the block when
(i) F = 18 N (ii) F = 36 N
Sol. (i) Since F < 30 both the blocks will move together
F.B.D

6kg F = 18N

18
a  3 m / s2
6
(ii) When F = 36 N
When F > 30 both the blocks will move separately so we treat each block independently
F.B.D of 2 kg block

B 2kg f = 10N (Friction force)

aB = 5 m/s2
F.B.D of 4 kg block
f = 10N 4kg F = 36N
A

36  10 26
aA   m / s2
4 4

B 2kg

A 4kg F
Ex.18

Find out the range of force in which both the blocks move together
Sol. If f1 is friction force between block A & lower surface and f2 is friction force between both the block’s
surface.
F.B.D
B 2kg f 2=10N
f2=10N
6N = f1 A 4kg F

f1 max = 1N1 = (0.1) (60) = 6 N


f2 max = 2N2 = (0.5) (20) = 10 N

Upper 2kg block is move only due to friction force so maximum acceleration of that block is

2kg f 2 =10N

: 0744-2209671, 08003899588 | url : www.motioniitjee.com, : info@motioniitjee.com


Page # 52 FRICTION

10
amax   5 m / s2
2
This is the maximum acceleration for which both the blocks will move together.
Therefore for a  5ms–2 we can take both the blocks as one system.
F.B.D.
2
5m/s
6kg F
f1=6N

For F < 6 N. Blocks will not move at all.


Now the value of Fmax for which both the blocks will move together.
Fmax – 6 = 6 × 5
Fmax = 36 N
Conclusion if
0N < F < 6N No blocks will move
6N < F < 36 N Both blocks will move together
F > 36 N Both move separately.

Ex.19 B 2kg F

A 4kg
frictionless

The lower block A will move only due to friction force


F.B.D.
f 2kg F

4kg f(frictional force)

fmax = N = (0.5) (20) = 10 N

F.B.D. of 4 kg blocks
amax

4kg f = 10N

The maximum acceleration of 4 kg block is


10
 amax   2.5 m / s 2
4
This is the maximum acceleration for which both the blocks move together
2
2.5 m/s

6kg F

Fmax for which both the blocks will move together


Fmax = 2.5 × 6 = 15 N

B 2kg F

Ex.20 A 4kg

Corporate Head Office : Motion Education Pvt. Ltd., 394 - Rajeev Gandhi Nagar, Kota-5 (Raj.)
FRICTION Page # 53

If f2 is the friction force between A & B and f1 is the friction force between A & floor
f1 max = 6 N
f2 max = 10 N
Lower block A will move only due to friction force So amax for 4 kg block

4kg f2 = 10N
f1 =6N
10  6
amax   1 m / s2
4
This is the maximum acceleration for which both the blocks will move together
2
1m/s

6kg F
f1 =6N
F–6=6×1
F = 12 N
If F is less than 6N both the blocks will be stationary
Conclusion :
0 < F < 6 N = Both blocks are stationary
6 N < F < 12 N = Both move together
F > 12 N = Both move separately

Ex.21 Find the accelerations of blocks A and B for the following cases. 1
2
(A) 1 = 0 and 2 = 0.1 (P) aA = aB = 9.5 m/s2
(B) 2 = 0 and 1 = 0.1 (Q) aA = 9 m/s2,
aB = 10 m/s2 1 kg 10 N
1kg
(C) 1 = 0.1 and (R) aA = aB = g = 10 m/s2
2 = 1.0 A B
(D) 1 = 1.0 and (S) aA = 1, aB = 9 m/s2
2 = 0.1
Sol. (a) R, (b) Q, (c) P, (d) S
(i) FBD in (case (i))
{1 = 0, 2 = 0.1}
O 2 N

N=10 1kg N=10


1kg
A B

mg 2N mg

While friction’s work is to oppose the relative motion and here if relative motion will start then friction
comes and without relative motion there is no friction so both the block move together with same
acceleration and friction will not come.

A B
 aA = aB = 10 m/s2
mg mg

: 0744-2209671, 08003899588 | url : www.motioniitjee.com, : info@motioniitjee.com


Page # 54 FRICTION

1 0

A 10 B 10
(ii) 1kg 1kg
1
10 0
10
Friction between wall and block A oppose relative motion since wall is stationary so friction wants to
slop block A also and maximum friction will act between wall and block while there is no friction
between block.

: Friction between wall and block will oppose relative motion between wall and block only it will not do
anything for two block motion.
1

A B

10 10
aA = 9 m/s ; aB = 10 m/s2
2

1 f

(iii) A B
f
10
10
10

Friction between wall and block will be applied maximum equal to 1N but maximum friction available
between block A and B is 10 N but if this will be there then relative motion will increase while friction
is to oppose relative motion. So friction will come less than 10 so friction will be f that will be static.
1 f

A B
f
10 10
19
by system (20–1) = 2 × a  a = =9.5 m/s2
2 1
10
11 – 10
(iv) aA =  1m / s 2
1 A B
10 – 1
aB =  9 m / s2 10 1
1 10
10

Corporate Head Office : Motion Education Pvt. Ltd., 394 - Rajeev Gandhi Nagar, Kota-5 (Raj.)
FRICTION Page # 55

10. FRICTION INVOLVING PSEUDO CONCEPT :

Ex.22 What is the minimum acceleration with which bar A should be


1
shifted horizontally to keep the bodies 1 and 2 stationary
relative to the bar ? The masses of the bodies are equal and
the coefficient of friction between the bar and the bodies equal
to . The masses of the pulley and the threads are negligible A 2
while the friction in the pulley is absent. see in fig.
Sol. Let us place the observer on A.
Since we have non-inertial frame we have pseudo forces.
For body ‘1’ we have, T
ma
T = ma + mg ....(1) 1
For body ‘2’ we have, mg
T
N
N = ma
A ma N
mg – T – ma = 0 2
 mg = T + mg ....(2)
mg
From (1) and (2) amin  1  
 g 
 1   a

=0.5
F M=4kg m=
Ex.23 1kg

Find out the range of force for which smaller block is at rest with respect to bigger block.
Sol. Smaller block is at rest w.r.t. the bigger block. Let both the block travel together with acceleration a
F.B.D of smaller block w.r. to the bigger block.
f
fmax = × N
N = ma
f = ma ...(1) ma N
(Pseudo)
 f = mg ...(2)
from (1) & (2)
a = g/ = 20 m/s2 mg
So F = 20 (M + m) = 20 (5) = 100 N
If F  100 N Both will travel together

Ex.24 The rear side of a truck is open and a box of 40 kg mass is placed 5m away from the open end as
shown. The coefficient of friction between the box & the surface below it is 0.15. On a straight
road, the truck starts from rest and accelerates with 2 ms–2 . At what distance from the starting
point does the box fall off the truck (i.e. distance travelled by the truck) ? [Ignore the size of
the box]

/////////////////////////////////////////

: 0744-2209671, 08003899588 | url : www.motioniitjee.com, : info@motioniitjee.com


Page # 56 FRICTION
Sol. In the reference frame of the truck FBD of 40 kg block
40 ma
N kg (psuedo force)

15
Net force  ma – N  40 × 2 – × 40 × 10
100

20 1
mablock  80 – 60  ablock = = m/s2
40 2
This acceleration of the block in reference frame of truck so time taken by box to fall down from
truck
1 1 1
Srel = urelt + a t2  5 = 0 + × × t2  t2 = 20
2 rel 2 2
So distance moved by the truck

1 1
 × atruck × t2  × 2 × (20) = 20 meter..
2 2

Ex.25 Mass m2 placed on a plank of mass m1 lying on a smooth horizontal plane. A horizontal force
F = 0t (0 is a constant) is applied to a bar. If acceleration of the plank and bar are a1 and a2
respectively and the coefficient of friction between m1 and m2 is . Then find acceleration a
with time t.

m2 F

m1

Sol. If F < m2g then both blocks move with common acceleration, i.e., a1 = a2
When F > m2g, then
Equation for block of mass m
F – m2g = m2a2 ...(1)
and m2g = m1a1 ...(2) a2
From equation (1)
0t – m2g = m2a2 a1
a
2
=a

i.e., acceleration a2 varies with time linearly,


1
a

its slope positive and intercept negative.


0 t0 t
From equation (2) a1 is independent of time.
So, the graph between a & t is as follow.

Corporate Head Office : Motion Education Pvt. Ltd., 394 - Rajeev Gandhi Nagar, Kota-5 (Raj.)
N.L.M. , FRICTION Page # 57

Exercise - I OBJECTIVE PROBLEMS (JEE MAIN)


(A) NEWTON'S LAW OF MOTION
1. A man getting down a running bus, falls
forward because-
(A) due to inertia of rest, road is left behind
and man reaches forward
(B) due to inertia of motion upper part of body
continues to be in motion in forward direction
while feet come to rest as soon as they touch
the road
(C) he leans forward as a matter of habbit (A) 0 N, 0 N (B) 300 N, 200 N
(D) of the combined effect of all the three (C) 300 N, 1000 N (D) 2000 N, 300 N
factors stated in (A), (B) and (C) Sol.
Sol.

2 You are on a friction less horizontal plane. How


can you get off if no horizontal force is exerted by 4. At a given instant, A is moving with velocity of 5
pushing against the surface ? m/s upwards. What is velocity of B at the time
(A) by jumping
(B) by spitting or sneezing
(C) by rolling your body on the surface
(D) by running on the plane A
Sol.
B
(A) 15 m/s (B) 15 m/s (C) 5 m/s  (D) 5 m/s 
Sol.

3. The forces acting on an object are shown in the


fig. If the body moves horizontally at a constant
speed of 5 m/s, then the values of the forces P
and S are, respectively-

: 0744-2209671, 08003899588 | url : www.motioniitjee.com, : info@motioniitjee.com


Page # 58 N.L.M. , FRICTION
5. Find the velocity of the hanging block if the 7. A body of mass 5 kg is suspended by the strings
velocities of the free ends of the rope are as indicated making angles 60º and 30º with the horizontal -
in the figure.

2m/s

1m/s

(a) T1 = 25 N (b) T2 = 25 N
(A) 3/2 m/s  (B) 3/2 m/s  (c) T1 = 25 3 N (d) T2 = 25 3 N
(C) 1/2 m/s  (D) 1/2 m/s  (A) a, b (B) a, d
Sol. (C) c, d (D) b, c
Sol.

6. In the figure shown the velocity of different blocks 8. Find velocity of block 'B' at the instant shown in
is shown. The velocity of C is figure.

37°
B
6 m/s

4 m/s
6m/s

A 10 m/s
A B C D
(A) 25 m/s (B) 20 m/s
(A) 6 m/s (B) 4 m/s (C) 22 m/s (D) 30 m/s
(C) 0 m/s (D) none of these Sol.
Sol.

Corporate Head Office : Motion Education Pvt. Ltd., 394 - Rajeev Gandhi Nagar, Kota-5 (Raj.)
N.L.M. , FRICTION Page # 59
9.In the arrangement shown in fig. the ends P and Q 11. The 50 kg homogeneous smooth sphere rests on
of an unstretchable string move downwards with the 30° incline A and bears against the smooth vertical
uniform speed U. Pulleys A and B are fixed. Mass M wall B. Calculate the contact forces at A and B.
moves upwards with a speed.
A B

A B
 30°

Q 1000 500
P
(A) NB = N, N = N
A
3 3

M 1000 500
(B) NA = N, N = N
B
(A) 2 U cos  (B) U cos  3 3
2U U 100 500
(C) (D) (C) NA = N, N = N
cos  cos  3 B
3
Sol.
1000 50
(D) NA = N, N = N
B
3 3
Sol.

10. The velocity of end ‘A’ of rigid rod placed between


two smooth vertical walls moves with velocity ‘u’ along
12. Find out the reading of the weighing machine in
vertical direction. Find out the velocity of end ‘B’ of
the following cases.
that rod, rod always remains in constant with the
vertical walls.

A g 2k
2k W g
M M
W
'u' 30º 30º

B (A) 10 3 (B) 10 2 (C) 20 3 (D) 30 3


Sol.
(A) u tan 2 (B) u cot 
(C)u tan  (D) 2u tan 
Sol.

: 0744-2209671, 08003899588 | url : www.motioniitjee.com, : info@motioniitjee.com


Page # 60 N.L.M. , FRICTION
13. A mass M is suspended by a rope from a rigid 15. Three blocks A, B and C are suspended as shown
support at A as shown in figure. Another rope is tied in the figure. Mass of each blocks A and C is m. If
at the end B, and it is pulled horizontally with a force system is in equilibrium and mass of B is M, then :
F. If the rope AB makes an angle  with the vertical in
equilibrium, then the tension in the string AB is :

A
 B
F B
A C

M (A) M = 2m (B) M < 2 m (C) M > 2m (D) M = m


(A) F sin  (B) F/sin  (C) F cos  (D) F/cos  Sol.
Sol.

16.A weight can be hung in any of the following four


14. Three block are connected as shown, on a ways by string of same type. In which case is the
horizontal frictionless table and pulled to the right string most likely to break ?
with a force T3 = 60 N. If m1 = 10 kg, m2 = 20 kg
and m3 = 30 kg, the tension
T2 is-
(A) W
(B) (C)
W (D)
W
W
(A) 10 N (B) 20 N
(A) A (B) B (C) C (D) D
(C) 30 N (D) 60 N Sol.
Sol.

Corporate Head Office : Motion Education Pvt. Ltd., 394 - Rajeev Gandhi Nagar, Kota-5 (Raj.)
N.L.M. , FRICTION Page # 61

17. A force-time graph for a linear motion is shown 19.A stunt man jumps his car over a crater as shown
in figure where the segments are circular. The (neglect air resistance)
linear momentum gained between zero and 8 sec- (A) during the whole flight the driver experiences
onds in - weightlessness
(B) during the whole flight the driver never experiences
weightlessness
Force Newton

+2 (C) during the whole flight the driver experiences


weightlessness only at the highest point
Time
2 4 6 8
(sec)
–2
(D) the apparent weight increases during upward
journey
(A) – 2 N.s (B) 0 N.s Sol.
(C) 4 N.s (D) – 6  N.s
Sol.

18. A particle moves in the xy plane under the


acti on of a force F such that the value of its
linear momentum (P) at any time t is, Px = 2
cost, Py = 2 sint. The angle  between P and F
at that time t will be -
(A) 0º (B) 30º (C) 90º (D) 180º
Sol.
20. A particle of mass 50 gram moves on a straight
line. The variation of speed with time is shown in figure.
find the force acting on the particle at t = 2, 4 and 6
seconds.
v(m/s)

15

10

5
0 2 4 6 8 t(s)
(A) 0.25 N along motion, zero, 0.25 opposite to motion
(B) 0.25 N along motion, zero, 0.25 along to motion
(C) 0.25 N opposite motion, zero, 0.25 along to motion
(D) 0.25 N opposite motion, zero, 0.25 opposite to
motion

: 0744-2209671, 08003899588 | url : www.motioniitjee.com, : info@motioniitjee.com


Page # 62 N.L.M. , FRICTION
Sol. Sol.

21. Two blocks are in contact on a frictionless table.


One has mass m and the other 2m. A force F is applied
on 2m as shown in the figure. Now the same force F is
applied from the right on m. In the two cases 23. A rope of mass 5 kg is moving vertically in vertical
respectively, the ratio force of contact between the position with an upwards force of 100 N acting at the
two block will be : upper end and a downwards force of 70 N acting at
the lower end. The tension at midpoint of the rope is
F (A) 100 N (B) 85 N (C) 75 N (D) 105 N
2m m F 2m m
Sol.

(A) same (B) 1 : 2 (C) 2 : 1 (D) 1 : 3


Sol.

22. A body of mass 8 kg is hanging another body of


mass 12 kg. The combination is being pulled by a string
with an acceleration of 2.2 m s–2. The tension T1 and
T2 will be respectively : (use g = 9.8 m/s2)

T1
12kg
24. A particle of small mass m is joined to a very
a heavy body by a light string passing over a light pulley.
T2 Both bodies are free to move. The total downward
force in the pulley is
8kg
(A) mg (B) 2 mg
(A) 200 N, 80 N (B) 220 N, 90 N (C) 4 mg (D) can not be determined
(C) 240 N, 96 N (D) 260 N, 96 N

Corporate Head Office : Motion Education Pvt. Ltd., 394 - Rajeev Gandhi Nagar, Kota-5 (Raj.)
N.L.M. , FRICTION Page # 63

Sol. 26. Two masses M1 and M2 are attached to the ends


of a light string which passes over a massless pulley
attached to the top of a double inclined smooth plane
of angles of inclination and . The tension in the
string is :

M2
M1 fixed
 

M2 (sin ) g M1(sin  ) g
(A) (B)
M1  M2 M1  M2

M1 M2 (sin   sin ) g
(C) M1  M2 (D) zero
25. The pulley arrangements shown in figure are
identical the mass of the rope being negligible. In case Sol.
I, the mass m is lifted by attaching a mass 2m to the
other end of the rope. In case II, the mass m is lifted
by pulling the other end of the rope with cosntant
downward force F = 2mg, where g is acceleration due
to gravity. The acceleration of mass in case I is

F=2mg

m 2m m

(I) (II)
(A) zero
(B) more than that in case II
(C) less than that in case II
(D) equal to that in case II
Sol. 27. Two masses are hanging vertically over
frictionless pulley. The acceleration of the two
masses is-

m1
(A) g
m2

m2
(B) m g
1

 m 2  m1 
(C)  m  m  g
 1 2

 m1  m2 
(D)  g

 m 2  m1 

: 0744-2209671, 08003899588 | url : www.motioniitjee.com, : info@motioniitjee.com


Page # 64 N.L.M. , FRICTION
Sol. Sol.

28. Three equal weights A, B, C of mass 2 kg each


are hanging on a string passing over a fixed
frictionless pulley as shown in the fig. The tension 30 Two objects A and B of masses mA and mB are
in the string connecting weights B and C is- attached by strings as shown in fig. If they are
(A) zero given upward acceleration, then the ratio of tension
(B) 13 Newton T1 : T2 is -
(C) 3.3 Newton
(D) 19.6 Newton

Sol.

(A) (mA + mB)/mB (B) (mA + mB)/mA


m A  mB m A  mB
(C) m  m (D) m  m
A B A B

Sol.

29. In the arrangement shown in figure, pulleys are


massless and frictionless and threads are inextensible.
The Block of mass m1 will remain at rest, if

m1 m2 m3
1 1 1
(A) m  m  m (B) m1 = m2 + m3
1 2 3

4 1 1 1 2 3
(C) m  m  m (D) m  m  m
1 2 3 3 2 1

Corporate Head Office : Motion Education Pvt. Ltd., 394 - Rajeev Gandhi Nagar, Kota-5 (Raj.)
N.L.M. , FRICTION Page # 65

31. In the figure a smooth pulley of negligible 33. A monkey of mass 20 kg is holding a vertical rope.
weight is suspended by a spring balance. Weights The rope can break when a mass of 25 kg is suspended
of 1kg and 5 kg are attached to the opposite ends from it. What is the maximum acceleration with which
of a string passing over the pulley and move with the monkey can climb up along the rope?
acceleration because of gravity. During the motion, (A) 7 ms–2 (B) 10 ms–2 (C) 5 ms–2 (D) 2.5 ms–2
Sol.
the spring balance reads a weight of -
(A) 6 kg
(B) less than 6 kg
(C) more than 6 kg
(D) may be more or less
than 6 kg
Sol.

34. Fi nd the accel eration of 3 kg mass when


acceleration of 2 kg mass is 2 ms–2 as shown in figure.

3 kg 2 kg 10N
–2
2ms
(A) 3 ms–2 (B) 2 ms–2 (C) 0.5 ms–2 (D) zero
32. Both the blocks shown here are of mass m and Sol.
are moving with constant velocity in direction shown
in a resistive medium which exerts equal constant force
on both blocks in direction opposite to the velocity.
The tension in the string connecting both of them will
be (Neglect friction)

(A) mg (B) mg/2 (C) mg/3 (D) mg/4


Sol.

: 0744-2209671, 08003899588 | url : www.motioniitjee.com, : info@motioniitjee.com


Page # 66 N.L.M. , FRICTION
35. In the figure at the free end a force F is applied to Sol.
keep the suspended mass of 18 kg at rest. The value
of F is-
(A) 180 N
(B) 90 N
(C) 60 N
(D) 30 N

Sol.

38. A trolley is accelerating down an incline of angle 


with acceleration gsin. Which of the following is
correct. ( is the constant angle made by the string
with vertical)


in
m

gs

36. If the tension in the cable supporting an
elevator is equal to the weight of the elevator, the
elevator may be -

(a) going up with increasing speed
(b) going down with increasing speed (A)  = 
(c) going up with uniform speed (B)  = 0º
(d) going down with uniform speed (C) Tension in the string, T = mg
(A) a, d (B) a, b, c (D) Tension in the string, T = mg sec 
(C) c, d (D) a, b Sol.
Sol.

39. A block of mass m resting on a wedge of angle 


as shown in the figure. The wedge is given an
acceleration a. What is the minimum value of a so
that the mass m falls freely ?
37. As an inclined plane is made slowly horizontal
A
by reducing the value of angle  with horizontal.
The component of weight parallel to the plane of m
a block resting on the inclined plane-
(A) decreases a
(B) remains same
(C) increases 
(D) increases if the plane is smooth B C
(A) g (B) g cos  (C) g cot  (D) g tan 

Corporate Head Office : Motion Education Pvt. Ltd., 394 - Rajeev Gandhi Nagar, Kota-5 (Raj.)
N.L.M. , FRICTION Page # 67
Sol. Sol.

(FRICTION)
40. A block is placed on a rough floor and a 42. A block A kept on an inclined surface just
horizontal force F is applied on it. The force of begins to slide if the inclination is 30º. The block
friction f by the floor on the block is measured for is replaced by another block B and it is found that
different values of F and a graph is plotted between it just begins to slide if the inclination is 40º.
them -
(A) mass of A > mass of B
(a) The graph is a straight line of slope 45° (B) mass of A < mass of B
(b) The graph is straight line parallel to the F axis (C) mass of A = mass of B
(c) The graph is a straight line of slope 45º for small (D) all the three are possible
F and a straight line parallel to the F-axis for largeF. Sol.
(d) There is small kink on the graph
(A) c, d (B) a, d
(C) a, b (D) a, c
Sol.

41. Mark the correct statements about the friction


between two bodies -
(a) static friction is always greater than the kinetic
friction
(b) coefficient of static friction is always greater
than the coefficient of kinetic friction 43. Two cars of unequal masses use similar tyres.
If they are moving at the same initial speed, the
(c) limiting friction is always greater than the kinetic
minimum stopping distance -
friction
(A) is smaller for the heavier car
(d) limiting friction is never less than static friction (B) is smaller for the lighter car
(A) b, c, d (B) a, b, c (C) is same for both cars
(C) a, c, d (D) a, b, d (D) depends on the volume of the car

: 0744-2209671, 08003899588 | url : www.motioniitjee.com, : info@motioniitjee.com


Page # 68 N.L.M. , FRICTION
Sol. 45. A body is placed on a rough inclined plane of
inclination . As the angle  is increased from 0º to
90º the contact force between the block and the
plane
(A) remains constant
(B) first remains constant then decreases
(C) first decreases then increases
(D) first increases then decreases
Sol.

44. A monkey of mass m is climbing a rope hanging


from the roof with acceleration a. The coefficient of
static friction between the body of the monkey and 46 A wooden block of mass m resting on a rough
the rope is . Find the direction and value of friction horizontal table (coefficient of friction = ) is pulled
force on the monkey. by a force F as shown in figure. The acceleration of
the block moving horizontally is :
F


m

F cos  F sin 
(A) Upward, F = m(g + a) (A) (B)
m M
(B) downward, F = m(g + a)
(C) Upward, F = mg F
(C) (cos    sin ) – g (D) none
(D) downward, F = mg m
Sol. Sol.

Corporate Head Office : Motion Education Pvt. Ltd., 394 - Rajeev Gandhi Nagar, Kota-5 (Raj.)
N.L.M. , FRICTION Page # 69

47. A chain is lying on a rough table with a 49. A block of mass 2kg rests on a rough inclined
fraction 1/n of its length hanging down from the plane making an angle of 30º with the horizontal. The
coefficient of static friction between the block and
edge of the table. if it is just on the point of sliding
the plane is 0.7. The frictional force on the block is-
down from the table, then the coefficient of friction (A) 0.7 × 9.8 Newton
between the table and the chain is - (B) 9.8 Newton
1 1 (C) 0.7 × 9.8 3 Newton
(A) (B)
n (n  1)
(D) 9.8 × 3 Newton
1 n1 Sol.
(C) (D)
(n  1) (n  1)
Sol.

50. A box 'A' is lying on the horizontal floor of the


compartment of a train running along horizontal rails
from left to right. At time 't', it decelerates. Then the
reaction R by the floor on the box is given best by
R R
48. A body of mass m moves with a velocity v on a A A
(A) (B)
surface whose friction coefficient is . If the body floor floor
covers a distance s then v will be :
R
(A) 2gs (B) gs (C) gs / 2 (D) 3gs R
Sol. (C) A (D) A
floor floor
Sol.

: 0744-2209671, 08003899588 | url : www.motioniitjee.com, : info@motioniitjee.com


Page # 70 N.L.M. , FRICTION
51. For the equilibrium of a body on an inclined plane Sol.
of inclination 45º. The coefficient of static friction will
be
(A) greater than one (B) less than one
(C) zero (D) less than zero
Sol.

54. A block moves down a smooth inclined plane


of inclination . Its velocity on reaching the bottom
is v. If it slides down a rough inclined plane of
some inclination, its velocity on reaching the bottom
is v/n, where n is a number greater than 0. The
coefficient of friction is given by -
FG1  1 IJ
(A)  = tan 
H nK 2

52. Starting from rest a body slides down a 45° inclined F 1I


(B) = cot  G 1  J
plane in twice the time it takes to slide down the H nK 2
same distance in the absence of friction. The co-
efficient of friction between the body and the inclined F 1I
(C)  = tan  G 1  J
1/ 2

plane is : H nK 2
(A) 0.75 (B) 0.33 (C) 0.25 (D) 0.80
Sol. F 1I
(D) = cot  G 1  J
1/ 2

H nK 2

Sol.

55. A block of mass 5 kg and surface area 2 m2 just


begins to slide down an inclined plane when the angle
of inclination is 30°. Keeping mass same, the surface
area of the block is doubled. The angle at which this
starts sliding down is :
53. A block of mass M rests on a rough horizontal
(A) 30° (B) 60° (C) 15° (D) none
surface as shown. Coefficient of friction between Sol.
the block and the surface is . A force F = Mg
acting at angle  with the vertical side of the block
pulls it in which of the following cases the block
can be pulled along the surface ?
(A) tan   
(B) tan (/2)  
(C) cot   
(D) cot (/2)  

Corporate Head Office : Motion Education Pvt. Ltd., 394 - Rajeev Gandhi Nagar, Kota-5 (Raj.)
N.L.M. , FRICTION Page # 71

Exercise - II
(A) NEWTON'S LAW OF MOTION 3. If block A has a velocity of 0.6 m/s to the right,
(SINGLE CORRECT) determine the velocity of block B.
1. The pulleys in the diagram are all smooth and light.
The acceleration of A is a upwards and the acceleration A
of C is f downwards. The acceleration of B is

B
(A) 1.8 m/s in downward direction
A C (B) 1.8 m/s in upward direction
(C) 0.6 m/s in downward direction
(D) 0.6 m/s in upward direction
B Sol.
(A) 1/2 (f – a) up (B) 1/2 (a + f) down
(C) 1/2 (a + f) up (D) 1/2 (a – f) up
Sol.

4. Block B moves to the right with a constant velocity


v0. The velocity of body A relative to B is :
v0
2. If acceleration of A is 2 m/s2 to left and acceleration
of B is 1 m/s2 to left, then acceleration of C is - A B
A B
v0 v0
(A) , towards left (B) , towards right
2 2
C 3v 0 3v 0
(C) , towards left (D) , towards right
(A) 1 m/s upwards
2
(B) 1 m/s downwards
2 2 2
(C) 2 m/s2 downwards (D) 2 m/s2 upwards Sol.
Sol.

: 0744-2209671, 08003899588 | url : www.motioniitjee.com, : info@motioniitjee.com


Page # 72 N.L.M. , FRICTION
5. Find the acceleration of C w.r.t. ground. 7. A sperical ball of mass m = 5 kg rests between two
planes which make angles of 30° and 45° respectively
with the horizontal. The system is in equilibrium. Find
A B
C the normal forces exerted on the ball by each of the
a b
planes. The planes are smooth.

(A) ai – (2a  2b)j (B) a î – ( 2 a  b ) ˆj

(C) a î – ( a  2b ) ˆj (D) b î – ( 2a  2b ) ĵ
Sol. 45°

°
30
(A) N45 = 96.59 N, N30 = 136.6 N
(B) N30 = 96.59 N, N45 = 136.6 N
(C) N45 = 136.6 N, N30 = 96.56 N
(D) none of these
Sol.

6. Find the acceleration of B.

Question No. 8 to 9 (2 questions)


In the figure the tension in the diagonal string is 60 N.
A B

F1
45°
a
F3
acos  1 a sin  1 a cos 2 cos  1
(A) cos  (B) cos  (C) cos  (D) cos  W
2 2 1 2

Sol. 8. Find the magnitude of the horizontal force F1 and

F2 that must be applied to hold the system in the
position shown.
60 20 40 60
(A) N (B) N (C) N (D) N
3 2 2 2
Sol.

Corporate Head Office : Motion Education Pvt. Ltd., 394 - Rajeev Gandhi Nagar, Kota-5 (Raj.)
N.L.M. , FRICTION Page # 73

9. In the above questions what is the weight of the 11. Two masses m and M are attached to the strings
suspended block ? as shown in the figure. If the system is in equilibrium,
then
60 40 60 50
(A) N (B) N (C) N (D) N
2 2 3 2
Sol.

45° M
45°

2M 2m
(A) tan  1  (B) tan  1 
m M
2M 2m
(C) cot   1  (D) cot   1 
m M
Sol.
10. Objects A and B each of mass m are connected
by light inextensible cord. They are constrained to
move on a frictionless ring in a vertical plane as shown
in figure. The objects are released from rest at the
positions shown. The tension in the cord just after
release will be
A

T mg
T
B C

mg 12. A flexible chain of weight W hangs between two


fixed points A & B which are at the same horizontal
mg mg mg level. The inclination of the chain with the horizontal
(A) mg 2 (B) (C) (D)
2 2 4 at both the points of support is . What is the tension
Sol. of the chain at the mid point ?

A B
 

W
W W
(A) .cos ec  (B) . tan
2 2
W
(C) .cot  (D) none
2
Sol.

: 0744-2209671, 08003899588 | url : www.motioniitjee.com, : info@motioniitjee.com


Page # 74 N.L.M. , FRICTION
Question No. 13 to 15 (3 questions) Sol.
A particle of mass m is constrained to move on x-axis.
A force F acts on the particle. F always points toward
the position labeled E. For example, when the particle
is to the left of E, F points to the right. The magnitude
of F is constant except at point E where it is zero.
A
+ve
m E x
The system is horizontal. F is the net force acting on
the particle. The particle is displaced a distance A
towards left from the equilibrium position E and released
from rest at t = 0

13. What is the period of the motion ?


 2Am   2Am 
(A) 4  
F  (B) 2  
F 
 
A
 2Am  15. Find minimum time it will take to reach from x  
2
(C)  
F  (D) None to 0.

Sol. 3
mA mA
(A) ( 2  1) (B) ( 2  1)
2F F
mA
(C) 2 ( 2  1) (D) none
F
Sol.

16. A constant force F is applied in horizontal direction


as shown. Contact force between M and m is N and
between m and M’ is N’ then
M'>M
F M m M'

smooth
14. Velocity-time graph of the particle is (A) N or N’ equal (B) N > N’
v v (C) N’ > N (D) cannot be determined
Sol.
t
(A) (B) t

v v

(C) t (D) t

Corporate Head Office : Motion Education Pvt. Ltd., 394 - Rajeev Gandhi Nagar, Kota-5 (Raj.)
N.L.M. , FRICTION Page # 75

Question No. 17 to 19 (3 questions) 20. Two blocks, each having mass M , rest on
A block of mass 1kg is suspended by a string of mass frictionless surfaces as shown in the figure. If the
1 kg, length 1m as shown in figure. (g = 10 m/s2) pulleys are light and frictionless, and M on the incline
Calculate : is allowed to move down, then the tension in the
string will be

1m
M
fixed M
1 kg
2 3
17. The tension in string at its lowest point. (A) Mgsin (B) Mgsin
3 2
(A) 10 N (B) 15 N (C) 20 N (D) 25 N
Sol. Mgsin 
(C) (D) 2 Mg sin 
2
Sol.

18. The tension in string at its mid-point


(A) 10 N (B) 15 N (C) 20 N (D) 25 N 21. Calculate the acceleration of the block B in the
Sol. above figure, assuming the surfaces and the pulleys
P1 and P2 are all smooth and pulleys and string and
light
2m 4m

P1
F A B
P2

3F 2F
(A) a = m/s2 (B) a = m/s2
17m 17m
3F 3F
19. Force exerted by support on string. (C) a = m/s2 (D) a = m/s2
15m 12m
(A) 10 N (B) 15 N (C) 20 N (D) 25 N
Sol. Sol.

: 0744-2209671, 08003899588 | url : www.motioniitjee.com, : info@motioniitjee.com


Page # 76 N.L.M. , FRICTION
22. In previous Question surface is replaced by block Sol.
C of mass m as shown in figure. Find the acceleration
of block B.
2m 4m

P1
F A B
P2
c

3F 3F
(A) a = 20m m/s2 (B) a = m/s2
21m

2F 3F
(C) a = 21m m/s2 (D) a = 18m m/s2

Sol.
24. Block of 3 kg is initially in equilibrium and is hanging
by two identical springs A and B as shown in figures.
If spring A is cut from lower point at t = 0 then, find
acceleration of block in ms–2 at t = 0.

A B

3 kg

(A) 5 (B) 10 (C) 15 (D) 0


Sol.

25. Two masses of 10 kg and 20 kg respectively are


connected by a massless spring as shown in figure. A
force of 200 N acts on the 20 kg mass at the instant
when the 10 kg mass has an acceleration of 12 ms–2
23. In the arrangement shown in the fig, the block of towards right, the aceleration of the 20 kg mass is :
mass m = 2 kg lies on the wedge on mass M = 8 kg.
10kg 20kg
Find the initial acceleration of the wedge if the surfaces
are smooth and pulley & strings are massless. 200 N
P1 (A) 2 ms –2
(B) 4 ms –2
(C) 10 ms –2
(D) 20 ms–2
60° Sol.
m

M
P2
60°

30 3 20 3
(A) a = m/s2 (B) a = m/s2
23 23

30 2
(C) a = m/s2 (D) none of these
23

Corporate Head Office : Motion Education Pvt. Ltd., 394 - Rajeev Gandhi Nagar, Kota-5 (Raj.)
N.L.M. , FRICTION Page # 77

26. Two blocks are connected by a spring. The Sol.


combination is suspended, at rest, from a string
attatched to the ceiling, as shown in the figure. The
string breaks suddenly. Immediately after the string
breaks, what is the initial downward acceleration of
the upper block of mass 2m ?

2m

m
(A) 0 (B) 3g/2 (C) g (D) 2g
Sol.

28. What will be the reading of spring balance in the


figure shown in following situations. (g = 10 m/s2)
(i) a = 0, v = 0
(A) 100 N (B) 80 N (C) 120 N (D) 150 N
(ii) a = 0, v = 2 m/s
(A) 100 N (B) 80 N (C) 120 N (D) 150 N
(iii) a = 0, v = – 2m/s
(A) 100 N (B) 80 N (C) 120 N (D) 150 N

a v

27. A man of mass 60 kg is standing on a weighing M = 10 kg


machine placed in a lift moving with velocity 'v' and
acceleration 'a' as shown in figure. Calculate the reading (iv) a = 2 m/s2, v =0
of weighing machine in following situation: (A) 100 N (B) 80 N (C) 120 N (D) 150 N
(g = 10 m/s2) (v) a = – 2m/s2, v = 0
(i) a = 0, v=0 (A) 100 N (B) 80 N (C) 120 N (D) 150 N
(A) 600 N (B) 500 N (C) 450 N (D) 700 N (vi) a = 2 m/s2, v = 2 m/s
(ii) a = 0, v = 2m/s (A) 100 N (B) 80 N (C) 120 N (D) 150 N
(A) 600 N (B) 500 N (C) 450 N (D) 700 N (vii) a = 2 m/s2, v = –2m/s
(iii) a = 0, v = –2m/s (A) 100 N (B) 80 N (C) 120 N (D) 150 N
(A) 450 N (B) 500 N (C) 600 N (D) 700 N (viii) a = – 2 m/s2, v = – 2 m/s
(A) 100 N (B) 80 N (C) 120 N (D) 150 N
a v Sol.

W.M.
(iv) a = 2m/s2, v=0
(A) 600 N (B) 500 N (C) 450 N (D) 720 N
(v) a = –2m/s2 v=0
(A) 600 N (B) 480 N (C) 450 N (D) 700 N
(vi) a = 2m/s2, v = 2m/s
(A) 600 N (B) 480 N (C) 450 N (D) 720 N
(vii) a = 2 m/s2, v = –2m/s
(A) 600 N (B) 720 N (C) 450 N (D) 700 N
(viii) a = –2m/s2 v = –2 m/s
(A) 600 N (B) 480 N (C) 450 N (D) 700 N

: 0744-2209671, 08003899588 | url : www.motioniitjee.com, : info@motioniitjee.com


Page # 78 N.L.M. , FRICTION
Question No. 29 to 31 (3 questions) Sol.
An object of mass 2 kg is placed at rest in a frame
(S ) moving with velocity 10 i  5 j m/s and having
1

acceleration 5 i  10 j m / s 2 . The object is also seen by


an observer standing in a frame (S2) moving with
velocity 5 i  10 j m / s
29. Calculate 'Pseudo force' acting on object. Which
frame is responsible for this force.
(A) F = – 10 i – 20 j due to acceleration of frame S 1

(B) F = – 20 i – 20 j due to acceleration of frame S1


32. In the figure the reading of the spring balanced
(C) F = – 10 i – 30 j due to acceleration of frame S1 will be : [g = 10 m/s2]
(D) none of these 2m/s2
Sol.

10kg
5kg

30°

(A) 50 N (B) 40 N (C) 60 N (D) 70 N


Sol.

30. Calculate net force acting on object with respect


to S2 frame.
(A) F = 20 î  20 ĵ (B) F = 10 i  20 j

(C) F = 5 î  20 ˆj (D) F = 10 î  5 ĵ
33. A pendulum of mass m hangs from a support fixed
Sol.
to a trolley. The direction of the string when the trolley
rolls up of plane of inclination  with acceleration a0 is
(String and bob remain fixed with respect to trolley)
a0

 a0 
(A)  = tan–1 (B)  = tan–1  g 
31. Calculate net force acting on object with respect
of S1 frame.
 g –1 a 0  g sin  
(A) 0 (B) 1 (C)  = tan–1  a  (D)  = tan  g cos  
(C) 2 (D) none of these  0

Corporate Head Office : Motion Education Pvt. Ltd., 394 - Rajeev Gandhi Nagar, Kota-5 (Raj.)
N.L.M. , FRICTION Page # 79

Sol. Sol.

(MULTIPLE CORRECT)

34. A student calculates the acceleration of m1 in


(m1 – m 2 )g
figure shown as a1 = m  m . Which assumption is
1 2
not required to do this calculation.

36. Two men of unequal masses hold on to the two


sections of a light rope passing over a smooth light
pulley. Which of the following are possible?
m1
m2

(A) pulley is frictionless (B) string is massless


(C) pulley is massless (D) string is inextensible
Sol.

(A) The lighter man is stationary while the heavier


man slides with some acceleration
(B) The heavier man is stationary while the lighter
man climbs with some acceleration
(C) The two men slide with the same acceleration in
the same direction
(D) The two men move with accelerations of the same
magnitude in opposite directions
Sol.

35. Which graph shows best the velocity-time graph


for an object launched vertically into the air when air
resistance is given by |D| = bv? The dashed line shows
the velocity graph if there were no air resistance.

v v

(A) t (B) t

v v

(C) t (D) t

: 0744-2209671, 08003899588 | url : www.motioniitjee.com, : info@motioniitjee.com


Page # 80 N.L.M. , FRICTION
37. Adjoining figure shows a force of 40 N acting at 39. In the system shown in the figure m1 > m2. System
30° to the horizontal on a body of mass 5 kg resting is held at rest by thread BC. Just after the thread BC
on a smooth horizontal surface. Assuming that the is burnt :
acceleration of free-fall is 10 ms –2, which of the
following statements A, B, C, D, E is (are) correct?
40 N
5 kg 30°
spring B m2
[1] The horizontal force acting on the body is 20 N k
[2] The weight of the 5 kg mass acts vertically
downwards m1 A
[3] The net vertical force acting on the body is 30 N C
(A) 1, 2, 3 (B) 1, 2 (C) 2 only (D) 1 only (A) acceleration of m2 will be upwards
Sol. (B) magnitude of acceleration of both blocks will be
 m1 – m2 
equal to  m  m  g
 1 2
(C) acceleration of m1 will be equal to zero
(D) magnitude of acceleration of two blocks will be
non-zero and unequal.
Sol.

38. For ordinary terrestrial experiments, which of the


following observers below are inertial.
(A) a child revolving in a “giant wheel”. 40. A particle is resting on a smooth horizontal floor.
(B) a driver in a sports car moving with a constant At t = 0, a horizontal force starts acting on it.
high speed of 200 km/h on a straight road. Magnitude of the force increases with time according
(C) the pilot of an aeroplane which is taking off. to law F = .t, where  is a constant. For the figure
(D) a cyclist negotiating a sharp turn. shown which of the following statements is/are correct
Sol. ?
2
y
1

O x
(A) Curve 1 shows acceleration against time
(B) Curve 2 shows velocity against time
(C) Curve 2 shows velocity against acceleration
(D) none of these
Sol.

Corporate Head Office : Motion Education Pvt. Ltd., 394 - Rajeev Gandhi Nagar, Kota-5 (Raj.)
N.L.M. , FRICTION Page # 81

41. Two blocks A and B of equal mass m are connected 43. A force of magnitude F1 acts on a particle so as
through a massless string and arranged as shown in to accelerate it from rest to a velocity v. The force F1
figure. Friction is absent everywhere. When the system is then replaced by another force of magnitude F2
is released from rest. which decelerates it to rest.
(A) F1 must be the equal to F2
A (B) F1 may be equal to F2
(C) F1 must be unequal to F2
(D) None of these
fixed Sol.
30° B

mg
(A) tension in string is
2
mg
(B) tension in string is
4
(C) acceleation of A is g/2
3
(D) acceleration of A is g
4
Sol.

44. In the figure, the blocks A, B and c of mass m


each have acceleration a1. a2 and a3 respectively. F1
and F2 are external forces of magnitudes 2 mg and mg
respectively.

42. Figure shows the displacement of a particle going


along the x-axis as a funtion of time :
x m m m
E F1=2mg
A B 2m C F2=mg
D
C (A) a1 = a2 = a3 (B) a1 > a2 > a3
B
A (C) a1 = a2, a2 > a3 (D) a1 > a2 , a2 = a3
t Sol.
(A) the force acting on the particle is zero in the
region AB
(B) the force acting on the particle is zero in the
region BC
(C) the force acting o the particle is zero in the region
CD
(D) the force is zero no where.
Sol.

: 0744-2209671, 08003899588 | url : www.motioniitjee.com, : info@motioniitjee.com


Page # 82 N.L.M. , FRICTION
45. A rope is stretched between two boats at rest. A 47. Five persons A, B, C, D & E are pulling a cart of
sailor in the first boat pulls the rope with a constant mass 100 kg on a smooth surface and cart is moving
force of 100 N. First boat with the sailor has a mas of with acceleration 3 m/s2 in east direction. When person
250 kg whereas mass of second boat is double of that ‘A’ stops pulling, it moves with acceleration 1 m/s2 in
mass. If the initial distance between the boats was the west direction. When person ‘B’ stops pulling, it
100 m, the time taken for two boats to meet each moves with acceleration 24 m/s2 in the north direction.
other is - The magnitude of acceleration of the cart when only
A & B pull the cart keeping their directions same as
the old directions, is :
(A) 26 m/s2 (B) 3 71m / s 2
(C) 25 m/s 2
(D) 30 m/s2
Sol.
(A) 13.8 s (B) 18.3 s (C) 3.18 s (D) 31.8 s
Sol.

(B) FRICTION
(SINGLE CORRECT)
48. Find the direction of friction forces on each block
46. A chain of length l is placed on a smooth spherical and the ground (Assume all surfaces are rough and all
surface of radius r with one of its ends fixed at the velocities are with respect to ground).
top of the surface. Length of chain is assumed to be
r E 2 m/s
l . Acceleration of each element of chain when
2
upper end is released is - D 1 m/s

C 3 m/s

B 5 m/s

5 m/s A
g  r rg  
(A) r 1 – cos   (B)   1– cos r  Sol.

g   rg  r
(C) r 1 – sin r  (D)   1– sin  

Sol.

Corporate Head Office : Motion Education Pvt. Ltd., 394 - Rajeev Gandhi Nagar, Kota-5 (Raj.)
N.L.M. , FRICTION Page # 83
49. In the following figure, find the direction of friction
Sol.
on the blocks and ground

VA=3m/s A F = 5N
7N
VB=6m/s B

Sol.

52. In the arrangement shown in the figure, mass of


the block B and A is 2m and m respectively. Surface
between B and floor is smooth. The block B is
connected to the block C by means of a string pulley
system. If the whole system is released, then find the
minimum value of mass of block C so that block A
remains stationary w.r.t. B. Coefficient of friction
 between A and B is .
50. A force F  i  4 j acts on block shown. The force
of friction acting on the block is A
F
y
1 Kg
B
x C
(A)  i . i
(B) 18 (C)  2.4 i (D)  3 i
m 2m  1 3m 6m
Sol. (A) (B) (C) (D)
  1  1  1
Sol.

53. Two masses A and B of 10 kg and 5 kg respectively


are connected with a string passing over a fricionless
pulley fixed at the corner of a table as shown in figure.
The coefficient of friction of A with the table is 0.2.
51. If force F is increasing with time and at t=0, F=0
The minimum mass of C that may be placed on A to
where will slipping first start ?
prevent if from moving is equal to :
F 3 C
2 10kg
1 A

(A) between 3 kg and 2 kg 5 kg


B
(B) between 2 kg and 1 kg
(C) between 1 kg and ground
(A) 15 kg (B) 10 kg (C) 5 kg (D) zero
(D) both (A) and (B)

: 0744-2209671, 08003899588 | url : www.motioniitjee.com, : info@motioniitjee.com


Page # 84 N.L.M. , FRICTION
Sol. 56. A block of mass m lying on a rough horizontal
plane is acted upon by a horizontal force P and another
force Q inclined an at an angle  to the vertical. The
minimum value of coefficient of friction between the
block and the surface for which the block will remain
in equilibrium is :

Q

P

///////////////////////////////////////

P  Q sin  P cos   Q
54. If the coefficient of friction between an insect (A) (B)
and bowl is  and the radius of the bowl, is r, the mg  Q cos  mg – Q sin 
maximum height to which the insect can crawl in the P  Q cos  P sin  – Q
bowl is : (C) (D)
mg  Q sin  mg – Q cos 
r  1 
  Sol.
(A) 2 (B) 1 –
r

1   1 2 
(C) r 1   2 (D) r 1   2 – 1
Sol.

57. A small mass slides down an inclined plane of


55. With what minimum velocity should block be
inclination  with the horizontal. The co-efficient of
projected from left end A towards end B such that it
friction is  = 0x where x is the distance through
reaches the other end B of conveyer belt moving with which the mass slides down and 0, a constant. Then
constant velocity v. Friction coefficient between block the distance covered by the mass before it stops is :
and belt is .
2 4 1 1
m v0 B (A)  tan  (B)  tan  (C) 2 tan  (D)  tan 
A
v 0 0 0 0
Sol.

L
(A) gL (B) 2gL (C) 3gL (D) 2 gL
Sol.

Corporate Head Office : Motion Education Pvt. Ltd., 394 - Rajeev Gandhi Nagar, Kota-5 (Raj.)
N.L.M. , FRICTION Page # 85
58. In the above question the speed of the mass Sol.
when travelled half the maximum distance is
gtan  sin  gtan  sin 
(A) 0 (B) 2 0

gtan  sin 
(C) 8 0 (D) none of these

Sol.

61. Two blocks of masses m1 and m2 are connected


with a massless unstretched spring and placed over a
plank moving with an acceleration 'a' as shown in
figure. the coefficient of friction between the blocks
and platform is .
m1 m2
a

(A) spring will be stretched if a > g


(B) spring will be compressed if a  g
59. A body is moving down a long inclined plane of (C) spring will neither be compressed nor be stretched
slope 37º. The coefficient of friction between the body for a  g
and plane varies as  = 0.3 x, where x is distance (D) spring will be in its natural length under all
travelled down the plane. The body will have maximum conditions.
3 Sol.
speed. (sin 37º = and g = 10 m/s2)
5
(A) at x = 1.16 m (B) at x = 2m
(C) at bottom of plane (D) at x = 2.5 m
Sol.

62. A block placed on a rough inclined plane of


inclination ( = 30º) can just be pushed upwards by
applying a force “F” as shown. If the angle of inclination
of the inclined plane is increased to ( = 60º), the
same block can just be prevented from sliding down
60. Block B of mass 100 kg rests on a rough surface by application of a force of same magnitude. The
of friction coefficient  = 1/3. A rope is tied to block B coefficient of friction between the block and the
as shown in figure. The maximum acceleration with inclined plane is
which boy A of 25 kg can climbs on rope without
making block move is

3 1 2 3 1
(A) (B)
3 1 3 1
4g g g 3g 3 1
(A) (B) (C) (D)
3 3 2 4 (C) (D) none of these
3 1

: 0744-2209671, 08003899588 | url : www.motioniitjee.com, : info@motioniitjee.com


Page # 86 N.L.M. , FRICTION
Sol. (b) In the above question the coefficient of friction
between Q and the rough plane is :
4 1 3 2
(A) (B) (C) (D)
5 5 5 5
Sol.

63. A fixed wedge with both surface inclined at 45° to


the horizontal as shown in the figure. A particle P of
mass m is held on the smooth plane by a light string
which passes over a smooth pulley A and attached to
a particle Q of mass 3m which rests on the rough
plane. The system is released from rest. Given that
g (c) In the above question the magnitude and direction
the acceleration of each particle is of magnitude of the force exerted by the string on the pulley is :
5 2
then A 6mg 6mg
(A) downward (B) upward
5 5
mg mg
(C) downward (D) downward
5 4
th

Ro

P Q
oo

Sol.
ug

m 3m
Sm

45° fixed 45°

(a) the tension in the string is :


6mg mg mg
(A) mg (B) (C) (D)
5 2 2 4
Sol.

Corporate Head Office : Motion Education Pvt. Ltd., 394 - Rajeev Gandhi Nagar, Kota-5 (Raj.)
N.L.M. , FRICTION Page # 87
64. A force F = t is applied to block A as shown in Sol.
figure. The force is applied at t = 0 seconds when the
system was at rest and string is just straight without
tension. Which of the following graphs gives the friction
force between B and horizontal surface as a function
a time 't'.
B A
m m F

 s  k s  k

f f
(A) (B)
t t
66. When F = 2N, the frictional force between 10 kg
block and 5 kg block is
(A) 2N (B) 15N (C) 10N (D) None
f f
Sol.

(C) (D)
t t

Sol.

67. The maximum “F” which will not cause motion of


any of the blocks
(A) 10N (B) 15N
(C) data insufficient (D) None
Sol.

68. The maximum acceleration of 5 kg block


(A) 1 m/s2 (B) 3 m/s2 (C) 0 (D) None
Sol.
For 65. to 69 refer given figure (5 questions)
65. When F = 2N, the frictional force between 5 kg
block and ground is

10kg F

5kg

(A) 2N (B) 0 (C) 8 N (D) 10 N

: 0744-2209671, 08003899588 | url : www.motioniitjee.com, : info@motioniitjee.com


Page # 88 N.L.M. , FRICTION
69. The acceleration of 10 kg block when F = 30 N 71. A board is balanced on a rough horizontal
(A) 2 m/s2 (B) 3 m/s2 (C) 1 m/s2 (D) None semicircular log. Equilibrium is obtained with the help
Sol. of addition of a weight to one of the ends of the
board when the board makes an angle  with the
horizontal. Coefficient of friction between the log
and the board is

(A) tan  (B) cos  (C) cot  (D) sin 


Sol.

70. A truck starting from rest moves with an


acceleration of 5 m/s2 for 1 sec and then moves with
constant velocity. The velocity w.r.t. ground v/s time
graph for block in truck is (Assume that block does
not fall off the truck)

72. A stationary body of mass m is slowly lowered


  0.2 onto a massive plateform of mass M (M >> m) moving
at a speed V0 = 4 m/s as shown in fig. How far will the
body slide along the platform ( = 0.2 and g = 10 m/
s2 ) ?

5 m/s 3 m/s (A) 4 m (B) 6 m (C) 12 m (D) 8 m


(A) (B) Sol.
1 sec 1 sec

5 m/s
(C) (D) None of these

2.5 sec
Sol.

(MULTIPLE CORRECT)
73. A block of mass 2.5 kg is kept on a rough horizontal
surface. It is found that the block does not slide if a
horizontal force less than 15 N is applied to it. Also it
is if found that it takes 5 second to slide throughout
the first 10 m if a horizontal force of 15 N is applied
and the block is gently pushed to start the motion.
Taking g = 10 m/s2, then
(A) s = 0.60 (B) k = 0.52
(C) k = 0.60 (D) s = 0.52

Corporate Head Office : Motion Education Pvt. Ltd., 394 - Rajeev Gandhi Nagar, Kota-5 (Raj.)
N.L.M. , FRICTION Page # 89
Sol. Sol.

76. A man pulls a block heavier than himself with a


light horizontal rope. The coefficient of friction is the
same between the man and the ground, and between
the block and the ground.
74. The coefficient of friction between 4 kg and 5 kg
blocks is 0.2 and between 5 kg block and ground is
0.1 respectively. Choose the correct statements

P 4 kg (A) The block will not move unless the man also moves
Q 5 kg (B) The man can move even when the block is
F
stationary
(A) Minimum force needed to cause system to move (C) If both move, the acceleration of the man is
is 17N greater than the acceleration of the block
(B) When force is 4N static friction at all surfaces is 4 (D) None of the above assertions is correct
N to keep system at rest. Sol.
(C) Maximum acceleration of 4 kg block is 2 m/s2
(D) Slipping between 4 kg and 5 kg blocks start when
F is 17 N
Sol.

Question No. 77 to 78 (2 questions)


In figure, two blocks M and m are tied together with
an inextensible and light string. The mass M is placed
on a rough horizontal surface with coefficient of friction
 and the mass m is hanging vertically against a smooth
vertical wall. The pulley is frictionless.

M
Rough
75. In a tug-of-war contest, two men pull on a m
horizontal rope from opposite sides. The winner will be Smooth
the man who
(A) exerts greater force on the rope 77. Choose the correct statement(s)
(B) exerts greater force on the ground (A) The system will accelerate for any value of m
(C) exerts a force on the rope which is greater than (B) The system will accelerate only when m > M
the tension in the rope (C) The system will accelerate only when m > M
(D) makes a smaller angle with the vertical (D) Nothing can be said

: 0744-2209671, 08003899588 | url : www.motioniitjee.com, : info@motioniitjee.com


Page # 90 N.L.M. , FRICTION
Sol. Sol.

78. Choose the correct statement(s) related to the


tension T in the string
(A) When m < M, T = mg
(B) When m < M, T = Mg 80. In above problem, choose the correct value(s) of
(C) When m > M, Mg < T < mg F which the blocks M and m remain stationary with
(D) When m > M, mg < T < Mg respect to M0
Sol.
g m(M0  M  m)g
(A) (M0  M  m) (B)
 M – m
mg
(C) (M0  M  m) (D) none of these
M
Sol.

Question No. 79 to 81 (3 questions)


Imagine a situation in which the horizontal surface of
block M0 is smooth and its vertical surface is rough
with a coefficient of friction  81. Consider a special situation in which both the faces
of the block M0 are smooth, as shown in adjoining
Smooth M figure. Mark out the correct statement(s)

Smooth M
Mo m
F
Rough M0 m
F
Smooth
79. Identify the correct statement(s)
(A) If F=0, the blocks cannot remain stationary
(B) For one unique value of F, the blocks M and m (A) If F = 0, the blocks cannot remain stationary
remain stationary with respect to M0 (B) For one unique value of F, the blocks M and m
(C) The limiting friction between m and M 0 is remain stationary with respect to block M0
independent of F (C) There exists a range of F for which blocks M and
(D) There exist a value of F at which friction force is m remain stationary with respect to block M0
equal to zero. (D) Since there is no friction, therefore, blocks M and
m cannot be in equilibrium with respect to M0

Corporate Head Office : Motion Education Pvt. Ltd., 394 - Rajeev Gandhi Nagar, Kota-5 (Raj.)
N.L.M. , FRICTION Page # 91

Sol. 84. Car is accelerating with acceleration = 20 m/s2. A


box that is placed inside the car, of mass m = 10 kg is
put in contact with the vertical wall as shown. The
friction coefficient between the box and the wall is 
= 0.6.
  0.6

10kg
2
20m/s

(A) The acceleration of the box will be 20 m/sec2


(B) The friction force acting on the box will be 100 N
(C) The contact force between the vertical wall and

82. The value(s) of mass m for which the 100 kg the box will be 100 5 N
block remains is static equilibrium is (D) The net contact force between the vertical wall
and the box is only of electromagnetic in nature.
Sol.

100 m
  0.3
37°

(A) 35 kg (B) 37 kg (C) 83 kg (D) 85 kg


Sol.

83. The contact force exerted by one body on another


body is equal to the normal force between the bodies.
It can be said that :
(A) the surface must be frictionless
(B) the force of friction between the bodies is zero
(C) the magnitude of normal force equals that of
friction
(D) It is possible that the bodies are rough and they
do not slip on each other.
Sol.

: 0744-2209671, 08003899588 | url : www.motioniitjee.com, : info@motioniitjee.com


Page # 92 N.L.M. , FRICTION

Exercise - III (JEE ADVANCED)

1. Two masses A and B, lie on a frictionless table. 3. To Paint the side of a building, painter normally
They are attached to either end of a light rope which hoists himself up by pulling on the rope A as in figure.
passes around a horizontal movable pulley of negligible The painter and platform together weigh 200 N . The
mass. Find the acceleration of each mass MA = 1 kg, rope B can withstand 300 N. Find
MB = 2 kg, MC = 4 kg. The pulley P2 is vertical.

P1
B
P2
A
C

Sol.
(a) the maximum acceleration of the painter.
(b) tension in rope A
(i) when painter is at rest
(ii) when painter moves up with an acceleration
2 m/s2.
Sol.

2. Block A of mass m/2 is connected to one end of


light rope which passes over a pulley as shown in the
fig. Man of mass m climbs the other end of rope with a
relative acceleration of g/6 with respect to rope find
acceleration of block A and tension in the rope. 4. An inclined plane makes an angle 30º with the
horizontal. A groove OA = 5 m cut in the plane makes
an angle 30º with OX. A short smooth cylinder is free
to slide down the influence of gravity. Find the time
taken by the cylinder to due to reach from A to O.
g/6 (g = 10 m/s2)
m
m/2 er
ind A
cyl 30°
A
Sol. 30°
O x
Sol.

Corporate Head Office : Motion Education Pvt. Ltd., 394 - Rajeev Gandhi Nagar, Kota-5 (Raj.)
N.L.M. , FRICTION Page # 93

5. Same spring is attached with 2 kg, 3 kg and 1 kg Sol.


blocks in three different cases as shown in figure. If
x1, x2 and x3 be the extensions in the spring in these
three cases then find the ratio of their extensions.

2 kg 3 kg 1 kg
2 kg 2 kg 2 kg
(a) (b) (c) 8. What force must man exert on rope to keep platform
in equilibrium ?
Sol.

Sol.

6. Find the reading of spring balance as shown in


figure. Assume that mass M is in equilibrium. (All
surfaces are smooth)
2.5
Spring kg
balance
M

37º 9. Inclined plane is moved towards right with an


acceleration of 5ms–2 as shown in figure. Find force in
Sol. newton which block of mass 5 kg exerts on the incline
plane. (All surfaces are smooth)

5kg

37º
5 m/s2
Sol.
7. At what value of m1 will 8 kg mass be at rest.

8kg

5kg
m1

: 0744-2209671, 08003899588 | url : www.motioniitjee.com, : info@motioniitjee.com


Page # 94 N.L.M. , FRICTION
10. Find force in newton which mass A exerts on Sol.
mass B if B is moving towards right with 3 ms–2. Also
find mass of A. (All surfaces are smooth)

A
3m/s2
1kg
B
37º

Sol.

13. The vertical displacement of block A in meter is


given by y = t2/4 where t is in second. Calculate the
downward acceleration aB of block B.

A
11. Force F is applied on upper pulley. If F = 30t y
where t is time in seconds. Find the time when m1 B
loses contact with floor.
F 30t N Sol.

m1 m2
m1 = 4kg m2 = 1kg
14. An object of mass m is suspended in equilibrium
Sol. using a string of length l and a spring having spring
constant K (< 2 mg/l) and unstreched length l/2.

12. The 40 kg block is moving to the right with a (a) Find the tension in the string
speed of 1.5 m/s when it is acted upon by forces F1 & (b) What happens if K > 2 mg / l ?
F2. These forces vary in the manner shown in the Sol.
graph. Find the velocity of the block after t = 12 s
Neglect friction and masses of the pulleys and cords.

F(N)
F2
40
F2 F1
30

F1 20
10
t(s)
0 2 4 6 12

Corporate Head Office : Motion Education Pvt. Ltd., 394 - Rajeev Gandhi Nagar, Kota-5 (Raj.)
N.L.M. , FRICTION Page # 95

15. Three monkeys A, B, and C (d) Now repeat the above for configuration (2). First,
with masses of 10, 15 & 8 kg find the force, F, the person must exert straight down
respectively are climbing up & in order to accelerate the platform+ person system
down the rope suspended from with an upward acceleration a. Include a freebody
D. at the instant represented, diagram in your solution.
A is descending the rope with (e) Now, what force does the platform exert on the
an acceleration of 2 m/s2 & C is person when the acceleration of the system is a?
pull i ng hi msel f up wi th an Include a freebody diagram in your solution.
acceleration (f) Again, if the platform is massless, M = 0, and he
of 1.5 m/s 2 . Monkeys B i s wants to raise it with a constant velocity find F. Does
climbing up this configuration offer a mechanical advantage ? (That
with a constant speed of 0.8 m/ is, is F < mg?)
s. Treat the rope and monkeys Sol.
as a comp l e te s ys te m &
calculate the tension T in the
rope at D. (g = 10 m/s–2)

Sol.

16. A person of mass m is standing on a platform of


mass M and wants to raise this platform. Massless (B) FRICTION
pulleys are configured in two different ways as shown. 17. Give the acceleration of blocks :
We would like to know which configuration makes it 50N
easier to raise the platform. Answer the following 37°
 s  0.5  s  0.5
questions in terms of m, M, a and constant as (A)  k  0.4 5kg 40N (B)  k  0.4 10kg
appropriate. [Note : Assume the rope is also massless
and does not stretch.]
 s   k  0.6 10kg

(C)

m m 5kg

M M
Sol.
Fig(1) Fig(2)
(a) For configuration (1) find the force, F, the person
must exert straight up in order to accelerate the
platform + person system with an acceleration a.
Include a freebody diagram in your solution.
(b) What force does the platform exert on the person
when the acceleration of the system is a? Include a
freebody diagram in your solution.
(c) If platform is massless, M = 0, and he wants to
raise it with a constant velocity find F. Does this
configuration offer a mechanical advantage ? (That
is, is F < mg ?)

: 0744-2209671, 08003899588 | url : www.motioniitjee.com, : info@motioniitjee.com


Page # 96 N.L.M. , FRICTION
18. Determine the coefficient of friction (), so that 21. A worker wishes to pile a cone of sand into a
rope of mass m and length l does not slide down. circular area in his yard. The radius of the circle is r,
and no sand is to spill onto the surrounding area. If 
is the static coefficient of friction between each layer
l/3 of sand along the slope and the sand, the greatest
volume of sand that can be stored in this manner is :
Sol.

Sol.

22. A block of mass 15 kg is resting on a rough inclined


plane as shown in figure. The block is tied up by a
horizontal string which has a tension of 50 N. The
coefficient of friction between the surfaces of contact
19. A wooden block A of mass M is placed on a is (g = 10 m/s2)
frictionless horizontal surface. On top of A, another T
lead block B also of mass M is placed. A horizontal m
force of magnitude F is applied to B. Force F is horizontal
increased continuously from zero. Then draw the graph 45°
between A and F. [k < s]
Sol.
B F
A

Sol.

23. In the figure, what should be mass m so that


block A slide up with a constant velocity.

20. A rope so lies on a table that part of it lays over. A


The rope begins to slide when the length of hanging g
part is 25 % of entire length. The co-efficient of friction 1k m
37º
between rope and table is :
Sol. =0.5
Sol.

Corporate Head Office : Motion Education Pvt. Ltd., 394 - Rajeev Gandhi Nagar, Kota-5 (Raj.)
N.L.M. , FRICTION Page # 97

24. A block of mass 1 kg is horizontally thrown with a 27. A thin rod of length 1 m is fixed in a vertical
velocity of 10 m/s on a stationary long plank of mass position inside a train, which is moving horizontally
2 kg whose surface has  = 0.5. Plank rests on with constant acceleration 4 m/s2. A bead can slide
frictionless surface. Find the time when m1 comes to on the rod, and friction coefficient between them is
rest w.r.t. plank. 1/2. If the bead is released from rest at the top of the
Sol. rod, find the time when it will reach at the bottom.
Sol.

25. Block M slides down on frictionless incline as shown.


Find the minimum friction coefficient so that m does
not slide with respect to M.
28. A body of mass 2kg rests on a horizontal plane
m having coefficient of friction  = 0.5. At t = 0 a horizontal

M force F is applied that varies with time F = 2t. The
time constant t0 at which motion starts and distance
moved in t = 2t0 second will be _______ and ________
37º respectively.
Sol. Sol.

26. The coefficient of static and kinetic friction between


the two blocks and also between the lower block and
the ground are s = 0.6 and k = 0.4. Find the value of
tension T applied on the lower block at which the
upper block begins to slip relative to lower block.

M = 2kg

T 29. Find the acceleration of the blocks and magnitude


M = 2kg & direction of frictional force between block A and
table, if block A is pulled towards left with a force of
Sol. 50N.

A x
F = 50N
5Kg

2
g = 10m/s
B
4Kg

: 0744-2209671, 08003899588 | url : www.motioniitjee.com, : info@motioniitjee.com


Page # 98 N.L.M. , FRICTION
Sol. 31. Coefficient of friction between 5 kg and 10 kg
block is 0.5. If friction between them is 20N. What is
the value of force being applied on 5 kg. The floor is
frictionless.
F
5kg
10kg

Sol.

30. A block A of mass 2kg rests on another block B of


mass 8kg which rests on a horizontal floor. The
coefficient of friction between A and B is 0.2 while
that between B and floor is 0.5. When a horizontal
force F of 25N is applied on the block B, the force of
friction between A and B is _________.
Sol.

Corporate Head Office : Motion Education Pvt. Ltd., 394 - Rajeev Gandhi Nagar, Kota-5 (Raj.)
N.L.M. , FRICTION Page # 99

1. The diagram shows particles A and B, of masses


0.2 kg and m kg respectively, connected by a light 3. A 1kg block ‘B’ rests as shown on a bracket ‘A’ of
inextensible string which passes over a fixed smooth same mass. Constant foroes F1 = 20 N and F2 = 8N
peg. The system is released from rest, with B at a start to act at time t = 0 when the distance of block
height of 0.25m above the floor. B descends, hitting B from pulley is 50 cm. Time when block B reaches the
the floor 0.5s later. All resistances to motion may be pulley is _______.
ignored.

50cm
F2
F1
B
A
B A
0.2 kg
m kg 0.25m

4. Two men of masses m1 and m2 hold on the opposite


(a) Find the acceleration of B as it descends.
ends of a rope passing over a frictionless pulley. The
(b) Find the tension in the string while B is descending
mass m1 climbs up the rope with an acceleration of
and find also the value of m.
1.2 m/s2 relative to the rope. The man m2 climbs up
(c) When B hits the floor it comes to rest immediately,
the rope with an acceleration of 2.0 m/s2 relative to
and the string becomes slack. Find the length of time
the rope. Find the tension in the rope if m1 = 40 kg
for which B remains at rest on the ground before being
and m2 = 60 kg. Also find the time after which they
jerked into motion again.
will be at same horizontal level if they start from rest
and are initially separated by 5m.
2. An ornament for a courtyard at a word’s fair is to
be made up of four identical, frictionless metal sphere,
each weighing 2 6 Newton. The spheres are to be
arranged as shown, with three resting on a horizontal
surface and touching each other; the fourth is to rest
freely on the other three. The bottom three are kept
from separating by spot welds at the points of contact
with each other. Allowing for a factor of satety of 3N,
how much tension must the spot welds with stand.
5. In figure shown, pulleys are ideal m1 > 2 m2. Initially
the system is in equilibrium and string connecting m2
to rigid support below is cut. Find the initial acceleration
of m2?

m2
Top View Horizontal View
m1

: 0744-2209671, 08003899588 | url : www.motioniitjee.com, : info@motioniitjee.com


Page # 100 N.L.M. , FRICTION

6. The system shown adjacent is in equilibrium. Find 9. A block of mass m lies on wedge of mass M as
the acceleration of the blocks A, B & C all of equal shown in figure. Answer following parts separately.
masses m at the instant when (a) With what minimum acceleration must the wedge
(Assume springs to be ideal) be moved towards right horizontally so that block m
(a) The spring between ceiling & A is cut. falls freely.

m
M
K

A
(b) Find the minimum friction coefficient required
B
between wedge M and ground so that it does not
K move while block m slips down on it.

C
10. A car begins to move at time t = 0 and then
(b) The string (inextensible) between A & B is cut.
accelerates along a straight track with a speed given
(c) The spring between B & C is cut.
by V(t) = 2t2 ms–1 for 0  t  2 After the end of
Also find the tension in the string when the system is
acceleration, the car continues to move at a constant
at rest and in the above 3 cases.
speed. A small block initially at rest on the floor of the
7. In the system shown. Find the initial acceleration car begins to slip at t = 1sec. and stops slipping at t =
of the wedge of mass 5M. The pulleys are ideal and 3sec. Find the coefficient of static and kinetic friction
the cords are inextensible. (there is no friction
anywhere). between the block and the floor.

M
11. In the figure shown,
5M (i) For what maximum value of force F can all these
2M
blocks move together.
(ii) Find the value of force F at which sliding starts at
8. A smooth right circular cone of semi vertical angle other rough surfaces
 = tan–1(5/12) is at rest on a horizontal plane. A
rubber ring of mass 2.5 kg which requires a force of 1kg
F
15N for an extension of 10cm is placed on the cone. 2kg
3kg
Find the increase in the radius of the ring in equilibrium.

(iii) Find acceleration of all blocks, nature and value


of friction force of force F = 18N.

Corporate Head Office : Motion Education Pvt. Ltd., 394 - Rajeev Gandhi Nagar, Kota-5 (Raj.)
N.L.M. , FRICTION Page # 101

Exercise - IV PREVIOUS YEAR QUESTIONS

LEVEL - I JEE MAIN


1. One end of massless rope, which passes over a 3. A light string passing over a smooth light pulley
massless and frictionless pulley P is tied to a hook C connects two blocks of masses m1 and m2 (vertically).
while the other end is free. maximum tension that the If the acceleration of the system is g/8,then ratio of
rope can bear is 360 N. Which what value of maximum the masses is [AIEEE 2002]
safe acceleration (in ms–2) can a man of 60 kg climb (A) 8 : 1 (B) 9 : 7 (C) 4 : 3 (D) 5 : 3
down the rope ? [AIEEE 2002] Sol.

4. When forces F1, F2, F3 are acting on a particle of


(A) 16 (B) 6 (C) 4 (D) 80
mass m such that F2 and F3 are mutually perpendiclar,
Sol.
then the particle remains stationary. If the force F1 is
now removed, then the acceleration of the particle is
[AIEEE 2002]

F1 F1 F3 ( F2  F3 ) F2
(A) (B) (C) (D)
m mF1 m m
Sol.

2. Three identical blocks of masses m = 2 kg are


drawn by a force F = 10.2N with an acceleration of
0.6 ms–2 on a frictionless surface, then what is the
tension (in N) in the string between that blocks B and
C? [AIEEE 2002]
5. A rocket witha lift-off mass 3.5 × 104 kg is blasted
C B A F upwards with an initial acceleration of 10 ms–2. Then
the initial thrust of the blast is [AIEEE 2003]
(A) 3.5 × 105 N (B) 7.0 × 105 N
(A) 9.2 (B) 7.8 (C) 4 (D) 9.8 (C) 14.0 × 105 N (D) 1.75 × 105 N
Sol. Sol.

: 0744-2209671, 08003899588 | url : www.motioniitjee.com, : info@motioniitjee.com


Page # 102 N.L.M. , FRICTION
6. A light spring balance hangs from the hook of the 9. A horizontal force of 10 N is necessary to just hold
other light spring balance and a block of mass M kg a block stationary against a wall. The coefficient of
hangs from the former one. Then the true statement friction between the block and the wall is 0.2. The
about the scale reading is [AIEEE 2003] weight of the block is
(A) Both the scales read M kg each [AIEEE 2003]
(B) The scale of the lower one reads M kg and of the F
upper one zero
(C) The reading of the two scales can be any thing 10 N
but the sum of the readings will be M kg
(D) Both the scales read M/2 kg
Sol.

(A) 20 N (B) 50 N (C) 100 N (D) 2 N


Sol.

7. A block of mass M is pulled along a horizontal


frictionless surface by a rope of mass m. If a force P
is applied at the free end of the rope, the force exerted
by the rope on the block is [AIEEE 2003]
Pm Pm PM
(A) (B) (C) P (D)
M m M m M m 10.Three forces start acting simultaneously on a
Sol.

particle moving with velocity v . These forces are
represented in magnitude and direction by the three
sides of a triangle ABC (as shown). The particle will
now move with velocity [AIEEE 2003]

8. A marble block of mass 2 kg lying on ice when


given a velocity of 6 ms–1 is stopped by friction in 10
s. Then the coefficient of friction is B
 A
[AIEEE 2003] (A) less than v
(A) 0.02 (B) 0.03 (C) 0.06 (D) 0.01
Sol. (B) greater than v

(C) | v | in the direction of largest force BC



(D) v, remaining unchanged
Sol.

Corporate Head Office : Motion Education Pvt. Ltd., 394 - Rajeev Gandhi Nagar, Kota-5 (Raj.)
N.L.M. , FRICTION Page # 103
11.A spring balance is attached to the ceiling of a
14.A machine gun fires a bullet of mass 40 g with a
lift. A man hangs his bag on the spring and the spring
velocity 1200 ms–1. The man holding it, can exert a
reads 49 N, when the lift is stationary. If the lift moves
maximum force of 144 N on the gun. How many bullets
downward with an acceleration of 5 ms–2, the reading
can he fire per second at the most ? [AIEEE 2004]
of the spring balance will be [AIEEE 2003]
(A) 1 (B) 4 (C) 2 (D) 3
(A) 24 N (B) 74 N (C) 15 N (D) 49 N
Sol.
Sol.

15.The upper half of an inclined plane with inclination


 is perfectly smooth, while the lower half is rough. A
12.A block rests on a rough inclined plane making an body starting from rest at the top will again come to
angle of 30° with the horizontal. The coefficient of rest at the bottom, if the coefficient of friction for
static friction between the block and the plane is 0.8. the lower half is given by [AIEEE 2005]
If the frictional force on the block is 10 N, the mass of (A) 2 sin  (B) 2 cos 
the block (in kg) is (g = 10 m/s2) [AIEEE 2004] (C) 2 tan  (D) tan 
(A) 2.0 (B) 4.0 (C) 1.6 (D) 2.5 Sol.
Sol.

16.A smooth block is released at rest on a 45° incline


and then slides a distances d. The time taken to slide
13.Two masses m1 = 5 kg and m2 = 4.8 is n time as much to slide on rough incline than on a
kg tied to a string are hanging over a smooth incline. The coefficient of friction is
light frictionless pulley. What is the [AIEEE 2005]
acceleration of the masses when lift is
free to move ? (g = 9.8 ms–2) 1 1
[AIEEE 2004] (A)  k  1  (B) k  1
m1 n2 n2
(A) 0.2 ms–2
(B) 9.8 ms–2
(C) 5 ms–2
1 1
m2 (C)  s  1  (D) s  1
(D) 4.8 ms–2 n2 n2
Sol. Sol.

: 0744-2209671, 08003899588 | url : www.motioniitjee.com, : info@motioniitjee.com


Page # 104 N.L.M. , FRICTION
17.A particle of mass 0.3 kg is subjected to a force 20.A ball of mass 0.2 kg is thrown vertically upwards
F = – kx with k = 15 Nm–1. What will be its initial by applying a force by hand. If the hand moves 0.2 m
acceleration, if it is released from a point 20 cm away while applying the force and the ball goes upto 2 m
from the origin ? [AIEEE 2005] height further, find the magnitude of the force. Consider
(A) 3 ms–2 (B) 15 ms–2 g = 10 m/s2. [AIEEE 2006]
(C) 5 ms–2 (D) 10 ms–2 (A) 4 N (B) 16 N (C) 20 N (D) 22 N
Sol. Sol.

21.A mass of M kg is suspended by a weightless string.


The horizontal force that is required to displace it
18.Consider a car moving on a straight road with a until the string makes an angle of 45° with the initial
speed of 100 ms–1. The distance at which car can be vertical direction is [AIEEE 2006]
stopped, is [ k  0.5] . [AIEEE 2005] (A) Mg ( 2  1) (B) Mg 2
(A) 800 m (B) 1000 m
(C) 100 m (D) 400 m Mg
Sol. (C) (D) Mg ( 2  1)
2
Sol.

19.A block is kept on a frictionless inclined surface


with angle of inclination . The incline is given an
acceleration a to keep the block stationary. Then a is
equal to [AIEEE 2005] 22.A block of mass m is connected to another block
of mass M by a spring (massless) of spring constant
k. The blocks are kept on a smooth horizontal plane.
Initially the blocks are at rest and the spring is
unstreched. Then a constant force F starts acting on
the block of mass M to pull it. Find the force on the
a block of mass m. [AIEEE 2007]
 mF ( M  m) F
(A) (B)
M m
g
(A) (B) g cosec  mF MF
tan 
(C) (D)
(C) g (D) g tan  (m  M ) (m  M )
Sol. Sol.

Corporate Head Office : Motion Education Pvt. Ltd., 394 - Rajeev Gandhi Nagar, Kota-5 (Raj.)
N.L.M. , FRICTION Page # 105
23.Two fixed frictionless inclined plane making an angle 24.The minimum force required to start pushing a body
30° and 60° with the vertical are shown in the figure. up a rough (frictional coefficient  ) inclined plane is
Two block A and B are placed on the two planes. F1 while the minimum force needed to prevent it from
What is the relative vertical acceleration of A with sliding down is F2. If the inclined plane makes an angle
respect to B ? [AIEEE 2010]
 from the horizontal such tan   2 , then the ratio
A F1
F2 is [AIEEE 2011]
B
(A) 4 (B) 1 (C) 2 (D) 3
Sol.

30°
60°

(A) 4.9 ms–2 in horizontal direction


(B) 9.8 ms–2 in vertical direction
(C) Zero
(D) 4.9 ms–2 in vertical direction
Sol.

: 0744-2209671, 08003899588 | url : www.motioniitjee.com, : info@motioniitjee.com


Page # 106 N.L.M. , FRICTION

LEVEL - II JEE ADVANCED


1. Two blocks A and B of equal masses are released 3. A circular disc with a groove along its diameter is
from an inclined plane of inclination 45° at t = 0. Both placed horizontally. A block of mass 1 kg is placed as
the blocks are initially at rest. The coefficient of kientic shown. The co-efficient of friction between the block
friction between the block A and the inclined plane is and all surfaces of groove in contact is  = 2/5. The
0.2 while it is 0.3 for block B. Initially, the block A is disc has an acceleration of 25 m/s 2 . Find the
acceleration of the block with respect to disc.
2 m behind the block B. When and where their front
[JEE 2006]
faces will come in line. [Take g = 10m/s2].

2m
a=25m/s2
A B

B
A
45°
Sol.
Sol.

2. Two blocks A and B masses 2m and m, respectively,


4. Two particles of mass m each are tied at the ends
are connected by a massless and inextensible string. of a light string of length 2a. The whole system is
The whole system is suspended by a massless spring kept on a frictionless horizontal surface with the string
as shown in the figure. The magnitudes of acceleration held tight so that each mass is at a distance ‘a’ from
of A and B, immediately after the string is cut, are
the center P (as shown in the figure). Now, the mid-
respectively. [JEE 2006]
point of the string is pulled vertically upwards with a
small but constant force F. As a result, the particles
move towards each other on the surfaces. The
magnitude of acceleration, when the separation
between them becomes 2x, is [JEE 2007]
2m A F

m B

(A) g, g (B) g, g/2 (C) g/2, g (D) g/2, g/2


Sol.
m m
P

a a
F a F x
(A) 2m (B) 2m
a2  x 2 a2  x 2

F x F a2  x2
(C) (D)
2m a 2m x

Corporate Head Office : Motion Education Pvt. Ltd., 394 - Rajeev Gandhi Nagar, Kota-5 (Raj.)
N.L.M. , FRICTION Page # 107

Sol. Sol.

7. A block of base 10 cm × 10 cm and height 15 cm is


kept on an inclined plane. The coefficient of friction
between them is 3 . The inclination  of this inclined
5. STATEMENT-1
plane from the horizontal plane is gradually increased
A cloth Covers a table. Some dishes are kept on it.
from 0º. Then [JEE 2009]
The cloth can be pulled out without dislodging the
(A) at  = 30º, the block will start sliding down the plane
dishes from the table
(B) the block will remain at rest on the plane up to
because
certain  and then it will topple
STATEMENT-2
(C) at  = 60º, the block will start sliding down the
For every action there is an equal and opposite reaction
plane and continue to do so at higher angles
(A) Statement-1 is True, Statement-2 is True;
(D) at  = 60º, the block will start sliding down the plane
State me nt -2 i s a correc t ex pl anat i on for
and on further increasing , it will topple at certain 
Statement-1
Sol.
(B) Statement-1 is True, Statement-2 is True;
Statement-2 is NOT a correct explanation for
Statement - 1
(C) Statement-1 is True, Statement-2 is False
(D) Statement-1 is False, Statement-2 is True
[JEE 2007]
Sol.

8. A piece of wire is bent in the shape of a parabola


y=kx2 (y-axis vertical) with a bead of mass m on it.
The bead can slide on the wire without friction. It
stays at the lowest point of the parabola when the
wire is at rest. The wire is now accelerated parallel to
the x-axis with a constant acceleration a. The dis-
tance of the new equilibrium position of the bead,
where the bead can stay at rest with respect to the
wire, from the y-axis is : JEE (2009)
6. STATEMENT-1
It is easier to pull a heavy object than to push it on a a a 2a a
level ground. (A) (B) (C) (D)
gk 2gk gk 4gk
and
Sol.
STATEMENT-2
The magnitude of frictional force depends on the
nature of the two surfaces in contact.
(A) Statement-1 is true, Statement-2 is true;
Statement-2 is a correct explanation for Statement-1
(B) Statement-1 i s true, statement-2 is true’
statement-2 is NOT a correct explanati on for
statement-1
(C) Statement-1 is true, statement-2 is false
(D) Statement-1 is false, statement-2 is true
[ JEE 2008]

: 0744-2209671, 08003899588 | url : www.motioniitjee.com, : info@motioniitjee.com


Page # 108 N.L.M. , FRICTION
9. A block of mass m is on an inclined plane of angle . 11. A small block of mass of 0.1 kg lies on a fixed
The coefficient of friction betwen the block and the inclined plane PQ which makes an angle  with the
plane is  and tan  > . The block is held stationary horizontal. A horizontal force of 1 N acts on the block
by applying a force P parallel to the plane. The direc- through its center of mass as shown in the figure.
tion of force pointing up the plane is taken to the The block remains stationary if (take g = 10 m/s2)
positive. As P is varied from P = mg (sin  –  cos  )
to Pz = mg (sin  +  cos ), the frictional force f
versus P graph will look like

(A)  = 45o
f (B)  > 45o and a frictional force acts on the block
f
towards P.
P2
(C)  > 45o and a frictional force acts on the block
(A) P1 P (B) towards Q.
P1 P2 P
(D)  < 45o and a frictional force acts on the block
towards Q.
Sol.
f
f
P1 P2
P1 P
(C) P2 P (D)

[JEE 2010]
Sol.

10. A block is moving on an inclined plane making an


angle 45º with horizontal and the coefficient of friciton
is . the force required to just push it up the inclined
plane is 3 times the force requried to just prevent it
from sliding down. If we define N = 10, then N is
[JEE 2011]
Sol.

Corporate Head Office : Motion Education Pvt. Ltd., 394 - Rajeev Gandhi Nagar, Kota-5 (Raj.)
N.L.M. , FRICTION Page # 109

Exercise - I OBJECTIVE PROBLEMS (JEE MAIN)

(A) NEWTONS'S LAW OF MOTION

1. B 2. B 3. C 4. A 5. A 6. B 7. B

8. A 9. D 10. C 11. B 12. A 13. B 14. C

15. B 16. C 17. B 18. C 19. A 20. A 21. B

22. C 23. B 24. C 25. C 26. C 27. C 28. B

29. C 30. A 31. B 32. B 33. D 34. B 35. B

36. C 37. A 38. A 39. C 40. A 41. A 42. D

43. C 44. A 45. B 46. C 47. B 48. A 49. B

50. C 51. A 52. A 53. D 54. A 55. A

Exercise - II
NLM

(SINGLE CORRECT)

1. A 2. A 3. A 4. B 5. A 6. A 7. A

8. D 9. A 10. B 11. A 12. C 13. A 14. A

15. B 16. B 17. A 18. B 19. C 20. C 21. A

22. B 23. A 24. A 25. B 26. B

27. (i) A (ii) A (iii) C (iv) D (v) B (vi) D

(vii) B (viii) B

28. (i) A (ii) A (iii) A (iv) C (v) B (vi) C

(vii) C (viii) B

29. A 30. B 31. A 32. C 33. D

(MULTIPLE CORRECT)
34. C 35. B 36. A,B,D 37. C 38. B 39. A,C 40. A,B,C
41. B,D 42. A,B,C 43. B 44. B 45. B 46. B 47. C

FRICTION

(SINGLE CORRECT)

fED fDC

48. E 2 m/s D 1 m/s C 3 m/s


fED
fDC fCS

: 0744-2209671, 08003899588 | url : www.motioniitjee.com, : info@motioniitjee.com


Page # 110 N.L.M. , FRICTION

fCB fBA
B 5 m/s 5 m/s A f Ag
fBA fAg

fkAB
49. fkAB A fkAB B 50. A 51. C 52. C 53. A
f kBG

54. B 55. B 56. A 57. A 58. A 59. D 60. B


61. D 62. C 63. (a) B (b) D (c) A 64. A 65. A 66. A
67. A 68. C 69. A 70. C 71. A 72. A
73. A,B 74. C 75. B 76. A,B,C 77. C 78. A,C
79. A,D 80. B,C 81. A,B 82. B,C 83. B,D 84. A,B,C,D

Exercise - III (JEE ADVANCED)

(A) NEWTON'S LAW OF MOTION

4g 2g 3g 4g 13mg
1. , , 2. a  ,T 3. (a) 5m/s2, (b) (i) 100N, (ii) 120N
5 5 5 9 18
4. 2 sec 5. x2 > x1 > x3 x1 : x2 : x3 : 15 : 18 : 10 6. 12 N 7. 10/3 kg
8. 300 N 9. 55 10. 5N, 16/31 kg 11. 2sec 12. 12 m/s
kl
13. aB = 4m/s2 () 14. (a) T = mg – , (b) length of spring will less than ‘l’ and T = 0 in the string.
2
15. 322 N
(m  M)(a  g) mg (m  M)(a  g)
16. (a) T  , (b) N = m(a + g) + T, (c) T = , (d) T 
2 2 3
mg
(e) N = m (a + g) – T, (f) T =
3
(B) FRICTION

17. (A)4m/s2, (B) 1.2 m/s2, (C) 0 18.  = 2

a
aB

aA
1
19. 20. 0.33 21.   r3 22. 1/2 23. 1kg
3
F
24. 4/3 sec 25. 3/4 26. 40 N 27. 1/2 sec 28. 5 sec, 125/6 m

29. 10 i 30. 0 31. 30 N

Corporate Head Office : Motion Education Pvt. Ltd., 394 - Rajeev Gandhi Nagar, Kota-5 (Raj.)
N.L.M. , FRICTION Page # 111

1. (a) 2 ms–2, (b) 2.4 N 0.3 (c) 0.2 s 2. 2 N 3. 0.5 sec 4. 556.8 N , 1.47 sec

 m1  2m 2  3g 
5.  2m g 6. (a) a A   aB ; aC  0 ; T  mg / 2 (b) a A  2g  ; aB  2g  ; a C  0, T  0
 2  2

3mg mg cot 
(c) a A  aB  g / 2  ; T  ; T  2mg 7. 2g/23 8.  r  , 1cm
2 4  2k

m sin  cos 
9. (a) a = g cot, (b)  min  10. s = 0.4 , k = 0.3
m cos 2   M

11. 12 N, 21 N, 4 m/s2, 2 m/s2, 4 N, 6 N

Exercise - IV PREVIOUS YEAR QUESTIONS

LEVEL - I JEE MAIN

1. C 2. B 3. C 4. A 5. A 6. A 7. D

8. C 9. D 10. D 11. A 12. A 13. A 14. D

15. C 16. A 17. D 18. B 19. D 20. D 21. D

22. C 23. D 24. D

LEVEL - II JEE ADVANCED

1. 11.313 m 2. B 3. 10 m/s2 4. B 5. B 6. B
7. B 8. B 9. A 10. 5N 11. A , C

: 0744-2209671, 08003899588 | url : www.motioniitjee.com, : info@motioniitjee.com


Page # 112 CIRCULAR MOTION

CIRCULAR MOTION
1. CIRCULAR MOTION
When a particle moves in a plane such that its distance from a fixed (or moving) point remains
constant then its motion is called as the circular motion with respect to that fixed (or moving)
point. That fixed point is called centre and the distance between fixed point and particle is called
radius.
v v v
A
1

2
B
The car is moving in a straight line with respect to the man A. But the man B continuously rotate
d
his face to see the car. So with respect to man A 0
dt
d
But with respect to man B 0
dt
Therefore we conclude that with respect to A the motion of car is straight line but for man B it has
some angular velocity

2. KINEMATICS OF CIRCULAR MOTION :


2.1 Variables of Motion :
(a) Angular Position : Y
The angle made by the position vector with given line (reference
line) is called angular position Circular motion is a two P'
dimensional motion or motion in a plane. Suppose a particle P
 P
is moving in a circle of radius r and centre O. The position of 
r x
O
the particle P at a given instant may be described by the angle
 between OP and OX. This angle  is called the angular position
of the particle. As the particle moves on the circle its angular
position  change. Suppose the point rotates an angle  in
time t.
(b) Angular Displacement :
Definition :
Angle rotated by a position vector of the moving particle in a given time interval with some
reference line is called its angular displacement.
______________________________________________________________________________________________
Important point :
• It is dimensionless and has proper unit SI unit radian while other units are degree or revolution
2 rad = 360° = 1 rev
• Infinitely small angular displacement is a vector quantity but finite angular displacement is not
because the addition of the small angular displacement is cummutative while for large is not.
   
d1 + d 2 = d 2 + d1 but 1 + 2  2 + 1
• Direction of small angular displacement is decided by right hand thumb rule. When the fingers
are directed along the motion of the point then thumb will represents the direction of angular
displacement.
• Angular displacement can be different for different observers

Corporate Head Office : Motion Education Pvt. Ltd., 394 - Rajeev Gandhi Nagar, Kota-5 (Raj.)
CIRCULAR MOTION Page # 113

______________________________________________________________________________________________
(c) Angular Velocity 
(i) Average Angular Velocity
Total Angle of Rotation
av = ;
Total time taken
 2 – 1 
av = t – t =
2 1 t
where 1 and 2 are angular position of the particle at time t1 and t2 respectively.
(ii) Instantaneous Angular Velocity
The rate at which the position vector of a particle with respect to the centre rotates, is called as
instantaneous angular velocity with respect to the centre.

lim  d
 = t0 =
t dt

_______________________________________________________________________________________________
Important points :
• It is an axial vector with dimensions [T–1] and SI unit rad/s.
• For a rigid body as all points will rotate through same angle in same time, angular velocity is a
characteristic of the body as a whole, e.g., angular velocity of all points of earth about its own
axis is (2/24) rad/hr.
• If a body makes ‘n’ rotations in ‘t’ seconds then angular velocity in radian per second will be
2n
av =
t
If T is the period and ‘f’ the frequency of uniform circular motion
2  1
av = = 2f
T
d
• If  = a – bt + ct2 then  = = – b + 2ct
dt
Relation between speed and angular velocity :
 d
= lim =
t t dt
The rate of change of angular velocity is called the angular acceleration (). Thus,

d d2  Y
= =
dt dt 2
P'
The linear distance PP’ travelled by the particle in time t is
P

s = r lim S
or t0
t X
O r
 s d
= r lim
t0  t or =r or v = r
t dt
Here, v is the linear speed of the particle
It is only valid for circular motion

 v
v = r is a scalar quantity (   )
r

: 0744-2209671, 08003899588 | url : www.motioniitjee.com, : info@motioniitjee.com


Page # 114 CIRCULAR MOTION

Ex.1 If  depends on time t in following way


 = 2t2 + 3 then
(a) Find out  average upto 3 sec. (b)  at 3 sec
Total angular displaceme nt  f –  i
Sol. avg = 
total time t 2 – t1
f = 2 (3)2 + 3 = 21 rad
i = 2 (0) + 3 = 3 rad.
21 – 3
So, avg = = 6 rad/sec
3
d
instantaneous = = 4t
dt
at t = 3 sec = 4 × 3 = 12 rad/sec

(d) Relative Angular Velocity


Angular velocity is defined with respect to the point from which the position vector of the moving
particle is drawn Here angular velocity of the particle w.r.t. ‘O’ and ‘A’ will be different

P'

P
A O Ref lin

d d
 PO  ;  PA 
dt dt
Definition :
Relative angular velocity of a particle ‘A’ with respect to the other moving particle ‘B’ is the angular
velocity of the position vector of ‘A’ with respect to ‘B’. That means it is the rate at which position
vector of ‘A’ with respect to ‘B’ rotates at that instant

VA
A
VB
r

( VAB ) 
 AB  here VAB   Relative velocity  to position vector AB
rAB

Re lative velocity of A w.r.t. B perpendicu lar to line AB



Seperation between A and B

( VAB )   VA sin 1  VB sin 2

rAB  r

VA sin 1  VB sin  2
 AB 
r

Corporate Head Office : Motion Education Pvt. Ltd., 394 - Rajeev Gandhi Nagar, Kota-5 (Raj.)
CIRCULAR MOTION Page # 115

_______________________________________________________________________________________
Important points :
• If two particles are moving on the same circle or different coplanar concentric circles in same
direction with different uniform angular speed A and B respectively, the rate of change of
 
angle between OA and OB is
B B
A A
d
 B   A O Initial line O Initial line
dt

So the time taken by one to complete one revolution around O w.r.t. the other
2 2 TT
T   1 2
 rel  2   1 T1  T2
• If two particles are moving on two different concentric circles with different velocities then
angular velocity of B relative to A as observed by A will depend on their positions and velocities.
consider the case when A and B are closest to each other moving in same direction as shown in
figure. In this situation
  B
v rel  | vB  v A |  v B  v A vB
r
  vA
A
rrel  | rB  rA |  rB  rA rB
rA
( vrel )  vB  v A O
so,  BA  
rrel rB  rA

( vrel )  = Relative velocity perpendicular to position vector


_______________________________________________________________________________________
Ex.2 Two particles move on a circular path (one just inside and the other just outside) with angular
velocities  and 5  starting from the same point. Then, which is incorrect.

(a) they cross each other at regular intervals of time when their angular velocities are

oppositely directed
(b) they cross each other at points on the path subtending an angle of 60° at the centre if
their angular velocities are oppositely directed

(c) they cross at intervals of time if their angular velocities are oppositely directed

(d) they cross each other at points on the path subtending 90° at the centre if their angular
velocities are in the same sense
Sol. If the angular velocities are oppositely directed, they meet at intervals of
2 2 
time t =  = =
rel 6 3
Angle subtended at the centre by the crossing points

 = t = = 60°
3
When their angular velocities are in the same direction,
2 2   
t’ =  = = and ’ =  =
rel 4 2 2 2
Ans. (a)

Ex.3 Two moving particles P and Q are 10 m apart at a certain instant. The velocity of P is 8 m/s
making 30° with the line joining P and Q and that of Q is 6 m/s making 30° with PQ in the
figure. Then the angular velocity of Q with respect to P in rad/s at that instant is

: 0744-2209671, 08003899588 | url : www.motioniitjee.com, : info@motioniitjee.com


Page # 116 CIRCULAR MOTION

6 m/s

P 30°
30° 10 m Q

8 m/s
(A) 0 (B) 0.1 (C) 0.4 (D) 0.7
6 m/s

P 30°
Sol. 30° 10 m Q
8 m/s

Projection of VQP perpendicular to the line PQ


Angular velocity of Q relative to P =
Separation between P and Q

VQ sin  2 – VP sin 1 6 sin 30–(–8 sin 30 )


= = 0.7 rad/s
PQ 10
 (D)

(e) Angular Acceleration  :


(i) Average Angular Acceleration :
Let 1 and 2 be the instantaneous angular speeds at times t1 and t2 respectively, then the
average angular acceleration av is defined as

 2 – 1 
av = t – t =
2 1 t
(ii) Instantaneous Angular Acceleration :
It is the limit of average angular acceleration as t approaches zero, i.e.,

lim  = d =  d
 = t0
t dt d
_______________________________________________________________________________________________
Important points :
• It is also an axial vector with dimension [T–2] and unit rad/s2
• If  = 0, circular motion is said to be uniform.

d d d2
• As  = , = = ,
dt dt dt 2
i.e., second derivative of angular displacement w.r.t time gives angular acceleration.
•  is a axial vector and direction of  is along 
if  increases and opposite to  if  decreases
_______________________________________________________________________________________
(f) Radial and tangential acceleration

Acceleration of a particle moving in a circle has two components one is along e t (along tangent)
and the other along  ê r (or towards centre). Of these the first one is the called the tangential
acceleration. (at) and the other is called radial or centripetal acceleration (ar). Thus.

Corporate Head Office : Motion Education Pvt. Ltd., 394 - Rajeev Gandhi Nagar, Kota-5 (Raj.)
CIRCULAR MOTION Page # 117

dv
at  = rate of change of speed
dt
2 2
v
and ar  2r  r   v
r r
Here, at is the component which is responsible for changing the magnitude of speed of the particle
in circular motion. ar is the component which is responsible for changing the direction of particle in
circular motion.
the two component are mutually perpendicular. Therefore, net acceleration of the particle will be :
2 2 2
 dv   v2   dv 
a  ar2  a 2t  (r 2 ) 2         
 dt   r   dt 
Following three points are important regarding the above discussion :
dv
1. In uniform circular motion, speed (v) of the particle is constant, i.e.,  0 . Thus,
dt
at = 0 and a = ar = r2
dv
2. In accelerated circular motion, = positive, i.e., at is along e t or tangential acceleration of
dt
  dv 
particle is parallel to velocity v because v  r e t and ar  et
dt
dv
3. In decelerated circular motion, = negative and hence, tangential acceleration is anti-parallel
dt

to velocity v .

(g) Relation between angular acceleration and tangential acceleration


we know that
v = r
Here, v is the linear speed of the particle
Differentiating again with respect to time, we have
dv d
at = =r or at = r
dt dt
dv
Here, at = is the rate of change of speed (not the rate of change of velocity).
dt
Ex.4 A particle travels in a circle of radius 20 cm at a speed that uniformly increases. If the speed
changes from 5.0 m/s to 6.0 m/s in 2.0s, find the angular acceleration.
Sol. The tangential acceleration is given by

dv v 2 – v1
at = = t –t
dt 2 1

v dv
( Here speed increases uniformly at = = )
t dt
6.0 – 5.0
= m/s2 = 0.5 m/s2
2.0
The angular acceleration is  = at/r
0.5 m / s 2
= = 2.5 rad/s2
20 cm

Ex-5 A particle moves in a circle of radius 20 cm. Its linear speed at any time is given by v = 2t
where v is in m/s and t is in seconds. Find the radial and tangential acceleration at t = 3
seconds and hence calculate the total acceleration at this time.
Sol. The linear speed at 3 seconds is

: 0744-2209671, 08003899588 | url : www.motioniitjee.com, : info@motioniitjee.com


Page # 118 CIRCULAR MOTION

v = 2 × 3 = 6 m/s
The radial acceleration at 3 seconds
v2 66
= = = 180 m/s2
r 0.2
The tangential acceleration is given by
dv
 2 , because v = 2t.
dt
 tangential acceleration is 2 m/s2.

Net Acceleration = ar 2  a t 2 = (180)2  (2)2 = 180.01 m/s2

T-1 Is it possible for a car to move in a circular path in such a way that it has a tangential acceleration
but no centripetal acceleration ?

Ex.6 A particle moves in a circle of radius 2.0 cm at a speed given by v = 4t, where v is in cm/s
and t in seconds.
(a) Find the tangential acceleration at t = 1 s.
(b) Find total accleration at t = 1 s.
Sol. (a) Tangential acceleration
dv d
at = or at = (4 t) = 4 cm/s2
dt dt

v2 ( 4)2
ac = = =8  a= a2t  a2c = ( 4)2  (8) 2 = 4 5 m / s
2
R 2

Ex.7 A boy whirls a stone in a horizontal circle of radius 1.5 m and at height 2.0 m above level
ground. The string breaks, and the stone files off horizontally and strikes the ground after
traveling a horizontal distance of 10 m. What is the magnitude of the cetripetal acceleration
of the stone while in circular motion ?

2h 22
Sol. t= = = 0.64 s
g 9.8
10
v= = 15.63 m/s
t
vB2
a= = 0.45 m/s2
R

Ex.8 Find the magnitude of the acceleration of a particle moving in a circle of radius 10 cm with
uniform speed completing the circle in 4 s.
Sol. The distance covered in completing the circle is 2  r = 2 × 10 cm. The linear speed is
2  10cm
v = 2  r/t = = 5  cm/s.
4s
2
v 2 ( 5cm / s)
The acceleration is ar = = =2.5 2 cm/s2
r 10 cm

Ex.9 A particle moves in a circle of radius 20 cm. Its linear speed is given by v = 2t where t is in
second and v in meter/second. Find the radial and tangential acceleration at t = 3s.
Sol. The linear speed at t = 3s is
v = 2t = 6 m/s
The radial acceleration at t = 3s is

Corporate Head Office : Motion Education Pvt. Ltd., 394 - Rajeev Gandhi Nagar, Kota-5 (Raj.)
CIRCULAR MOTION Page # 119

36m 2 / s 2
ar = v2/r = = 180 m/s2
0.20 m
The tangential acceleration is
dv d( 2t)
at = = = 2 m/s2
dt dt
Y
Ex.10 Two particles A and B start at the origin O and travel in cm
opposite directions along the circular path at constant 5. 0
speeds vA = 0.7 m/s and vB = 1.5 m/s, respectively. B A
Determine the time when they collide and the magnitude
of the acceleration of B just before this happens. vB =1.5 m/s O x
vA=0.7m/s
10
Sol. 1.5 t + 0.7 t = 2R = 10  t = = 14.3 s
2.2

vB2
a= = 0.45 m/s2
R
Uniform circular Non-uniform circular
motion motion

(1) Speed of the particle speed of the particle is


is constant i.e.,  is not constant i.e. is not
constant constant
 
d| v| d| v|
(ii) a t  0 at  0
dt dt

v2
ar  0 ar  0
r
  
 anet  ar anet  ar  a t
anet at

ar= anet
ar

(h) Relations among Angular Variables


These relations are also referred as equations of rotational motion and are -
 = 0 + t ...(1)
d,  or 
1 2
 = 0t + t ...(2)
2
2 = 02 + 2 ...(3)
These are valid only if angular acceleration is constant and ar O
are analogous to equations of translatory motion, i.e.,
vo ds r
ra
1 2 t
v = u + at ; s = ut + at and v2 = u2 + 2as
2

: 0744-2209671, 08003899588 | url : www.motioniitjee.com, : info@motioniitjee.com


Page # 120 CIRCULAR MOTION

3. DYNAMICS OF CIRCULAR MOTION :


In circular motion or motion along any curved path Newton’s law is applied in two perpendicular
directions one along the tangent and other perpendicular to it. i.e., towards centre. The compnent
of net force towards the centre is called centripetal force. The component of net force along the
tangent is called tangential force.
dv
tangential force (Ft) = Mat = M = M  r ; where  is the angular acceleration
dt
mv 2
centripetal force (Fc) = m 2 r =
r
Ex.11 A small block of mass 100 g moves with uniform speed in a horizontal circular groove, with
vertical side walls, of radius 25 cm. If the block takes 2.0s to complete one round, find the
normal contact force by the slide wall of the groove.
Sol. The speed of the block is
2  ( 25 cm)
v= = 0.785 m/s
2.0 s
The acceleration of the block is
v2 (0 .785 m / s ) 2
a= = = 2.46 m/s2
r 0 . 25
towards the center. The only force in this direction is the normal contact force due to the side
walls. Thus from Newton’s second law, this force is
N = ma = (0.100 kg) (2.46 m/s2) = 0.246 N

3.1 Centripetal Force :

Concepts : This is necessary resultant force towards the centre called the centripetal force.
mv 2
F= = m2r
r
(i) A body moving with constant speed in a circle is not in equilibrium.
(ii) It should be remembered that in the absence of the centripetal force the body will move in
a straight line with constant speed.
(iii) It is not a new kind of force which acts on bodies. In fact, any force which is directed
towards the centre may provide the necessary centripetal force.

Ex.12 A small block of mass 100 g moves with uniform speed in a horizontal circular groove, with
vertical side walls, of radius 25 cm. If the block takes 2.0s to complete one round, find the
normal contact force by the slide wall of the groove.
Sol. The speed of the block is
2  ( 25 cm)
v= = 0.785 m/s
2.0 s
The acceleration of the block is
v2 (0 .785 m / s ) 2
a= = = 2.5 m/s2
r 0 . 25
towards the center. The only force in this direction is the normal contact force due to the slide
walls. Thus from Newton’s second law, this force is
N = ma = (0.100 kg) (2.5 m/s2) = 0.25 N

3.2 Centrifugal Force :


When a body is rotating in a circular path and the centripetal force vanishes, the body would leave
the circular path. To an observer A who is not sharing the motion along the circular path, the body
appears to fly off tangentially at the point of release. To another observer B, who is sharing the
motion along the circular path (i.e., the observer B is also rotating with the body which is released,

Corporate Head Office : Motion Education Pvt. Ltd., 394 - Rajeev Gandhi Nagar, Kota-5 (Raj.)
CIRCULAR MOTION Page # 121

it appears to B, as if it has been thrown off along the radius away from the centre by some force.
This inertial force is called centrifugal force.)
mv 2
Its magnitude is equal to that of the centripetal force = . Centrifugal force is a fictitious force
r
which has to be applied as a concept only in a rotating frame of reference to apply Newton’s law of
motion in that frame)
FBD of ball w.r.t non inertial frame rotating with the ball.


T

m 2r
mg

Suppose we are working from a frame of reference that is rotating at a constant, angular velocity
 with respect to an inertial frame. If we analyse the dynamics of a particle of mass m kept at a
distance r from the axis of rotation, we have to assume that a force mr2 act radially outward on
the particle. Only then we can apply Newton’s laws of motion in the rotating frame. This radially
outward pseudo force is called the centrifugal force.
T-2 A particle of mass m rotates in a circle of radius r with a uniform angular speed . It is viewed from
a frame rotating about same axis with a uniform angular speed . The centrifugal force on the
particle is
2
   0 
(A) m r
2
(B) m r
2
(C) m  r (D) m0r
0  2 

B
: A rod move with  angular velocity then we conclude
following for point A & B in a rod.
A = B s B > sA A
A = B vB > vA 
A = B atB > atA

Ex.13 Find out the tension T1, T2 is the string as shown in figure

2 rad/sec.
T1 T2  rad/sec.
1kg 2kg T2
1m 1m T1
m1 m2

We know that m1  m 2


 T1 = m12R1 + T2
T2 = m 2 R2
T2 = 2 × 4 × 2 = 16 N
So T1 = (1) (2)2 (1) + 16 N = 4 + 16 N
T1 = 20 N

: 0744-2209671, 08003899588 | url : www.motioniitjee.com, : info@motioniitjee.com


Page # 122 CIRCULAR MOTION

4. SIMPLE PENDULUM

Ex.14 A simple pendulum is constructed by attaching a bob of mass m to a string of length L fixed
at its upper end. The bob oscillates in a vertical circle. It is found that the speed of the bob
is v when the string makes an angle  with the vertical. Find the tension in the string at this
instant.
Sol. The force acting on the bob are (figure)
(a) the tension T (b) the weight mg.
As the bob moves in a vertical circle with centre at O,
the radial acceleration is v2/L towards O. Taking the
components along this radius and applying Newton’s
second law, we get
mgcos
T – mgcos  = mv2/L or, T = m(gcos  + v2/L) mg
mgsin
2
 2
 mv 2  v4
|Fnet | = (mg sin )    2 2
= m g sin  
 L  L2
 

5. CIRCULAR MOTION IN HORIZONTAL PLANE



A ball of mass m attached to a light and inextensible string

rotates in a horizontal circle of radius r with an angular speed
T Tcos 
 about the vertical. If we draw the force diagram of the ball. 
We can easily see that the component of tension force along m 2r
T sin
the centre gives the centripetal force and component of tension
along vertical balances the gravitation force. Such a system is mg
called a conical pendulum. FBD of ball w.r.t ground

Ex. 15 A particle of mass m is suspended from a ceiling through a string of length L. The particle
moves in a horizontal circle of radius r. Find (a) the speed of the particle and (b) the tension
in the string.
Sol. The situation is shown in figure.
The angle  made by the string with the vertical is given by
sin = r/L ... (i) T 
L
The forces on the particle are
(a) the tension T along the string and
r
(b) the weight mg vertically downward.
The particle is moving in a circle with a constant speed
mg
v. Thus, the radial acceleration towards the centre has
magnitude v2/r. Resolving the forces along the radial
direction and applying. Newton’s second law,
Tsin  = m(v2/r) ...(ii)
As there is no acceleration in vertical directions, we have from Newton’s law,
Tcos = mg ...(iii)
Dividing (ii) by (iii),

Corporate Head Office : Motion Education Pvt. Ltd., 394 - Rajeev Gandhi Nagar, Kota-5 (Raj.)
CIRCULAR MOTION Page # 123

v2
tan = or v = rgtan
rg
mg
And from (iii), T =
cos 
r g mgL
Using (i), v = and T=
2
(L – r )2 1/ 4
(L – r 2 )1/ 2
2

6. MOTION OF A MOTORCYCLIST ON A CURVED PATH.

A cylist having mass m move with constant speed v on a curved path as shown in figure.

D
A
C E
We divide the motion of cyclist in four parts :
(1) from A to B (2) from B to C
(3) from C to D (4) from D to E
(1 and 3 are same type of motion)

(A) Motion of cyclist from A to B


mv 2 mv 2
N = mg cos  N+
R R
f B
2
mv
 N = mg cos – ...(1)
R s
f = mg sin  ..(2) gco
mgsin  m

(1) As cyclist move upward A mg
  decreases & cos  increases
 N increases
and
  decreases sin  decreases
 friction force required to balance mg sin  (As cyclist is moving with constant speed) also
decreases

(B) Motion of cyclist from B to C


B mv2
mv2 N
N  mg cos f R
R

mv 2 mgcos 
 N = mg cos – ...(1) mgsin
R
f = mg sin ...(2) mg C
Therefore from B to C Normal force decrease but
friction force increase becuse  increases.

(C) Motion of cyclist from D to E

mv 2
N  mg cos f = mg sin 
R

: 0744-2209671, 08003899588 | url : www.motioniitjee.com, : info@motioniitjee.com


Page # 124 CIRCULAR MOTION

from D to E  decreases D
therefore mg cos  increase
f
So N increase but f decreases

mv2
E
R
mg
Ex.16 A hemispherical bowl of radius R is rotating about its axis of symmetry which is kept vertical.
A small ball kept in the bowl rotates with the bowl without slipping on its surface. If the
surface of the bowl is smooth and the angle made by the radius through the ball with the
vertical is . Find the angular speed at which the bowl is rotating.
Sol. Let  be the angular speed of rotation of the bowl.

Two force are acting on the ball.
1. normal reaction N 2. weight mg
The ball is rotating in a circle of radius r (= R sin ) with
centre at A at an angular speed . Thus, R 
N sin  = mr2 = mR2 sin  N
r A
N = mR2 ...(i)
and N cos  = mg ...(ii)
mg
Dividing Eqs. (i) by (ii),

1  2R g
we get   =
cos  g R cos 

Ex.17 If friction is present between the surface of ball and bowl then find out the range of  for
which ball does not slip (is the friction coefficient)
Friction develop a range of  for which the particle will be at rest.
Sol.
(a) When  > 0
In this situation ball has a tendency to slip upwards
so the friction force will act downwards. So F.B.D of ball 
N = m2r sin  + mg cos . ...
f + mg sin  = m2r cos  ...(2) N
 fmax = N = (m2r sin  + mg cos)  m r
r = R sin  2
m r
Substituting the values of fmax & r in eq. (2) we get

  (m2r sin  + mg cos)  m2r cos –mg sin 
 (m2R sin 2 + mg cos )  m2 R sin cos f
mg
– mg sin 

gcos  gsin 

R sin (cos –  sin)


(b) when < 0 f
In this situation ball has a tendency to slip downwards N
so the friction force will act upwards. So F.B.D of ball 
2 m
m r
r
 f + m2r cos  = mg sin  
f = m (g sin – 2r cos ) ...(1)
N = mg cos  + m2r sin  ...(2)
fmax = N = (mg cos  + m2r sin) mg
for equillibrium

Corporate Head Office : Motion Education Pvt. Ltd., 394 - Rajeev Gandhi Nagar, Kota-5 (Raj.)
CIRCULAR MOTION Page # 125

 (g cos  + 2r sin )  (gsin – 2 r cos )


 Substituting r = Rsin then

g(sin  –  cos  )

R sin ( sin   cos  )

7. CIRCULAR TURNING ON ROADS :

When vehicles go through turnings, they travel along a nearly circular arc. There must be some
force which will produce the required centripetal acceleration. If the vehicles travel in a horizontal
circular path, this resultant force is also horizontal. The necessary centripetal force is being
provided to the vehicles by following three ways.
1. By Friction only
2. By Banking of Roads only
3. By Friction and Banking of Roads both.
In real life the necessary centripetal force is provided by friction and banking of roads both. Now let
us write equations of motion in each of the three cases separately and see what are the constant
in each case.

7.1 By Friction Only


Suppose a car of mass m is moving at a speed v in a horizontal circular arc of radius r. In this case,
the necessary centripetal force to the car will be provided by force of friction f acting towards
center
mv 2
Thus, f 
r
Further, limiting value of f is N
or fL = N = mg (N = mg)
mv 2
Therefore, for a safe turn without sliding  fL
r
mv 2 v2
or  mg or   or v  rg
r rg
Here, two situations may arise. If  and r are known to us, the speed of the vehicle should not
v2
exceed rg and if v and r are known to us, the coefficient of friction should be greater than .
rg

7.2 By Banking of Roads Only


Friction is not always reliable at circular turns if high speeds and sharp turns are involved to avoid
dependence on friction, the roads are banked at the turn so that the outer part of the road is some
what lifted compared to the inner part.
Applying Newton’s second law along the N
radius and the first law in the vertical direction.
mv 2
Nsin = or N cos  mg
r

from these two equations, we get
W
v2
tan = or v= rgtan
rg

: 0744-2209671, 08003899588 | url : www.motioniitjee.com, : info@motioniitjee.com


Page # 126 CIRCULAR MOTION

7.3 By Friction and Banking of Road Both


If a vehicle is moving on a circular road which is rough and banked also, then three forces may act
on the vehicle, of these force, the first force, i.e., weight (mg) is fixed both in magnitude and
direction.
N


y

f
mg
 x
Figure (ii)
The direction of second force i.e., normal reaction N is also fixed (perpendicular or road) while the
direction of the third i.e., friction f can be either inwards or outwards while its magnitude can be
vari ed upt o a maxi mum l i mi t
(fL = N). So the magnitude of normal reaction N and directions plus magnitude of friction f are so
mv 2
adjusted that the resultant of the three forces mentioned above is towards the center. Of
r
these m and r are also constant. Therefore, magnitude of N and directions plus magnitude of
friction mainly depends on the speed of the vehicle v. Thus, situation varies from problem to
problem. Even though we can see that :
(i) Friction f will be outwards if the vehicle is at rest v = 0. Because in that case the component
weight mg sin is balanced by f.
(ii) Friction f will be inwards if
v > rgtan
(iii) Friction f will be outwards if
v< rgtan and
(iv) Friction f will be zero if
v= rgtan
(v) For maximum safe speed (figure (ii)
mv 2
N sin + f cos = ...(i)
r
N cos – f sin = mg ...(ii)
As maximum value of friction
f = N

sin    cos  v2 rg(  tan )


 =  vmax =
cos  –  sin  rg (1 –  tan )

rg( – tan )
Similarly ; vmin =
(1  tan )

v2
: • The expression tan  = also gives the angle of banking for an aircraft, i.e., the angle
rg
through which it should tilt while negotiating a curve, to avoid deviation from the circular
path.
v2
• The expression tan  = also gives the angle at which a cyclist should lean inward, when
rg
rounding a corner. In this case,  is the angle which the cyclist must make with the vertical to
negotiate a safe turn.

Corporate Head Office : Motion Education Pvt. Ltd., 394 - Rajeev Gandhi Nagar, Kota-5 (Raj.)
CIRCULAR MOTION Page # 127

8. DEATH WELL :

A motor cyclist is driving in a horizontal circle on the inner surface of vertical cyclinder of radius R.
Friction coefficient between tyres of motorcyclist and surface of cylinder is . Find out the minimum
velocity for which the motorcyclist can do this. v is the speed of motor cyclist and m is his mass.
mv 2 f
N=
R mv 2 N
f = mg R
mv 2 mg
fmax =
R
Cyclist does not drop down when
mv 2
fmax  mg   mg
R
gR
v

9. MOTION OF A CYCLIST ON A CIRCULAR PATH :

Suppose a cyclist is going at a speed v on a circular horizontal road of radius r which is not banked.
Consider the cycle and the rider together as the system. The centre of mass C (figure shown) of
the system is going in a circle with the centre at O and radius r.

B
Mv 2 
r
C

N Mg

A f D
(b)
Let us choose O as the origin, OC as the X-axis and vertically upward as the Z-axis. This frame is
v
rotating at an angular speed  = about the Z-axis. In this frame the system is at rest. Since we
r
are working from a rotating frame of reference, we will have to apply a centrifugal force on each
particle. The net centrifugal force on the system will be M2r = Mv2/r, where M is the total mass of
the system. This force will act through the centre of mass. Since the system is at rest in this
frame, no other pseudo force is needed.
Figure in shows the forces. The cycle is bent at an angle  with the vertical. The forces are
(i) weight Mg,
(ii) normal force N
(iii) friction f and
Mv 2
(iv) centrifugal force
r
In the frame considered, the system is at rest. Thus, the total external force and the total external
torque must be zero. Let us consider the torques of all the forces about the point A. The torques of
N and f about A are zero because these forces pass through A. The torque of Mg about A is Mg(AD)
Mv 2 Mv 2
in the clockwise direction and that of is (CD) in the anticlockwise direction. For rotational
r r
equilibrium,

: 0744-2209671, 08003899588 | url : www.motioniitjee.com, : info@motioniitjee.com


Page # 128 CIRCULAR MOTION

Mv 2
Mg(AD) = (CD)
r

AD v 2
or, 
CD rg

v2
or, tan = ...(10.9)
rg

 v2 
Thus, the cyclist bends at an angle tan –1   with the vertical.
 rg 

T.3 A car driver going at a speed of v suddenly finds a wide wall at a distance r. Should he apply breaks
or turn the car in a circle of radius r to avoid hitting the wall ?

10. EFFECT OF EARTHS ROTATION ON APPARENT WEIGHT :

The earth rotates about its axis at an angular speed of


one revolution per 24 hours. The line joining the north
and the south N
poles is the axis of rotation.
Every point on the earth moves in a circle. A point at
equator moves in a circle of radius equal to the radius C P
of the earth and the centre of the circle is same as  M
the centre of the earth. For any other point on the O R
earth, the circle of rotation is smaller than this. Consider
a place point on the earth (figure.)
Drop a perpendicular PC from P to the axis SN. The S
place P rotates in a circle with the centre at C. The
radius of this circle is CP. The angle between the line
OM and the radius OP through P is called the latitude of
the place point. We have
CP = OP cos or, r = R cos
where R is the radius of the earth.
If we calculate work from the frame of reference of the earth,
we shall have to assume the existence of pseudo force. In
Particular, a centrifugal force m2r has to be assumed on any North
particle of mass m placed at P. If we consider a f
block of mass m at point P then this block is at rest with N
respect to earth. If resolve the forces along and perpendicular C mr 2
to the 
mg
line joining the centre of earth then O R
N + mr2 cos = mg  N = mg – mr2cos
 N = mg – mR2 cos2
South

Ex.18 A body weighs 98N on a spring balance at the north pole. What will be its weight recorded on
the same scale if it is shifted to the equator ? Use g = GM/R2 = 9.8 m/s2 and the radius of
the earth R = 6400 km.
Sol. At poles, the apparent weight is same as the true weight.
Thus, 98N = mg = m(9.8 m/s2)
At the equator, the apparent weight is
mg’ = mg – m2R
The radius of the earth is 6400 km and the angular speed is

Corporate Head Office : Motion Education Pvt. Ltd., 394 - Rajeev Gandhi Nagar, Kota-5 (Raj.)
CIRCULAR MOTION Page # 129

2 rad
=
24  60  60s
= 7.27 × 10–6 rad/s
mg’ = 98N – (10 kg) (7.27 × 10–5 s–1)2 (6400 km) = 97.66 N

SOME SOLVED EXAMPLES

Ex.1 Two blocks each of mass M are connected to the


ends of a light frame as shown in figure. The frame
is rotated about the vertical line of symetry. The M M
rod breaks if the tension in it exceeds T0. Find the
maximum frequency with which
the frame may be rotated without breaking the rod.

Sol. Consider one of the blocks. If the frequency of revolution is f, the angular velocity is  = 2f. The
acceleration towards the centre is 2 = 4 2 f2  . The only horizontal force on the block is the
tension of the rod. At the point of breaking, this force is T0. So from Newton’s law,
1/ 2
T0 = M.4 2 f 2 l or, 1  T0 
f
2  M 

Ex.2 Prove that a motor car moving over a convex bridge is lighter than the same car resting on
the same bridge.
Sol. The motion of the motor car over a convex bridge AB is the motion along the segment of a circle AB
(figure);
R

A mg B

The centripetal force is provided by the difference of weight mg of the car and the normal
reaction R of the bridge.
mv 2 mv 2
 mg  R  or R  mg 
r r
Clearly R < mg, i.e., the weight of the moving car is less than the weight of the stationary car.

Ex.3 A body weighing 0.4 kg is whirled in a vertical circle with a string making 2 revolutions per
second. If the radius of the circle is 1.2m. Find the tension (a) at the top of the circle, (b) at
the bottom of the circle. Give : g = 10 m s–2 and  = 3.14
Sol. Mass, m = 0.4 kg ;
1
time period = second, radius, r = 1.2 m
2
2
Angular velocity,  = = 4 rad s–1 = 12.56 rad s–1
1/ 2
mv 2
(a) At the top of the circle, T = –mg
r
= mr – mg = m (r2 – g)
2

= 0.4 (1.2 × 12.56 × 12.56 – 9.8) N = 71.2 N


(b) At the lowest point, T = m(r2 + g) = 80 N

: 0744-2209671, 08003899588 | url : www.motioniitjee.com, : info@motioniitjee.com


Page # 130 CIRCULAR MOTION

Ex.4 A metal ring of mass m and radius R is placed on a smooth horizontal table and is set rotating
about its own axis in such a way that each part of ring moves with velocity v. Find the
tension in the ring.
Sol. Consider a small part ACB of the ring that subtends
an angle  at the centre as shown in figure.
Let the tension in the ring be T. T
The forces on this elementary portion ACB are 
A
(i) tension T by the part of the ring left to A 2
(ii) tension T by the part of the ring to B O
C
(iii) weight (m) g 
(iv) nomal force N by the table. 2 B
As the elementary portion ACB moves in a circle of
radius R at constant speed v its acceleration towards T

( m)v 2
centre is .
R
Resolving the force along the radius CO
   
T cos90 –  + T cos 90 –
  v2
 = m ...(i)
 2  2 R
 v2
2Tsin = m ...(ii)
2 R
m
Length of the part ACB = R. The mass per unit length of the ring is
2R
Rm m
 mass of this portion ACB, m = =
2R 2
Putting this value of m in (ii),
 mv 2
2Tsin =
2 2R

     
mv 

2   sin 
T=
2  Since  is small So  2  is equal to 1,
2R        
 sin    
  2    2 
mv 2
T=
2R
Ex.5 A small smooth ring of mass m is threaded on a light inextensible string of length 8L which
has its ends fixed at points in the same vertical line at a distance 4L apart. The ring describes
horizontal circles at constant speed with both parts of the string taut and with the lower
portion of the string horizontal. Find the speed of the ring and the tension in the string. The
ring is then tied at the midpoint of the string and made to perform horizontal circles at
constant speed of 3 gL . Find the tension in each part of the string.
Sol. When the string passes through the ring, the tension in the string is the same
in both parts. Also from geometry
BP = 3L and AP = 5L A
4
T cos = T = mg ...(i) 
5
5L
 3 8 4L
T + T sin  = T 1   = T
 5 5
3L ring
B P
T
mg

Corporate Head Office : Motion Education Pvt. Ltd., 394 - Rajeev Gandhi Nagar, Kota-5 (Raj.)
CIRCULAR MOTION Page # 131

mv 2 mv 2
= = ...(ii)
BP 3L

v2
2
3Lg

v= 6Lg A
60° 4L
mg 5 T1
From (i) T = = mg ring
4/5 4 4L
P
In the second case, ABP is an equilateral triangle.
T1cos60° = mg + T2 cos60° T2
4L mg
mg B
T 1 – T2 = = 2mg ...(iii)
cos 60

mv 2 9mgL
T1sin60° + T2sin60° = =
r 4L sin 60
9mg
T1 + T2 = = 3 mg ...(iv)
4 sin2 60
Solving equation (iii) and (iv)
5 1
T1 = mg ; T2 = mg
2 2

Ex.6 A large mass M and a small mass m hang at the two ends of the string that passes through
a smooth tube as shown in Figure. The mass m moves around in a circular path, which lies in
the horizontal plane. The length of the string from the mass m to the top of the tube is  and
 is the angle this length makes with vertical. What should be the frequency of rotation of
mass m so that M remains stationary ?

Horizontal  l
plane
T Tcos
Tsin m

Sol. mg

M
Mg
The forces acting on mass m and M are shown in Figure. When mass M is stationary
T = Mg ..(i)
where T is tension in string.
For the smaller mass, the vertical component of tension T cos  balances mg and the horizontal
component T sin  supplies the necessary centripetal force.
T cos = mg ...(ii)
T sin  = mr2 ...(iii)
 being the angular velocity and r is the radius of horizontal circular path.
Form (i) and (iii), Mg sin  = mr2
Mg sin Mg sin  Mg
= = =
mr m  sin  m
1 1  1 Mg
Frequency of rotation = = =  Frequency =
T 2 /  2 2 ml

: 0744-2209671, 08003899588 | url : www.motioniitjee.com, : info@motioniitjee.com


Page # 132 CIRCULAR MOTION

Ex.7 The 4 kg block in the figure is attached to the vertical rod by means of two strings. When the
system rotates about the axis of the rod, the two strings are extended as indicated in Figure.
How many revolutions per minute must the system make in order that the tension in upper
string is 60 N. What is tension in the lower string ?

A
1.25 m

P
O
2m
4 kg

1.25 m

B A
Sol. The forces acting on block P of mass 4 kg are shown in the
Figure. If  is the angle made by strings with vertical, T1 and T2 
tensions in strings for equilibrium in the vertical direction T1 T1 cos
T1cos = T2cos + mg
(T1 – T2) cos  = mg O m
P
1 4  OA 1  T2 cos
cos  = =  cos   AP  125
.  
125
. 5   mg
T2

mg 5mg 5
 T1 – T2 = = = × 4 × 9.8 = 49 N
cos  4 4
Given T1 = 60 N
T2 = T1 – 49 = 60 N – 49 N = 11 N
The net horizontal force (T1sin + T2sin) provides the necessary centripetal force m2r.
 (T1 + T2) sin  = m2r
( T1  T2 )sin 
 2 =
mr
3
sin  = 1 – cos 2  = 1 – (4 / 5) 2 = 5
3
( 60  11)
r = OP = 125
. – 1 = 0.75   =
2 2 2 5 = 0.75   = 14.2 = 3.768 rad/s
4  0.75
 3.768
Frequency of revolution = = = 0.6 rev/s or 36 rev/min
2 2  3.14

Corporate Head Office : Motion Education Pvt. Ltd., 394 - Rajeev Gandhi Nagar, Kota-5 (Raj.)
WORK, POWER & ENERGY Page # 133

WORK, POWER & ENERGY


1. WORK :

Work is said to be done by a force when the force produces a displacement in the body on which it
acts in any direction except perpendicular to the direction of the force.

1.1 Work done by constant force


Consider an object undergoes a displacement S along a straight line while acted on a force F that
makes an angle  with S as shown.
The work done W by the agent is the product of the component of force in the direction of
displacement and the magnitude of displacement.
i.e., W = FS cos  ...(1)
F

S
Work done is a scalar quantity and its S.I. unit is N-m or joule (J). We can also write work done as
a scalar product of force and displacement.
 
W  F. S ...(2)
where S is the displacement of the point of application of the force
From this definition, we conclude the following points
(A) work done by a force is zero if displacement is perpendicular to the force ( = 90°)


T


S
Example.
The tension in the string of a simple pendulum is always perpendicular to displacement. (Figure)
So, work done by the tension is zero.
(B) if the angle between force and displacement is acute ( < 90°), we say that the work done by the
force is positive.
Example :
When a load is lifted, the lifting force and the displacement act in the same direction. So, work
done by the lifting force is positive.
Example :
When a spring is stretched, both the stretching force and the displacement act in the same
direction. So work done by the stetching force is positive.
(C) If the angle between force and displacement is obtuse ( > 90°), we say that the work done by the
force is negative.
Example :
When a body is lifted, the work done by the gravitational force is negative. This is because the
gravitational force acts vertically downwards while the displacement is in the vertically upwards
direction.
__________________________________________________________________________________
Important points about work :
1. Work is said to be done by a force when its point of application moves by some distance.Force does
no work if point of application of force does not move (S = 0)
Example :
A person carrying a load on his head and standing at a given place does no work.

: 0744-2209671, 08003899588 | url : www.motioniitjee.com, : info@motioniitjee.com


Page # 134 WORK, POWER & ENERGY

2. Work is defined for an interval or displacement. There is no term like instantaneous work similar to
instantaneous velocity.

=0.2
2kg 10 N
frictionless 2kg 10 N 2m
2m
Work done by 10 N force in both the cases are same = 20 N
3. For a particular displacement, work done by a force is independent of type of motion i.e. whether it
moves with constant velocity, constant acceleration or retardation etc.
4. If a body is in dynamic equilibrium under the action of certain forces, then total work done on the
body is zero but work done by individual forces may not be zero.
5. When several forces act, work done by a force for a particular displacement is independent of
other forces.
6. A force is independent of reference frame. Its displacement depends on frame so work done by a
force is frame dependent therefore work done by a force can be different in different reference
frame.
________________________________________________________________________________________
2. UNITS OF WORK :
In cgs system, the unit of work is erg.
One erg of work is said to be done when a force of one dyne displaces a body through one
centimetre in its own direction.
 1 erg = 1 dyne × 1 cm = 1 g cm s–2 × 1 cm = 1 g cm2 s–2
Note : Another name for joule is newton metre.

Relation between joule and erg


1 joule = 1 newton × 1 metre
1 joule = 105 dyne × 102 cm = 107 dyne cm
1 joule = 107 erg
1 erg = 10–7 joule
Dimensions of Work :
[Work] = [Force] [Distance] = [MLT–2] [L] = [ML2T–2]
Work has one dimension in mass, two dimensions in length and ‘–2’ dimensions in time, On the basis
of dimensional formula, the unit of work is kg m2 s–2.
Note that 1 kg m2 s–2 = (1 kg m s–2) m = 1 N m = 1 J.
3. WORK DONE BY MULTIPLE FORCES :
  
If several forces act on a particle, then we can replace F in equation W = F . S by the net force

 F where
   
 F  F1  F2  F3 ......
 
 W=   F . S ...(i)

This gives the work done by the net force during a displacement S of the particle.
We can rewrite equation (i) as :
     
W = F1. S  F2 . S  F3 . S.....
or W = W1 + W2 + W3 + .........
So, the work done on the particle is the sum of the individual work done by all the forces acting on
the particle.

Corporate Head Office : Motion Education Pvt. Ltd., 394 - Rajeev Gandhi Nagar, Kota-5 (Raj.)
WORK, POWER & ENERGY Page # 135

Ex.1 A block of mass M is pulled along a horizontal surface by applying a force at an angle  with
horizontal. Coefficient of friction between block and surface is . If the block travels with
uniform velocity, find the work done by this applied force during a displacement d of the
block.
Sol. The forces acting on the block are shown in Figure. As the block moves with uniform velocity the
resultant force on it is zero.
 F cos  = N ...(i)
F sin  + N = Mg ...(ii)
Eliminating N from equations (i) and (ii), N
F cos  = (Mg – Fsin ) F
Mg
F M
cos    sin 
Work done by this force during a displacement d
Mgdcos  Mg
W = F . d cos  =
cos    sin 


Ex.2 A particle moving in the xy plane undergoes a displacement S  (2.0 ˆi  3.0ˆj)m while a constant

force F  (5.0 ˆi  2.0ˆj)N acts on the particle.
(a) Calculate the magnitude of the displacement and that of the force.
(b) Calculate the work done by the force.
 
Sol. (a) s  ( 2.0 i  3.0 j ) F  (5.0 i  2.0 j)

| s |  x2  y 2  ( 2.0) 2  (3.0)2  13 m

|F| Fx2  Fy2  ( 5 . 0 ) 2  ( 2 .0 ) 2  5 . 4N
 
(b) Work done by force, W  F . s

 (5.0 î  2.0 ĵ) . ( 2.0 î  3.0 ĵ) N. m = 10 + 0 + 0 + 6 = 16 N.m = 16 J

Ex.3 A block of mass m is placed on an inclined plane which is moving with constant velocity v in
horizontal direction as shown in figure. Then find out work done by the friction in time t if the
block is at rest with respect to the incline plane.

v=const.
m

Sol. F.B.D of block with respect to ground.

f v

f m
v

 m
g
mg sin 

B
A
mg mgsin
Block is at rest with respect to wedge
 f = mg sin 

: 0744-2209671, 08003899588 | url : www.motioniitjee.com, : info@motioniitjee.com


Page # 136 WORK, POWER & ENERGY

In time t the displacement of block with respect to ground d = vt


Work done by friction for man A
Wf = (component of friction force along displacement) × displacement
Wf = mgsin.vt cos(180°–)
Wf = – mg vt cos sin 
Wf for man B = 0 (displacement is zero with respect to man B)

4. WORK DONE BY A VARIABLE FORCE :

(A) When F as a function of x, y, z


When the magnitude and direction of a force vary in three dimensions, it can be expressed as a
function of the position. For a variable force work is calculated for infinitely small displacement and
for this displacement force is assumed to be constant.
 
dW = F. ds
The total work done will be sum of infinitely small work
B B
   
WA  B =  F. ds =  (F cos )ds
A A

It terms of rectangular components,



F  Fx i  Fy j  Fz k

ds  dx i  dy j  dzk
xB yB zB

WA  B =  Fx dx +  Fy dy +  F dz
z
xA yA zA

Ex.4 A force F = (4.0 x i + 3.0 y j ) N acts on a particle which moves in the x-direction from the
or ig in to
x = 5.0 m. Find the work done on the object by the force.
Sol. Here the work done is only due to x component of force because displacement is along x-axis.
x2 5
5
i.e., W = 
x1
Fx dx =
 4x dx = 2x  2
0
= 50 J
0

Ex.5 A force F = 0.5x + 10 acts on a particle. Here F is in newton and x is in metre. Calculate the
work done by the force during the displacement of the particle from x = 0 to x = 2 metre.

Sol. Small amount of work done dW in giving a small displacement dx is given by

dW  F . dx
or dW = Fdx cos 0°
F
or dW = Fdx [ cos 0° = 1]
x 2 x 2
10
Total work done, W = +ve
 Fdx   (0.5x  10)dx
x2 x 2
x 0 x 0
x 2 0 2 x
x2 x 2
=  0.5xdx   10dx = 0.5  10 x x 0
2
x0 x 0 x 0

0.5 2
= [2 – 02] + 10[2 – 0] = (1 + 20) = 21 J
2

Corporate Head Office : Motion Education Pvt. Ltd., 394 - Rajeev Gandhi Nagar, Kota-5 (Raj.)
WORK, POWER & ENERGY Page # 137

(B) When F is given as a function of Time(t) :

Ex.6 The force F = 2t2 is applied on the 2 kg block. Then find out the work done by this force in
2sec. Initially at time t = 0, block is at rest.
at t = 0, v = 0
Sol. 2kg F=2t2

F = ma
 2t2 = 2a  a =t2
v t
dv dv  t 2 dt (At t = 0 it is at rest)

dt
 t2   
0 0
t3
 v=
3
Let the displacement of the block be dx from t = t to t = t +dt then, work done by the force F in
this time interval dt is.
dw = F.dx = 2t2.dx

2 dx
dw = 2t . . dt  dw = 2t 2 ( v)dt
dt
w 2 2 2
t3 2 5 2  t6  64
 dw  2t 2 .
 dt  W = 3 t dt   W= 36 = Joule
3   9
0 0 0 0

5. AREA UNDER FORCE DISPLACEMENT CURVE :

Graphically area under the force-displacement is the work done


+ve work +ve work +ve work
Fx Fy Fz

y
x –ve work z
The work done can be positive or negative as per the area above the x-axis or below the x-axis
respectively.
Ex.7 Force acting on a particle varies with x as shown in figure. Calculate the work done by the
force as the particle moves from x = 0 to x = 6.0 m.
Sol. The work done by the force is equal to the area under the
curve from
x = 0 to x = 6.0 m.
Fx(N)
This area is equal to the area of the rectangular section from
x = 0 to x = 4.0 m plus the area of the triangular section from 5
x = 4.0 m to
x = 6.0 m. The area of the rectangle is (4.0) (5.0) N.m = 20
1
J, and the area of the triangle is (2.0), (5.0) N.m = 5.0 J. 0 1 2 3 4 5 6 x(m)
2
Therefore, the total work done is 25 J.

6. INTERNAL WORK :
Suppose that a man sets himself in motion backward by pushing against a wall. The forces acting
on the man are his weight 'W' the upward force N exerted by the ground and the horizontal force
N exerted by the wall. The works of 'W' and of N are zero because they are perpendicular to the
motion. The force N' is the unbalanced horizontal force that imparts to the system a horizontal
acceleration. The work of N', however, is zero because there is no motion of its point of application.
We are therefore confronted with a curious situation in which a force is responsible for acceleration,

: 0744-2209671, 08003899588 | url : www.motioniitjee.com, : info@motioniitjee.com


Page # 138 WORK, POWER & ENERGY

but its work, being zero, is not equal to the increase in kinetic energy of the system.
N'

The new feature in this situation is that the man is a composite system with several parts that can
move in relation to each other and thus can do work on each other, even in the absence of any
interaction with externally applied forces. Such work is called internal work. Although internal
forces play no role in acceleration of the composite system, their points of application can move so
that work is done; thus the man's kinetic energy can change even though the external forces do no
work.
"Basic concept of work lies in following lines
Draw the force at proper point where it acts that give proper importance of the point of application
of force.
Think independently for displacement of point of application of force, Instead of relation the
displacement of applicant point with force relate it with the observer or reference frame in which
work is calculated.
 displacement vector of po int of 
 
W = (Force vector )   application of force as seen by 
 
 observer 

7. CONSERVATIVE FORCE :
A force is said to be conservative if work done by or against the force in moving a body depends
only on the initial and final positions of the body and does not depend on the nature of path followed
between the initial and final positions.

m m m

m m m

(a) (b) (c)

Consider a body of mass m being raised to a height h vertically upwards as shown in above figure.
The work done is mgh. Suppose we take the body along the path as in (b). The work done during
horizontal motion is zero. Adding up the works done in the two vertical parts of the paths, we get
the result mgh once again. Any arbitrary path like the one shown in (c) can be broken into
elementary horizontal and vertical portions. Work done along the horizontal path is zero. The work
done along the vertical parts add up to mgh. Thus we conclude that the work done in raising a
body against gravity is independent of the path taken. It only depends upon the intial and final
positions of the body. We conclude from this discussion that the force of gravity is a conservative
force.

Examples of Conservative forces.


(i) Gravitational force, not only due to Earth due in its general form as given by the universal law of
gravitation, is a conservative force.
(ii) Elastic force in a stretched or compressed spring is a conservative force.

Corporate Head Office : Motion Education Pvt. Ltd., 394 - Rajeev Gandhi Nagar, Kota-5 (Raj.)
WORK, POWER & ENERGY Page # 139

(iii) Electrostatic force between two electric charges is a conservative force.


(iv) Magnetic force between two magnetic poles is a conservative force.
Forces acting along the line joining the centres of two bodies are called central forces. Gravitational
force and Electrosatic forces are two important examples of central forces. Central forces are
conservative forces.

Properties of Conservative forces


• Work done by or against a conservative force depends only on the initial and final position
of the body.
• Work done by or against a conservative force does not depend upon the nature of the path
between initial and final position of the body.
• Work done by or against a conservative force in a round trip is zero.
If a body moves under the action of a force that does no total work during any round trip, then the
force is conservative; otherwise it is non-conservative.
The concept of potential energy exists only in the case of conservative forces.
• The work done by a conservative force is completely recoverable.
Complete recoverability is an important aspect of the work done by a conservative force.

Work done by conservative forces


Ist format : (When constant force is given)


Ex.8 Calculate the work done to displace the particle from (1, 2) to (4, 5). if F  4 ˆi  3ˆj
  
Sol. dw = F.d r ( dr  dxi  dyj  dzk )

dw = ( 4 i  3 j ).( dxi  dyj)  dw = 4dx + 3dy


w 4 5

 dw =  4dx +  3dy 4 5
 w = 4 x1  3 y2
0 1 2

w = (16 – 4) + (15 – 6)  w = 12 + 9 = 21 Joule

II format : (When F is given as a function of x, y, z)



If F  Fx i  Fy j  Fz k
then

dw = (Fx i  Fy j  Fzk ).(dx i  dyj  dzk )  dw = Fxdx + Fydy + FZdz

 
Ex.9 An object is displaced from position vector r1  (2 ˆi  3ˆj)m to r2  (4 ˆi  6ˆj)m under a force

F  (3x 2 ˆi  2yˆj)N . Find the work done by this force.

  r2
rf r2
 2 3
 y 2 ](( 42,,63 ))  83 J Ans.
Sol. W  F.dr  (3 x 2 î  2yĵ)  (dx î  dyˆj  dzk̂ ) =
 
 (3 x dx  2ydy)  [x

r1
ri r1

IIIrd format

(perfect differential format)
Ex.10 If F  ˆ  xjˆ then find out the work done in moving the particle from position (2, 3) to (5, 6)
yi
 
Sol. dw = F. ds

: 0744-2209671, 08003899588 | url : www.motioniitjee.com, : info@motioniitjee.com


Page # 140 WORK, POWER & ENERGY

dw = ( y i  xj).(dxi  dyj )
dw = ydx + xdy
Now ydx + xdy = d(xy) (perfect differential equation)
 dw = d(xy)
for total work done we integrate both side

 dw   d( xy)
Put xy = k
then at (2, 3) ki = 2 × 3 = 6
at (5, 6) kf = 5 × 6 = 30
30
30
then w=  dk  k  6  w = (30 – 6) = 24 Joule
6

8. NON-CONSERVATIVE FORCES :
A force is said to be non-conservative if work done by or against the force in moving a body
depends upon the path between the initial and final positions.
The frictional forces are non-conservative forces. This is because the work done against friction
depends on the length of the path along which a body is moved. It does not depend only on the
initial and final positions. Note that the work done by fricitional force in a round trip is not zero.
The velocity-dependent forces such as air resistance, viscous force, magnetic force etc., are non
conservative forces.

Ex.11 Calculate the work done by the force F  y i to move the particle from (0, 0) to (1, 1) in the
following condition
(a) y = x (b) y = x2
Sol. We know that
 
dw = F.ds  dw = ( y i ) .(dx i )
dw = ydx ...(1)
In equation (1) we can calculate work done only when we know the path taken by the particle.
either
y = x or y = x2 so now
(a) when y = x
1 1

dw  xdx
0
  w = Joule
2
(b) when y = x2
1
2 1
 dw   x dx  w=
3
Joule
0

Difference between conservative and Non-conservative forces

Corporate Head Office : Motion Education Pvt. Ltd., 394 - Rajeev Gandhi Nagar, Kota-5 (Raj.)
WORK, POWER & ENERGY Page # 141

S. Non-Conse rva tive


Conse rva tive force s
No. force s
W ork done does not W ork done depends on
1
depend upon path path.
W ork done in a round W ork done in a round trip
2
trip is z ero. is not z ero.
Forc es are velocity -
3 Central in nature. dependent and retarding
in nature.
W hen only a
c ons ervative forc e ac ts
within a s y s tem , the
W ork done agains t a non-
k inetic energy and
c ons ervative forc e m ay
4 potential energy can
be dis ipiated as heat
c hange. However their
energy.
s um , the m ec hanic al
energy of the s y s tem ,
does not c hange.
W ork done is W ork done in not
5
c om pletely rec overable. c om pletely rec overable.

9. ENERGY
A body is said to possess energy if it has the capacity to do work. When a body possessing energy
does some work, part of its energy is used up. Conversely if some work is done upon an object, the
object will be given some energy. Energy and work are mutually convertiable.
There are various forms of energy. Heat, electricity, light, sound and chemical energy are all
familiar forms. In studying mechanics, we are however concerned chiefly with mechanical energy.
This type of energy is a property of movement or position.
9.1 Kinetic Energy
Kinetic energy (K.E.), is the capacity of a body to do work by virtue of its motion.
If a body of mass m has velocity v its kinetic energy is equivalent to the work, which an external
force would have to do to bring the body from rest up to its velocity v.
The numerical value of the kinetic energy can be calculated from the formula
1 2
K.E. = mv ...(8)
2
• Since both m and v are always positive, K.E. is always positive and does not depend upon the
2

direction of motion of the body.


9.2 Potential Energy
Potential energy is energy of the body by virtue of its position. A body is capable to do work by
virtue of its position, configuration or state of strain.
Now relation between Potential energy and work done is
W.D = – U
where U is change in potential energy
There are two common forms of potential energy, gravitational and elastic.
Important points related to Potential energy :
1. Potential energy is a straight function (defined only for position)
2. Potential energy of a point depends on a reference point
3. Potential energy difference between two position doesn't depend on the frame of reference.
4. Potential energy is defined only for conservative force because work done by conservative force is
path independent.
5. If we define Potential energy for non conservative force then we have to define P.E. of a single
point through different path which gives different value of P.E. at single point that doesn't make
any sense.

: 0744-2209671, 08003899588 | url : www.motioniitjee.com, : info@motioniitjee.com


Page # 142 WORK, POWER & ENERGY

9.2.1 (a) Gravitational Potential Energy :


It is possessed by virtue of height.
When an object is allowed to fall from one level to a lower level it gains speed due to gravitational
pull, i.e., it gains kinetic energy. Therefore, in possessing height, a body has the ability to convert
its gravitational potential energy into kinetic energy.
The gravitational potential energy is equivalent to the negative of the amount of work done by the
weight of the body in causing the descent.
If a mass m is at a height h above a lower level the P.E. possessed by the mass is (mg) (h).
Since h is the height of an object above a specified level, an object below the specified level has
negative potential energy.
Therefore GPE = ± mgh ...(9)

mg
h1

mg Specific level where


h P.E.=mgh P.E. is zero
h2 P.E. of m1 is m1gh1
P.E of m2 is –m2gh2

fig(a) fig(b)
m2g
• The chosen level from which height is measured has no absolute position. It is important therefore
to indicate clearly the zero P.E. level in any problem in which P.E. is to be calculated.
• GPE = ± mgh is applicable only when h is very small in comparison to the radius of the earth. We
have discussed GPE in detail in 'GRAVITATION'.
9.2.2 (b) Elastic Potential Energy : It is a property of stretched or compressed springs.
The end of a stretched elastic spring will begin to move if it is released. The spring. therefore
possesses potential energy due to its elasticity. (i.e., due to change in its configuration)
The amount of elastic potential energy stored in a spring of natural length a and spring constant k
when it is extended by a length x (from the natural length) is equivalent to the amount of work
necessary to produce the extension.
1 2
Elastic Potential Energy = kx ...(10)
2
It is never negative whether the spring is extended or compressed.
Proof :
N.L. N.L.

K K
M M

x0
Consider a spring block system as shown in the figure and let us calculate work done by spring
when the block is displaceed by x0 from the natural length.
At any moment if the elongation in spring is x, then the force on the block by the spring is kx
towards left. Therefore, the work done by the spring when block further displaces by dx
dW = – kx dx
x0
1 2
 Total work done by the spring, W = –  kx dx = –
0
2
kx 0

1 2
Similarly, work done by the spring when it is given a compression x0 is – kx 0 .
2
: We assume zero potential energy at natural length of the spring :

Corporate Head Office : Motion Education Pvt. Ltd., 394 - Rajeev Gandhi Nagar, Kota-5 (Raj.)
WORK, POWER & ENERGY Page # 143

10. CONSERVATIVE FORCE AND POTENTIAL ENERGY :

U
Fs = –
s
i.e. the projection of the force field , the vector F, at a given point in the direction of the
displacement r equals the derivative of the potential energy U with respect to a given direction,
taken with the opposite sign. The designation of a partial derivative /s emphasizes the fact of
deriving with respect to a definite direction.
So, having reversed the sign of the partial derivatives of the function U with respect to x, y, z, we
obtain the projection Fx, Fy and Fz of the vector F on the unit vectors i, j and k. Hence, one can
readily find the vector itself :
 U U U 
F = Fxi + Fy j + Fzk, or F = –   x i   y j  z k 
 
The quantity in parentheses is referred to as the scalar gradient of the function U and is denoted
by grad U or  U. We shall use the second, more convenient, designation where  (“nabla”)
signifies the symbolic vector or operator
  
 = i x  j y  k z
Potential Energy curve :
• A graph plotted between the PE of a particle and its displacement from the centre of force field is
called PE curve.
• Using graph, we can predict the rate of motion of a particle at various positions.
dU
• Force on the particle is F(x) = –
dx
Q

S
U B C D
A P2
P1
R
P
O x
Case : I On increasing x, if U increases, force is in (–) ve x direction i.e. attraction force.
Case : II On increasing x, if U decreases, force is in (+) ve x-direction i.e. repulsion force.

Different positions of a particle :


Position of equilibrium
dU
If net force acting on a body is zero, it is said to be in equilibrium. For equilibrium = 0. Points P,,
dx
Q, R and S are the states of equilbrium positions.
Types of equilirbium :
• Stable equilibrium :
When a particle is displaced slightly from a position and a force acting on it brings it back to the
initial position, it is said to be in stable equilibrium position.
dU d2U
Necessary conditions:– = 0,and 2 =+ ve
dx dx
In figure P and R point shows stable equilibrium point.

: 0744-2209671, 08003899588 | url : www.motioniitjee.com, : info@motioniitjee.com


Page # 144 WORK, POWER & ENERGY

• Unstable Equilibrium :
When a particle is displaced slightly from a position and force acting on it tries to displace the
particle further away from the equilibrium position, it is said to be in unstable equilibrium.
dU d2U
Condition :  0 potential energy is maximum i.e. = = – ve
dx dx2
Q point in figure shows unstable equilibrium point
• Neutral equilibrium :
In the neutral equilibrium potential energy is constant. When a particle is displaced from its position
it does not experience any force acting on it and continues to be in equilibrium in the displaced
position. This is said to be neutral equilibrium.
In figure S is the neutral point
dU d2U
Condition : 0 , =0
dx dx2
a b
Ex.12 The potential energy between two atoms in a molecule is given by, U(x) = – , where a
x 12 x 6
and b are positive constants and x is the distance between the atoms. The system is in
stable equilibrium when -
1/6
a  2a   11a 
(A) x = 0 (B) x = (C) x =   (D) x =  5b 
2b  b 
Sol. (C)
a b
Given that, U(x) = 12

x x6
du
We, know F=–
dx
= (–12) a x–13 – (–6 b) x–7= 0
1/ 6
6b 12a  2a 
or  or x6 = 12a/6b = 2a/b or x=  
x7 4 x 13  b

Ex.13 The potential energy of a conservative system is given by U = ax2 – bx where a and b are
positive constants. Find the equilibrium position and discuss whether the equilibrium is
stable, unstable or neutral.
dU d
Sol. In a conservative field F = – F=– (ax2 – bx) = b – 2ax
dx dx
b
For equilibrium F = 0 or b – 2ax = 0  x =
2a

d2U
From the given equation we can see that  2a (positive), i.e., U is minimum.
dx2
b
Therefore, x = is the stable equilibrium positon.
2a

11. WORK ENERGY THEOREM :

If the resultant or net force acting on a body is Fnet then Newton's second law states that
Fnet = ma ...(1)
If the resultant force varies with x, the acceleration and speed also depend on x.

Corporate Head Office : Motion Education Pvt. Ltd., 394 - Rajeev Gandhi Nagar, Kota-5 (Raj.)
WORK, POWER & ENERGY Page # 145

dv
then a= v ...(2)
dx
from eq. (1)
Fnet
dv m m
Fnet = mv  Fnet.dx = m v dv
dx
vi vf
vf

Fnet . dx =  mvdv
vi

1 1
Wnet = mv 2f – mvi2
2 2
Wnet = kf – ki
Wnet = K ...(3)

Work done by net force Fnet in displacing a particle equals to the change in kinetic energy of the
particle i.e.
we can write eq. (3) in following way
(W.D)c + (W.D)N.C + (W.D)ext. + (W.D)pseudo
= K ...(4)
where (W.D)c = work done by conservative force
(W.D)N.C = work done by non conservative force.
(W.D)ext = work done by external force
(W.D)pseudo = work done by pseudo force.
we know that
(W.D)c = – U
 – U + (W.D)N.C + (W.D)ext + (W.D)pseudo = K
 (W.D)N.C + (W.D)ext. + (W.D)pseudo = (kf + uf) – (ki + ui)
 k + u = Mechanical energy.
 work done by forces (except conservative forces)
= change is mechanical energy.
If (W.D)N.C = (W.D)ext = (W.D)pseudo = 0
Kf + Uf = Ki + Ui
Initial mechanical energy = final mechanical energy
This is called mechanical energy conservation law.

Questions Based on work Energy Theorem :

(A) When only one conservative force is acting

Ex.14 The block shown in figure is released from rest. Find out the speed of the block when the
spring is compressed by 1 m.
N.L.

2kg
A
2m

: 0744-2209671, 08003899588 | url : www.motioniitjee.com, : info@motioniitjee.com


Page # 146 WORK, POWER & ENERGY

Sol. In the above problem only one conservative force (spring force) is working on the block so from
mechanical energy conservation
kf + uf = ki + ui ...(i)
at A block is at rest so ki = 0
N.L.
1 1
ui  kx12  k( 2) 2 = 2k Joule
2 2 B A
At position B if speed of the block is v then 2kg 2kg

1 1 x2=1m x1=2m
kf  mv 2   2  v 2  v 2
2 2

1 2 1 k
uf  kx 2   k  1 
2 2 2
Putting the above values in equation (i), we get
k 3k 3k
 v2   2k 2
 v   v m / sec
2 2 2

Ex.15 A block of mass m is dropped from height h above the ground. Find out the speed of the
block when it reaches the ground.
Sol.

Initial situation
Ugi  mgh , ki = 0

Final situation 1
Ug f  0 , K f  mv2
v 2

Figure shows the complete description of the problem only one conservative force is working on the
block. So from mechanical energy conservation

1
kf + uf = ki + ui  mv 2  0  0  mgh
2

v  2gh m / sec

(B) When two conservative force are acting in problem.

Ex.16 One end of a light spring of natural length d and spring constant k is fixed on a rigid wall and
the other is attached to a smooth ring of mass m which can slide without friction on a vertical
rod fixed at a distance d from the wall. Initially the spring makes an angle of 37º with the
horizontal as shown in fig. When the system is released from rest, find the speed of the ring
when the spring becomes horizontal.
[sin 37º = 3/5]

Corporate Head Office : Motion Education Pvt. Ltd., 394 - Rajeev Gandhi Nagar, Kota-5 (Raj.)
WORK, POWER & ENERGY Page # 147

A Ring
v=0

I h

37°
d v
B

Rod
Sol. If l is the stretched length of the spring, then from figure
d 4 5
 cos 37º  , i.e., l d
l 5 4
5 d
So, the stretch y=ld dd
4 4
5 3 3
and d  d
h = l sin 37º 
4 5 4
Now, taking point B as reference level and applying law of conservation of mechanical energy
between A and B,
EA = EB
1 2 1
or mgh + ky  mv 2
2 2
[as for, B, h = 0 and y = 0]
2
3 1  d 1 3 1
or mgd  k    mv 2 [as for A, h  d and y  d ]
4 2  4 2 4 4
3g k
or vd  Ans.
2d 16m

Ex.17 The block shown in figure is released from rest and initially the spring is at its natural length.
Write down the energy conservation equation. When the spring is compressed
by l1 ?
m
B

Sol. Here two conservative forces are included in the problem.


(i) Gravitational force (ii) spring force initial position
We assume zero gravitational potential energy
m Ug  mg(  1) sin 
N.L.
(  1 ) sin 

Us=0, K = 0
at A as shown in figure.
1

from mechanical energy conservation final position


m 1
kf + uf = ki + ui ...(i) Ug  0, Us  k12
2
1
K  mv2
1 1 2
mv 2  k  21  mg(  1   ) sin 
2 2

: 0744-2209671, 08003899588 | url : www.motioniitjee.com, : info@motioniitjee.com


Page # 148 WORK, POWER & ENERGY

(C) When only one non conservative force is included in problem.


Ex.18 Find out the distance travelled by the block as shown in figure. If the initial speed of the
block is v and  is the friction coefficient between the surface of block and ground.

m v

Sol. Applying work energy theorem, we get v v=0


1 2  m m
 (–mg ) = (0 + 0) –  mv  0 mg
2
Initial final
1 2 v2
 g  = v  
2 2g

(D) When both conservative and non-conservative force in the problem

Ex.19 A particle slides along a track with elevated ends and a flat central part as shown in figure.
The flat portion BC has a length l = 3.0 m. The curved portions of the track are frictionless.
For the flat part the coefficient of kinetic friction is k = 0.20, the particle is released at point
A which is at height h = 1.5 m above the flat part of the track. Where does the particle finally
comes to rest?
Sol. As initial mechanical energy of the particle is mgh and final is zero, so loss in mechanical energy
= mgh. This mechanical energy is lost in doing work against friction in the flat part,
So, loss in mechanical energy = work done against friction

h 15
.
or mgh =  mgs i.e., s= = = 7.5 m
 0.2
After starting from B the particle will reach C and then will rise
up till the remaining KE at C is converted into potential energy.
It will then again descend and at C will have the same value as A D
it had when ascending, but now it will move from C to B. The
same will be repeated and finally the particle will come to rest h
at E such that
BC + CB + BE = 7.5 B E C
or 3 + 3 + BE = 7.5
i.e., BE = 1.5
So, the particle comes to rest at the centre of the flat part.
Ex.20 A 0.5 kg block slides from the point A on a horizontal track with an initial speed 3 m/s towards
a weightless horizontal spring of length 1 m and force constant 2 N/m. The part AB of the
track is frictionless and the part BC has the coefficient of static and kinetic friction as 0.22
and 0.20 respectively. If the distance AB and BD are 2 m and 2.14 m respectively, find the
total distance through which the block moves before it comes to rest completely. [g = 10
m/s2]
Sol. As the track AB is frictionless, the block moves this distance without loss in its initial
1 1
KE = mv 2 = × 0.5 × 32 = 2.25 J. In the path BD as friction is present, so work done against
2 2
friction
= k mgs = 0.2 × 0.5 × 10 × 2.14 = 2.14 J
So, at D the KE of the block is = 2.25 – 2.14 = 0.11 J.
Now, if the spring is compressed by x

Corporate Head Office : Motion Education Pvt. Ltd., 394 - Rajeev Gandhi Nagar, Kota-5 (Raj.)
WORK, POWER & ENERGY Page # 149

1
0.11 = × k × x2 + k mgx
2

1
i.e., 0.11 = × 2 × x2 + 0.2 × 0.5 × 10x
2 A B D C
or x2 + x – 0.11 = 0
which on solving gives positive value of x = 0.1 m
After moving the distance x = 0.1 m the block comes to rest. Now the compressed spring exerts a
force :
F = kx = 2 × 0.1 = 0.2 N
on the block while limiting frictional force between block and track is fL = s mg = 0.22 × 0.5 × 10 =
1.1 N.
Since, F < fL. The block will not move back. So, the total distance moved by block
= AB + BD + 0.1
= 2 + 2.14 + 0.1
= 4.24 m

(E) Important Examples :


Ex.21 A smooth sphere of radius R is made to translate in a straight line with a constant acceleration
a. A particle kept on the top of the sphere is released from there at zero velocity with respect
to the sphere. Find the speed of the paritcle with respect to the sphere as a function of the
angle  it slides.
Sol. We solve the above problem with respect to the sphere. So apply a pseudo force on the particle
m ma
R
a
with respect to sphere
Now from work energy theorem.
work done by ma = change in mechanical energy
 ma R sin  = (kf + uf) – (ki + ui)
1 1
maR sin  = mv 2  mgR (1  cos )  mv 2 = maR sin  + mgR (1 – cos )
2 2
 v2 = 2R(a sin  + g – g cos )  v = [2R (a sin  + g – g cos )]1/2 m/sec

Ex.22 In the arrangement shown in figure mA = 4.0 kg and


mB = 4.0 kg. The system is released from rest and A
block B is found to have a speed 0.3 m/s after it
has descended through a distance of 1m. Find the
coefficient of friction between the block and the
table. Neglect friction elsewhere.
(Take g = 10 m/s2) B
Sol. From constraint relations, we can see that
v A = 2 vB
Therefore, vA = 2(0.3) = 0.6 m/s
as vB = 0.3 m/s (given)
Applying Wnc = U + K

: 0744-2209671, 08003899588 | url : www.motioniitjee.com, : info@motioniitjee.com


Page # 150 WORK, POWER & ENERGY

1 1
we get –  mA g SA = – mB g SB + mAvA2 + mBvB2
2 2
Here, SA = 2SB = 2m as SB = 1 m (given)

1 1
 – (4.0) (10) (2) = – (1) (10) (1) + (4) (0.6)2 + (1) (0.3)2
2 2
or – 80  = – 10 + 0.72 + 0.045 or 80 = 9.235 or  = 0.115 Ans.

Ex.23 A body of mass ‘m’ was slowly hauled up the hill as shown
in the figure by a force F which at each point was directed
along a tangent to the trajectory. Find the work performed
by this force, if the height of the hill is h, the length of its m F h
base is l and the coefficient of
friction is .
Sol. Four forces are acting on the body : l
1. weight (mg) 2. normal reaction (N)
3. friction (f) and 4. the applied force (F)
Using work-energy theorem
Wnet = KE
or Wmg + WN + Wf + WF = 0
Here, KE = 0, because Ki = 0 = Kf B
Wmg = – mgh  WN = 0 ds
(as normal reaction is perpendicular to displacement at all points) F
Wf can be calculated as under : A
f =  mg cos  dl
 (dWAB)f = – f ds
= – ( mg cos ) ds = –  mg (dl) (as ds cos  = dl)
 f = –  mg  dl = –  mgl
Substituting these values in Eq. (i), we get
WF = mgh + mgl

: Here again, if we want to solve this problem without using work-energy theorem we will first find
  
magnitude of applied force F at different locations and then integrate dW ( = F. dr ) with proper
limits.
12. POWER
Power is defined as the time rate of doing work.
When the time taken to complete a given amount of work is important, we measure the power of
the agent doing work.
The average power (P or Pav ) delivered by an agent is given by
WTotal work done
P or Pav =  t = Total time
where W is the amount of work done in time t.
Power is the ratio of two scalars-work and time. So, power is a scalar quantity. If time taken to
complete a given amount of work is more, then power is less.

Corporate Head Office : Motion Education Pvt. Ltd., 394 - Rajeev Gandhi Nagar, Kota-5 (Raj.)
WORK, POWER & ENERGY Page # 151

dW 
• The instantaneous power is, P = where dW is the work done by a force F in a small time dt.
dt
dW  dr   
• P= = .F  F. v where v is the velocity of the body..
dt dt
By definition of dot product,
P = Fvcos

where  is the smaller angle between F and v
This P is called as instantaneous power if dt is very small.

12.1 Unit of Power :


A unit power is the power of an agent which does unit work in unit time.
The power of an agent is said to be one watt if it does one joule of work in one second.
1 watt = 1 joule/second = 107 erg/second
1 newton  1 metre
Also, 1 watt = = 1 N ms–1.
1 sec ond
Dimensional formula of power
[ Work ] [ML2 T –2 ]
[Power ]  = = [ML2T–3]
[ Time] [T]

Ex.24 A one kilowatt motor pumps out water from a well 10 metre deep. Calculate the quantity of
water pumped out per second.
Sol. Power, P = 1 kilowatt = 103 watt

mg  S
S = 10 m ; Time, t = 1 second ; Mass of water, m = ? Power =
t
m  9.8  10
 103 =
1
10 3
or m= kg = 10.204 kg
9.8  10

Ex.25 The blades of a windmill sweep out a circle of area A. (a) If the wind flows at a velocity v
perpendicular to the circle, what is the mass of the air passing through in time t? (b) What is
the kinetic energy or the air? (c) Assume that the windmill converts 25% of the wind’s energy
into electrical energy, and that A = 30m2, v = 36 km h–1 and the density of air is 1.2 kg m–3.
What is the electrical power produced?
Sol. (a) Volume of wind flowing per second = Av
Mass of wind flowing per second = Av
Mass of air passing in t second = Avt
1 1 1
(b) Kinetic energy of air = mv 2 = ( Avt)v 2 = Av 3t
2 2 2
25 1 3 Av 3 t
(c) Electrical energy produced =  Av t =
100 2 8
Av 3 t Av 3 
Electrical power = =
8t 8
5
Now, A = 30 m2, v = 36 km h–1 = 36 × m s–1 = 10 m s–1,  = 1.2 kg ms–1
18

: 0744-2209671, 08003899588 | url : www.motioniitjee.com, : info@motioniitjee.com


Page # 152 WORK, POWER & ENERGY

30  10  10  12
.
 Electrical power = W = 4500 W = 4.5 kW
8

Ex.26 One coolie takes one minute to raise a box through a height of 2 metre. Another one takes
30 second for the same job and does the same amount of work. Which one of the two has
greater power and which one uses greater energy?

Work M  g  S M  9.8  2 –1
Sol. Power of first coolie =  = Js
Time t 60

M  9.8  2 –1  M  9.8  2 
Power of second coolie = Js = 2   J s–1 = 2 × Power of first coolie
30  60 

So, the power of the second coolie is double that of the first. Both the coolies spend the same
amount of energy.
We know that W = Pt
For the same work,
W = p1t1 = P2t2

P2 t1 1minute
or  2
P1 t 2 = 30 s or P2 = 2P1

13. VERTICAL CIRCULAR MOTION


C
To understand this consider the motion of a small body
(say stone) tied to a string and whirled in a vertical
circle.
Now we study the circular motion of the body in two B
parts. R
A) Motion of a body from A to B. T1 v1
B) Motion of a body from B to C. T
2
mA u mg mv1
A. Motion of a body from A to B. R
mv12
T1 = mg cos  + ...(1)
R
During the motion of the body from A to B.
 will increase so cos  will decrease.
Due to which mg cos  will decrease. From A to B speed of the body also decreases due to which

mv 2
decreases. Therefore tenstion in the string decreases from A to B.
R
But due to mg cos  tension can never be zero.

B. Motion of a body from B to C.


C v2 mv2
2

mv 22 R
T2   mg cos ...(2) T2
R
From B  C mg
B
mv 22
speed decreases due to which decreases.
R
 decreases due to which mg cos  increases.
Therefore from B C. Tension in the string decreases.
mv 22
String slacks at a point where = mg cos  i.e., T = 0
R

Corporate Head Office : Motion Education Pvt. Ltd., 394 - Rajeev Gandhi Nagar, Kota-5 (Raj.)
WORK, POWER & ENERGY Page # 153

13.1 Minimum velocity at point A for which body can complete the vertical circle
The condition for the body to complete the vertical circle is that the string should be taut all the
time i.e. the tension is greater than zero.
So the body can complete the vertical circle if the tension is not zero in between the region B to C.
Initially.
mv 2 finally
from figure (b) = TC + mg ...(1)
R mv 2
Apply energy conservation from A to C then v C R
Kf + Uf = Ki + Ui
1 1 Tc+mg
mv 2 + 2mgR = mu2  0 ...(2)
2 2 TA
body can complete vertical circle, when
TC  0 A u
mv 2 mg
– mg  0 fig(a) fig(b)
R
 v  gR
2
...(3)
Put the value from (3) to (2) and
u = umin
1 1 2
 m(Rg) + 2mgR = mumin
2 2
2
 umin = 5gR  umin  5gR
It the velocity is greater than equal to 5gR then the body will complete the vertical circle.

Tension at A

mu2
TA = mg +
R
TA
If u = umin = 5gR u
5mgR mu2
then TA = mg +  TA = 6mg mg 
R R
Tension at B
mv 2 v
TB =
R
energy conservation from A to B
TB B mv 2
R
1 1
mu2min = mgR + mv 2
2 2 mg
 v2 = 3gR  TB = 3mg
A

13.2 Condition for the body to reach B :


Let us calculate the umin such that the body just reaches
B.
Work done by tension = 0 O R v=0, Ug=mgR
Only gravitational force is working on the body which is K= 0
R
a conservative force. Therefore Applying conservation
of energy, we get A u Ug=0
min
1
1 K  mu2min
mgR = mu2min  umin = 2gR 2
2
 if u  2gR then the body will oscillate about A.

: 0744-2209671, 08003899588 | url : www.motioniitjee.com, : info@motioniitjee.com


Page # 154 WORK, POWER & ENERGY

13.3 When 2gR  u  5gR

If the velocity of projection is greater than 2gR but less than 5gR , the particle rises above the
horizontal diameter and the tension vanishes before reaching the highest point.
We have seen that the tension in the string at the highest point is lower than the tension at the
lowest point.
At the point D, the string OD makes an angle  with the vertical. The radial component of the
weight is mg cos  towards the centre O.

mv2  v2 
T + mg cos  =  T = m – g cos  
 ...(i)
R R 

1
Kinetic energy at D = mv 2 B
2 D
N
Potential energy at D = mg(AN) T
= mg (AO + ON)  mg(R + R cos) = mgR(1 + cos) O  mg
From conservation of energy
1 1
mu 2 = mv 2 + mgR(1+ cos)
2 2 A
v2 = u2 – 2gR(1 + cos )
Substituting in equation (i),
 u2 
T = m – 2g(1  cos ) – g cos 
 R 
 u2  2 
T = m – 3g cos   
 R  3 
This equation shows that the tension becomes zero. if

u2  2
 3g cos    ...(ii)
R  3
If the tension is not to become zero.
 2
u2 > 3Rg  cos   
 3
Equation (ii) gives the values of  at which the string becomes slack.

2 u2
cos + =
3 3Rg

u2 2
cos = –
3Rg 3

u2 – 2gR
cos  =
3gR
It is the angle from the vertical at which tension in the string vanishes to zero. And after that its
motion is projectile.

13.4 Tension in the string versus 


We may find an expression for the tension in the string when it makes an angle  with the vertical.
At C, the weight of the body acts vertically downwards, and the tension in the string is towards
the centre O.

Corporate Head Office : Motion Education Pvt. Ltd., 394 - Rajeev Gandhi Nagar, Kota-5 (Raj.)
WORK, POWER & ENERGY Page # 155

The weight mg is resolved radially and tangentially.


The radial component is mg cos  and the tangential component is mg sin .

mv2
T – mg cos  = , where v is the velocity at C.
R
 v2 
i.e., T = m  g cos  
 ...(i) B
R 
The velocity v can be expressed in terms of velocity u at A.
O
1  T v
2
The total energy at A = mu M C
2
A u  mg cos 
1 mg
The kinetic energy at C = mv 2
2
The potential energy at C = mg (AM)
= mg (AO – MO) = mg (R – R cos)
= mgR (1 – cos )
1
The total energy at C = mv 2  mgR(1 – cos )
2
 From conservation of energy
1 1
mu 2 = mv 2 + mgR (1 – cos)
2 2

u2 = v2 + 2gR (1 – cos ) or v2 = u2 – 2gR(1 – cos )


Substituting in equation (v),

 u2  mu 2  2
T = mg cos   – 2g(1 – cos ) =  3mg cos  –  ...(ii)
 R  R  3 
This expression gives the value of the tension in the
string in terms of the velocity at the lowest point and
the angle .
T
Equation (i) shows that tension in the string decreases 6mg
as  increases, since the term 'g cos ' decreases as 
increases. 3mg

when u = 5gR cos


–1 1
 T = 3mg (1 + cos )
Now  = 0  cos  = 1  TA = 6 mg
if,  = 90°  TB = 3 mg
 = 180°, cos = – 1
Tc = 0
13.5 Different situations :
(A) A BODY MOVING INSIDE A HOLLOW TUBE OR SPHERE
The previous discussion holds good for this case, but v2
instead of tension in the string we have the normal N'
mg
reaction of the
R
surface. If N is the normal reaction at the lowest point, then
N
the condition u  5Rg for the body to complete the u
circle holds for this case also. All other equations (can mg
be) similarly obtained by replacing tension T by normal
reaction N.

: 0744-2209671, 08003899588 | url : www.motioniitjee.com, : info@motioniitjee.com


Page # 156 WORK, POWER & ENERGY

(B) WHEN BODY IS ATTACHED TO A ROD OF LENGTH R


In this case since the body is attached to a rigid rod. The body can not leave the circular path.
Therefore, if the speed of the body becomes zero before the highest point C. It's motion will be
oscillatory about the centre of the rod.
Condition for completing the circle :
If the body just reaches the highest point then it will
completes the vertical circle
Applying energy conservation between the lowest and
highest point of circle, we get

v=0
Uf = mg2R
Kf = 0

1
Ui = 0, Ki = mu 2
u 2

1
2mgR = mu2  u = 4 gR
2

So, If the velocity at point A is greater than equal to 4gR then


body will compete the vertical circle.

(C) VERTICAL MOTION IN A DUAL RING

u
This system will behave as the preivious system. So
umin to
complete vertical circle umin = 4gR
Angle at which the normal reaction on the body will
change its direction from inward to outward the ring
is given by

u 2 – 2gR
cos =
3gR

(D) BODY MOVING ON A SPHERICAL SURFACE


The small body of mass m is placed on the top of a smooth sphere of radius R and the body slides
down the surface.
At any instant, i.e., at point C the forces are the normal reaction N and the weight mg. The radial
component of the weight is mgcos  acting towards the centre. The centripetal force is

Corporate Head Office : Motion Education Pvt. Ltd., 394 - Rajeev Gandhi Nagar, Kota-5 (Raj.)
WORK, POWER & ENERGY Page # 157

mv2 B
mg cos  – N = , m N
R C
where v is the velocity of the body at O. D

 v 2  mg
N = m  g cos  – ...(i) O
 R 
The body flies off the surface at the point where N becomes zero.

v2 v2 A
i.e., g cos  = ; cos  = ...(ii)
R Rg
To find v, we use conservation of energy
1
i.e., mv 2 = mg (BD)
2
= mg (OB – OD) = mgR (1 – cos )
v2 = 2Rg (1 – cos )

v2
2(1 – cos ) = ...(iii)
Rg
From equation (ii) and (iii) we get
cos  = 2 – 2 cos  ; 3 cos  = 2
2  2
cos  = ;  = cos–1   ...(iv)
3  3
This gives the angle at which the body goes of the surface. The height from the ground of that
point
= AD = R(1 + cos )
 2 5
=R  1   = R
 3  3

: 0744-2209671, 08003899588 | url : www.motioniitjee.com, : info@motioniitjee.com


Page # 158 CIRCULAR & WORK, POWER & ENERGY

Exercise - I OBJECTIVE PROBLEMS (JEE MAIN)


(A) CIRCULAR MOTION 4. A wheel having a diameter of 3 m starts from
1. A wheel is at rest. Its angular velocity increases rest and accelerates uniformly to an angular
uniformly and becomes 80 radian per second after 5 velocity of 210 r.p.m. in 5 seconds. Angular
second. The total angular displacement is : acceleration of the wheel is -
(A) 800 rad (B) 400 rad rad rad
(C) 200 rad (D) 100 rad (A) 1.4 2 (B) 3.3
s s2
Sol.
rad rad
(C) 2.2 2 (D) 1.1
s s2
Sol.

2. A wheel is subjected to uniform angular a


cceleration about its axis. Initially its angular
velocity is zero. In the first 2 sec, it rotates through
an angle ; in the next 2 sec, it rotates through an
additional angle 2. The ratio of 2 /1 is-
5. The second’s hand of a watch has length 6 cm.
(A) 1 (B) 2 (C) 3 (D) 5 Speed of end point and magnitude of difference of
Sol. velocities at two perpendicular positions will be :
(A) 2 & 0 mm/s (B) 2 2  & 4.44 mm/s

(C) 2 2  & 2 mm/s (D) 2 & 2 2  mm/s


Sol.

3. The linear and angular acceleration of a particle


are 10 m/sec2 and 5 rad/sec2 respectively it will
be at a distance from the axis of rotation - 6. The ratio of angular speed of hours hand and
(A) 50 m (B) 1/2 m (C) 1 m (D) 2 m seconds hand of a clock is-
Sol. (A) 1 : 1 (B) 1 : 60
(C) 1 : 720 (D) 3600 : 1
Sol.

Corporate Head Office : Motion Education Pvt. Ltd., 394 - Rajeev Gandhi Nagar, Kota-5 (Raj.)
CIRCULAR & WORK, POWER & ENERGY Page # 159

7. The angular velocity of earth about its axis of 10. A car of mass 1000 kg moves on a circular
rotation is- track of radius 20 m. if the coefficient of friction
(A) 2 / ( 60 × 60 × 24 ) rad / sec is 0.64, what is the maximum velocity with which
(B) 2 / ( 60 × 60 ) rad / sec the car can be moved?
(C) 2 / 60 rad / sec (A) 1.12 m/s (B) 11.2 m/s
(D) 2 / ( 365 × 24 × 60 × 60 ) rad / sec
Sol. 0 . 64  20 1000
(C) m/s (D) 0 . 64  20 m/s
1000
Sol.

8. A stone of mass 0.5 kg tied with a string of length 1


metre is moving in a circular path with a speed of 4 m/
sec. The tension acting on the string in Newton is- 11. The earth, radius 6400 km, makes one
(A) 2 (B) 8 (C) 0.2 (D) 0.8 revolution about its own axis in 24 hours. The
Sol. centripetal acceleration of a point on its equator
is nearly -
cm cm
(A) 340 2 (B) 3.4
sec sec 2
cm cm
(C) 34 (D) 0.34
sec 2 sec 2
Sol.

9. Two bodies A & B rotate about an axis, such that


angle A (in radians) covered by first body is
proportional to square of time, & B (in radians)
covered by second body varies linearly. At t = 0, A
= B = 0. If A completes its first revolution in 
sec. & B needs 4 sec. to complete half revolution
then; angular velocity A : B at t = 5 sec. are in 12. A stone of mass 0.1 kg tied to one end of
the ratio a string 1.0 m long is revolved in a horizontal
(A) 4 : 1 (B) 20 : 1 circle at the rate of 10/ revolution per second.
(C) 80 : 1 (D) 20 : 4 Calculate the tension of the string ?
Sol. (A) 30 N (B) 40 N
(C) 50 N (D) 60 N
Sol.

: 0744-2209671, 08003899588 | url : www.motioniitjee.com, : info@motioniitjee.com


Page # 160 CIRCULAR & WORK, POWER & ENERGY

13. A coin placed on a rotating turn table just  20 


slips if it is at a distance of 40 cm from the 16. A particle moves along a circle of radius  

centre if the angular velocity of the turntable is m with constant tangential acceleration. If the
doubled, it will just slip at a distance of velocity of the particle is 80 m/s at the end of the
(A) 10 cm (B) 20 cm second revolution after motion has begun, the
(C) 40 cm (D) 80 cm tangential acceleration is :
Sol. (A) 160  m/s2 (B) 40  m/s2
(C) 40 m/s 2
(D) 640  m/s2
Sol.

14. A particle moves in a circle of radius 25 cm


at two revolutions per second. The acceleration
of particle in m/s2 is -
(A) 2 (B) 82 (C) 42 (D) 22
Sol.
17. A particle moves with deaceleration along the
circle of radius R so that at any moment of time its
tangential and normal accelerations are equal in
moduli. At the initial moment t = 0 the speed of the
particle equals v0, then :
(i) the speed of the particle as a function of the
distance covered s will be
(A) v = v0 e–s/R (B) v = v0es/R
(C) v = v0e –R/s
(D) v = v0eR/s
(ii) the total acceleration of the particle as function
of velocity and distance covered
15. The graphs below show angular velocity as a v2 v
(A) a = 2 (B) a = 2
function of time. In which one is the magnitude of R R
the angular acceleration constantly decreasing? R 2R
(C) a = 2 (D) a =
v v
Sol.
(A) t (B) t

(C) t (D) t

Sol.

18. If angular velocity of a disc depends an angle


rotated  as  = 2 + 2, then its angular acceleration
 at  = 1 rad is
(A) 8 rad/sec2 (B) 10 rad/sec2
(C) 12 rad/sec2 (D) None

Corporate Head Office : Motion Education Pvt. Ltd., 394 - Rajeev Gandhi Nagar, Kota-5 (Raj.)
CIRCULAR & WORK, POWER & ENERGY Page # 161

Sol. 21. A particle moving along a circular path due to a


centripetal force having constant magnitude is an
example of motion with :
(A) constant speed and velocity
(B) variable speed and velocity
(C) variable speed and constant velocity
(D) constant speed and variable velocity.
Sol.

19. A particle is going in a uniform helical and spiral


path separately as shown in figure with constant
speed.

22. A curved section of a road is banked for a speed


v. If there is no friction between road and tyres of
(A) (B) the car, then :
(A) car is more likely to slip at speeds higher than v
(A) The velocity of the particle is constant in both
than speeds lower than v
cases
(B) car cannot remain in static equilibrium on the
(B) The acceleration of the particle is constant in
curved section
both cases
(C) car will not slip when moving with speed v
(C) The magnitude of acceleration is constant in
(D) none of the above
(A) and decreasing in (B)
Sol.
(D) The magnitude of acceleration is decreasing
continuously in both the cases
Sol.

23. Water in a bucket is whirled in a vertical circle


20. If the radii of circular paths of two particles of with a string attached to it. The water does not fall
same masses are in the ratio of 1 : 2, then in order down even when the bucket is inverted at the top
to have same centripetal force, their speeds should of its path. We conclude that :
be in the ratio of : mv 2 mv 2
(A) mg = (B) mg >
(A) 1 : 4 (B) 4 : 1 (C) 1 : 2 (D) 2 :1 R R
Sol. mv 2
(C) mg < (D) none of these
R
Sol.

: 0744-2209671, 08003899588 | url : www.motioniitjee.com, : info@motioniitjee.com


Page # 162 CIRCULAR & WORK, POWER & ENERGY

24. A man is standing on a rough ( = 0.5) horizontal Sol.


disc rotating with constant angular velocity of 5
rad/sec. At what distance from centre should he
stand so that he does not slip on the disc ?
(A) R  0.2 m (B) R > 0.2 m
(C) R > 0.5 m (D) R > 0.3 m
Sol.

27. A particle is moving in a circle:


(A) The resultant force on the particle must be
towards the centre.
(B) The cross product of the tangential acceleration
and the angular velocity will be zero.
(C) The direction of the angular acceleration and
the angular velocity must be the same.
(D) The resultant force may be towards the centre.
Sol.

25. A car travelling on a smooth road passes through


a curved portion of the road in form of an arc, of
circle of radius 10m. If the mass of car is 500 kg,
the reaction on car at lowest point P where its speed
is 20 m/s is

P 28. The dumbell is placed on a frictionless horizontal


table. Sphere A is attached to a frictionless pivot so
(A) 35 kN (B) 30 kN (C) 25 kN (D) 20 kN
that B can be made to rotate about A with constant
Sol.
angular velocity. If B makes one revolution in period
P, the tension in the rod is
2M
d B
A

4  2Md 8  2Md 42Md 2Md


(A) 2
(B) 2
(C) (D)
P P P P
Sol.

26. A road is banked at an angle of 30° to the


horizontal for negotiating a curve of radius 10 3 m.
At what velocity will a car experience no friction
while negotiating the curve?
(A) 54 km/hr (B) 72 km/hr
(C) 36 km/hr (D) 18 km/hr

Corporate Head Office : Motion Education Pvt. Ltd., 394 - Rajeev Gandhi Nagar, Kota-5 (Raj.)
CIRCULAR & WORK, POWER & ENERGY Page # 163

29. A string can bear a maximum tension of Sol.


100 Newton without breaking. A body of mass 1
kg is attached to one end of 1 m length of thin
string and it is revolved in a horizontal plane. The
maximum linear velocity which can be imparted to
the body without breaking the string, will be -
(A) 10 m/s (B) 1 m/s
(C) 100 m/s (D) 1000 m/s
Sol.

32. Which vector in the figures best represents the


acceleration of pendulum mass of the intermediate
point in its swing?

(A) (B)

30. A pendulum bob is swinging in a vertical plane


such that its angular amplitude is less than 90°. At
its highest point, the string is cut. Which trajectory (C) (D)
is possible for the bob afterwards.
Sol.

(A) (B)

(C) (D)

Sol.

33. Two racing cars of masses m1 and m2 are moving


in circles of radii r1 and r2 respectively. Their speeds
are such that each makes a complete circle in the
same time t. The ratio of the angular speeds of the
first to the second car is
(A) 1 : 1 (B) m1 : m2
(C) r1 : r2 (D) m1m2 : r1r2
Sol.

31. A conical pendulum is moving in a circle with


angular velocity  as shown. If tension in the string
is T, which of following equations are correct ?

(A) T = ml (B) T sin = ml


(C) T = mg cos (D) T = m2l sin

: 0744-2209671, 08003899588 | url : www.motioniitjee.com, : info@motioniitjee.com


Page # 164 CIRCULAR & WORK, POWER & ENERGY

34. A cyclist taking turn bends inwards while a car 37. A motor cyclist moving with a velocity of 72 km
passenger take the same turn is thrown outwards. per hour on a flat road takes a turn on the road at a
The reason is- point where the radius of curvature of the road is
(A) car is heavier then cycle 20 metres. The acceleration due to gravity is 10 m/
(B) car has four wheels while cycle has only two s2. In order to avoid skidding, he must not bend
with respect to the vertical plane by an angle greater
(C) difference in the speed of the two
than-
(D) Cyclist has to counteract the centrifugal force
(A) = tan–1 6 (B)  = tan–1 2
while in the case of car only the passenger is thrown (C)  = tan–1 25.92 (D)  = tan–14
by this force Sol.
Sol.

35. A cyclist is moving on a circular track of radius


80 m with a velocity of 72 km/hr. He has to lean (B) WORK, POWER AND ENERGY
from the vertical approximately through an angle 38. A rigid body of mass m is moving in a circle of
(A) tan–1 (1/4) (B) tan–1 (1) radius r with a constant speed v. The force on the
–1
(C) tan (1/2) (D) tan–1 (2) mv 2
Sol. body is and is directed towards the centre.
r
What is the work done by this force in moving the
body over half the cirumference of the circle.
mv 2 mv 2 r 2
(A) (B) Zero (C) (D)
r 2 r2 mv 2
Sol.

36. Keeping the banking angle same to increase the


maximum speed with which a vehicle can travel on a
curved road by 10%, the radius of curvature of road
has to be changed from 20 m to-
(A) 16 m (B) 18 m
(C) 24.25 m (D) 30.5 m
Sol. 39. A body travels through a distance of 10 m
on a straight line, under the influence of 5 N .
If the work done by the force is 25J, the angle
between the force and displacement is-
(A) 0º (B) 30º (C) 60º (D) 90º
Sol.

Corporate Head Office : Motion Education Pvt. Ltd., 394 - Rajeev Gandhi Nagar, Kota-5 (Raj.)
CIRCULAR & WORK, POWER & ENERGY Page # 165

40. A blocks is moved from rest through a distance 43. If a man increases his speed by 2 m/sec, his
of 4m along a straight line path. The mass of the K.E. is doubled. The original speed of the man is-
blocks is 5 kg. and the force acting on it is 20 (A) (2 + 2 ) m/s (B) (2 + 2 2 ) m/s
N . If the kinetic energy acquired by the block
be 40J, at what angle to the path the force is (C) 4 m/s (D) (1 + 2 ) m/s
acting- Sol.
(A) 30º (B) 60º
(C) 45º (D) none of the above
Sol.

41. The work done in pushing a block of mass 44. A block of mass m is suspended by a light thread
10 kg from bottom to the top of a frictionless from an elevator. The elevator is accelerating upward
inclined plane 5 m long and 3 m high is- with uniform acceleration a. The work done by
(g = 9.8 m/sec2 ) tension on the block during t seconds is :
(A) 392 J (B) 294 J
(C) 98 J (D) 0.98 J
Sol. T
a
m

m m
(A) (g  a)at 2 (B) (g – a)at 2
2 2
m
(C) gat 2 (D) 0
2
42. A force F = (3x i + 4 j ) Newton (where x is Sol.
in metres) acts on a particle which moves from
a position (2m, 3m) to (3m, 0m). Then the work
done is
(A) 7.5J (B) –12J
(C) –4.5 J (D) +4.5 J
Sol.

: 0744-2209671, 08003899588 | url : www.motioniitjee.com, : info@motioniitjee.com


Page # 166 CIRCULAR & WORK, POWER & ENERGY

45. The work done by the frictional force on a surface 48. The work done is joules in increasing the
in drawing a circle of radius r on the surface by a extension of a spring of stiffness 10 N/cm from 4 cm
pencil of negligible mass with a normal pressing force to 6 cm is :
N (coefficient of friction k) is : (A) 1 (B) 10
(A) 4 r 2  kN (B) –2r 2 kN (C) 50 (D) 100
Sol.
(C) –3r 2  kN (D) –2r kN
Sol.

49.When a conservative force does positive work


on a body
(A) the potential energy increases
(B) the potential energy decreases
46. A body of mass m accelerates uniformly from (C) total energy increases
rest to a speed v0 in time t0. The work done on the (D) total energy decreases
body till any time t is Sol.

1 2 t
 2  1 2  t0 
(A) 2 mv 0  2  (B) 2 mv 0  t 
 t0 
3
2 t  2 t 
(C) mv 0  t  (D) mv 0  
 0  t0 

Sol.

50. The P.E. of a certain spring when stretched from


natural length through a distance 0.3 m is 10 J. The
amount of work in joule that must be done on this
spring to stretch it through an additional distance
0.15 m will be
(A) 10 J (B) 20 J (C) 7.5 J (D) 12.5 J
Sol.

47. A light spring of length 20 cm and force constant


2 kg/cm is placed vertically on a table. A small block
of mass 1 kg falls on it. The length h from the surface
of the table at which the ball will have the maximum
velocity is -
(A) 20 cm (B) 15 cm
(C) 10 cm (D) 5 cm
Sol.
51. A man who is running has half the kinetic energy
of the boy of half his mass. The man speeds up by 1
m/s and then has the same kinetic energy as the
boy. The original speed of the man was
(A) 2m/ s (B) ( 2 – 1)m / s

(C) 2 m/s (D) ( 2  1)m / s

Corporate Head Office : Motion Education Pvt. Ltd., 394 - Rajeev Gandhi Nagar, Kota-5 (Raj.)
CIRCULAR & WORK, POWER & ENERGY Page # 167

Sol. 54. A car of mass 'm' is driven with acceleration 'a'


along a straight level road against a constant external
resistive force 'R'. When the velocity of the car is
'V', the rate at which the engine of the car is doing
work will be :
(A) RV (B) maV
(C) (R + ma) V (D) (ma – R)V
Sol.

52. A particle is released from rest at origin. It moves


under influence of potential field U = x2 – 3x, kinetic
energy at x = 2 is
(A) 2 J (B) 1 J (C) 1.5 J (D) 0 J
Sol.

55. A pump ejects 12000 kg of water at speed


of 4 m/s in 40 second. Find the average rate at
which the pump is working
(A) 0.24 KW (B) 2.4 W (C) 2.4 KW (D) 24 W
Sol.

53. A 1.0 kg block collides with a horizontal weightless


spring of force constant 2.75 Nm–1 as shown in figure.
The block compresses the spring 4.0 m from the
rest position. If the coefficient of kinetic friction
between the block and horizontal surface is 0.25,
the speed of the block at the instant of collision is
56. The tube AC forms a quarter circle in a vertical
plane. The ball B has an area of cross-section slightly
smaller than that of the tube, and can move without
friction through it. B is placed at A and displaced
slightly. It will
A
B
(A) 0.4 ms–1 (B) 4 ms–1 (C) 0.8 ms–1 (D) 8 ms–1
Sol.

C
(A) always be in contact with the inner wall of the
tube
(B) always be in contact with the outer wall of the
tube
(C) initially be in contact with the inner wall and
later with the outer wall
(D) initially be in contact with the outer wall and
later with the inner wall

: 0744-2209671, 08003899588 | url : www.motioniitjee.com, : info@motioniitjee.com


Page # 168 CIRCULAR & WORK, POWER & ENERGY

Sol. Sol.

59. A part i c l e m ov es wi th a v el oc i t y

57. The relationship between force and position v  (5 i – 3 j  6k ) m / s unde r the i nfl uence of a
is shown in fig (in one dimensional case). The 
work done in displacing a body from x = 1cm to c onst ant forc e F  (10 i  10 j  20k )N. The
x = 5 cm is : instantaneous power applied to the particle is :
(A) 200 J/s (B) 40 J/s
(C) 140 J/s (D) 170 J/s
Sol.

(A) 20 erg (B) 60 erg


(C) 70 erg (D) 700 erg
Sol.

60. The diagrams represent the potential energy U


of a function of the inter-atomic distance r. Which
diagram corresponds to stable molecules found in
nature.
U U

(A) (B)

r r
O O
U U

58. A truck of mass 30,000 kg moves up an inclined (C) (D)


plane of slope 1 in 100 at a speed of 30 kmph. The
power of the truck is (given g = 10 ms–2) r r
(A) 25 kW (B) 10 kW O O
(C) 5 kW (D) 2.5 kW

Corporate Head Office : Motion Education Pvt. Ltd., 394 - Rajeev Gandhi Nagar, Kota-5 (Raj.)
CIRCULAR & WORK, POWER & ENERGY Page # 169

Sol. 62. A particle is rotated in a vertical circle by


connecting it to a light rod of length l and keeping
the other end of the rod fixed. The minimum speed
of particle when the light rod is horizontal for which
the particle will complete the circle is
(A) gl (B) 2gl
(C) 3gl (D) none
Sol.

61. F = 2x2 – 3x –2. Choose correct option


(A) x = –1/2 is position of stable equilibrium
(B) x = 2 is position of stable equilibrium
(C) x = –1/2 is position of unstable equilibrium
(D) x = 2 is position of neutral equilibrium
Sol.

: 0744-2209671, 08003899588 | url : www.motioniitjee.com, : info@motioniitjee.com


Page # 170 CIRCULAR & WORK, POWER & ENERGY

Exercise - II
(A) CIRCULAR MOTION Sol.

1. A person applies a constant force F on a particle
of mass m and finds that the particle moves in a
circle of radius r with a uniform speed v as seen (in
the plane of motion) from an inertial frame of
reference.
(A) This is not possible.
(B) There are other forces on the particle.
mv 2
(C) The resultant of the other forces is towards
r
the centre.
(D) The resultant of the other forces varies in
magnitude as well as in direction.
Sol.

3. A simple pendulum of length l and mass (bob) M is


oscillating in a plane about a vertical line between
angular limits – and . For an angular displacement
, [|| < ] the tension in the string and velocity of
the bob are T and v respectively. The following
relations hold good under the above conditions :
(A) T cos  = Mg
Mv 2
(B) T – Mg cos  =
L
(C) Tangential acc. = g sin 
(D) T = Mg cos 
Sol.

2. A machine, in an amusement park, consists of a


cage at the end of one arm, hinged at O. The cage
revolves along a vertical circle of radius r (ABCDEFGH)
about its hinge O, at constant l inear speed
v = gr . The cage is so attached that the man of
weight ‘w’ standing on a weighing machine, inside
the cage, is always vertical. Then which of the
following is correct E
F× ×
×D
G× r ×C
× × 4. A spot light S rotates in a horizontal plane with a
H B constant angular velocity of 0.1 rad/s. The spot of
×
A light p moves along the wall at a distance 3 m. What
(A) the reading of his weight on the machine is the is the velocity of the spot P when  = 45° ?
Wall P
same at all positions
(B) the weight reading at A is greater than the weight 
reading at E by 2 w. 3m (Top view)
(C) the weight reading at G = w
(D) the ratio of the weight reading at E to that at A=0 S(Spot light)
(E) the ratio of the weight reading at A to that at C=2 (A) 0.6 m/s (B) 0.5 m/s
(C) 0.4 m/s (D) 0.3 m/s

Corporate Head Office : Motion Education Pvt. Ltd., 394 - Rajeev Gandhi Nagar, Kota-5 (Raj.)
CIRCULAR & WORK, POWER & ENERGY Page # 171

Sol.
Sol.

7. A particle moves along an arc of a circle of radius


R. Its velocity depends on the distance covered as
5. Two moving particle P and Q are 10 m apart at a v = a s , where a is a constant then the angle 
certain instant. The velocity of P is 8m/s making an between the vector of the total acceleration and
angle 30° with the line joining P and Q and that of Q the vector of velocity as a function of s will be
is 6m/s making an angle 30° with PQ as shown in R
the figure. Then angular velocity of P with respect (A) tan = (B) tan = 2s / R
2s
to Q is 2R s
6m/s (C) tan  = (D) tan =
s 2R
Sol.
10m 30°
P 30° Q

8m/s
(A) Zero (B) 0.1 rad/sec
(C) 0.4 rad/sec (D) 0.7 rad sec
Sol.

8. A particle A moves along a circle of radius R = 50


cm so that its radius vector r relative to the point O
(Fig.) rotates with the constant angular velocity
= 0.40 rad/s. Then modulus of the velocity of the
particle, and the modulus of its total acceleration
will be A
r
R
6. The magnitude of displacement of a particle
0
moving in a circle of radius a with constant angular
speed  varies with time t as
t (A) v = 0.4 m/s, a = 0.4 m/s2
(A) 2 a sin t (B) 2a sin
2 (B) v = 0.32 m/s, a = 0.32 m/s2
t (C) v = 0.32 m/s, a = 0.4 m/s2
(C) 2a cos t (D) 2a cos (D) v = 0.4 m/s, a = 0.32 m/s2
2

: 0744-2209671, 08003899588 | url : www.motioniitjee.com, : info@motioniitjee.com


Page # 172 CIRCULAR & WORK, POWER & ENERGY

Sol. 11. Three identical particles are joined together by


a thread as shown in figure. All the three particles
are moving on a smooth horizontal plane about point
O. If the speed of the outermost particle is v0, then
the ratio of the tensions in the three sections of the
string is : (Assume that the string remains straight)
O A B C
  
(A) 3 : 5 : 7 (B) 3 : 4 : 5
(C) 7 : 11 : 6 (D) 3 : 5 : 6
Sol.

9. Tangential acceleration of a particle moving in a


circle of radius 1 m varies with time t as (initial
velocity of particle is zero). Time after which total
cceleration of particle makes and angle of 30º with
radial acceleration is

12. A particle is kept fixed on a turntable rotating


60º uniformly. As seen from the ground, the particle goes
time(sec) in a circle, its speed is 20 cm/s and acceleration is
20 cm/s2. The particle is now shifted to a new position
(A) 4 sec (B) 4/3 sec
to make the radius half of the original value. The new
(C) 2 2/3
sec (D) 2 sec values of the speed and acceleration will be
(A) 10 cm/s, 10 cm/s2 (B) 10 cm/s, 80 cm/s2
Sol.
(C) 40 cm/s, 10 cm/s2 (D) 40 cm/s, 40 cm/s2
Sol.

10. A stone of mass of 16 kg is attached to a string


144 m long and is whirled in a horizontal smooth
surface. The maximum tension the string can
withstand is 16 newton. The maximum speed of
revolution of the stone without breaking it, will be :
(A) 20 ms–1 (B) 16 ms–1 (C) 14 ms–1 (D) 12 ms–1
Sol.
13. The kinetic energy k of a particle moving along
a circle of radius R depends on the distance covered
s as k = as2 where a is a positive constant. The
total force acting on the particle is :
1/ 2
s2  s2 
(A) 2a (B) 2as  1  2 
R  R 
R2
(C) 2 as (D) 2a
S

Corporate Head Office : Motion Education Pvt. Ltd., 394 - Rajeev Gandhi Nagar, Kota-5 (Raj.)
CIRCULAR & WORK, POWER & ENERGY Page # 173

Sol. 16. A uniform rod of mass m and length  rotates in


a horizontal plane with an angular velocity  about
a vertical axis passing through one end. The tension
in the rod at distance x from the axis is :
1 1 x2
(A) m 2 x (B) m 2
2 2 
1 x 2
2  1 m
(C) m 1 –  (D) [ 2 – x2 ]
2  2 
Sol.

14. A particle of mass m is fixed to one end of a


light spring of force constant k and unstretched
length  . The system is rotated about the other
end of the spring with an angular velocity , in gravity
free space. The increase in length of the spring will
be
k m

m 2  m 2 
(A) (B)
k k  m 2 17. The ratio of period of oscillation of the conical
m 2  pendulum to that of the simple pendulum is : (Assume
(C) (D) None of these the strings are of the same length in the two cases
k  m 2
Sol. and  is the angle made by the string with the vertical
in case of conical pendulum)
(A) cos (B) cos 
(C) 1 (D) none of these
Sol.

15. A unifrom circular ring of mass per unit length 


and radius R is rotating with angular velocity  about
its own axis in a gravity free space. Tension in the
ring is
1
(A) zero (B)  R2 2
2
(C)  R2 2 (D)  R 2
Sol.
(B) WORK, POWER AND ENERGY

18. No work is done by a force on an object if


(A) the force is always perpendicular to its velocity
(B) the force is al ways perpendi cul ar to i ts
acceleration
(C) the object is stationary but the point of
application of the force moves on the object.
(D) the object moves in such a way that the point
of application of the force remains fixed.

: 0744-2209671, 08003899588 | url : www.motioniitjee.com, : info@motioniitjee.com


Page # 174 CIRCULAR & WORK, POWER & ENERGY

Sol. 21. When total work done on a particle is positive


(A) KE remains constant (B) Momentum increases
(C) KE decreases (D) KE increases
Sol.

19. One end of a light spring of spring constant k is


fixed to a wall and the other end is tied to a block
placed on a smooth horizontal surface. In a 22. A particle with constant total energy E moves in
1 2 one dimension in a region where the potential energy
displacement, the work done by the spring is kx .
2 is U(x). The speed of the particle is zero where
The possible cases are : (A) U(x) = E (B) U(x) = 0
(A) the spring was initially compressed by a distance
x and was finally in its natural length dU( x) d2U( x)
(C) 0 (D) 0
(B) it was initially stretched by a distance x and dx dx2
finally was in its natural length
(C) it was initially in its natural length and finally in a Sol.
compressed position.
(D) it was initially in its natural length and finally in a
stretched position.
Sol.

23. A block of mass m slides down a plane inclined


at an angle . Which of the following will NOT increase
the energy lost by the block due to friction ?
(A) Increasing the angle of inclination
20. Work done by force of friction (B) Increasing the distance that the block travels
(A) can be zero (C) Increasing the acceleration due to gravity
(B) can be positive (D) Increasing the mass of the block
(C) can be negative Sol.
(D) information insufficient
Sol.

Corporate Head Office : Motion Education Pvt. Ltd., 394 - Rajeev Gandhi Nagar, Kota-5 (Raj.)
CIRCULAR & WORK, POWER & ENERGY Page # 175

24. A box of mass m is released from rest at position 26. A cart moves with a constant speed along a
on the frictionless curved track shown. It slides a horizontal circular path. From the cart, a particle is
distance d along the track in time t to reach position thrown up vertically with respect to the cart
2, dropping a vertical distance h. Let v and a be the (A) The particle will land somewhere on the circular
instantaneous speed and instantaneous acceleration, path
respectively, of the box at position 2. Which of the (B) The particle will land outside the circular path
following equations is valid for this situation? (C) The particle will follow an elliptical path
1 (D) The particle will follow a parabolic path
m Sol.
h
2
d

(A) h = vt (B) h = (1/2)gt2

(C) d = (1/2)at2 (D) mgh = (1/2)mv2


Sol.

25. A ball of mass m is attached to the lower end of


light vertical spring of force constant k. The upper 27. The potential energy in joules of a particle of
end of the spring is fixed. The ball is released from mass 1 kg moving in a plane is given by U = 3x + 4y,
rest with the spring at its normal (unstreched) length, the position coordinates of the point being x and y,
comes to rest again after descending through a measured in meters. If the particle is initially at rest
distance x. at (6, 4), then
(A) x = mg/k (A) its acceleration is of magnitude 5 m/s2
(B) x = 2 mg/k (B) its speed when it crosses the y-axis is 10 m/s
(C) The ball will have no acceleration at the position (C) it crosses the y-axis (x = 0) at y = –4
where it has descended through x/2. (D) it moves in a straight line passing through the
(D) The ball will have an upward acceleration equal origin (0, 0)
to g at its lowermost position. Sol.
Sol.

: 0744-2209671, 08003899588 | url : www.motioniitjee.com, : info@motioniitjee.com


Page # 176 CIRCULAR & WORK, POWER & ENERGY

Question No. 28 to 30 (3 questions) 30. The minimum value of H required so that the
A particle of mass m is released from a height particle makes a complete vertical circle is given by
H on a smooth curved surface which ends into a
(A) 5 R (B) 4 R (C) 2.5 R (D) 2 R
vertical loop of radius R, as shown
Sol.
m

R
C
H

28. Choose the correct alternative(s) if H = 2R


(A) The particle reaches the top of the loop with
zero velocity
(B) The particle cannot reach the top of the loop
(C) The particle breaks off at a height H = R from
the base of the loop
(D) The particle break off at a height R < H < 2R
Sol.
31. Equal force F(> mg) is applied to string in all the
three cases. Starting from rest, the point of
application of force moves a distance of 2 m down
in all cases. In which case the block has maximum
kinetic energy?

F m F F
m m
(1) (2) (3)
(A) 1 (B) 2
(C) 3 (D) equal in all 3 cases
29. If  is instantaneous angle which the line joining Sol.
the particle and the centre of the loop makes with
the vertical, then identify the correct statement(s)
related to the normal reaction N between the block
and the surface
(A) The maximum value N occurs at  = 0
(B) The minimum value of N occurs at N =  for H > 5R/2
(C) The value of N becomes negative for /2 <  < 3/2
(D) The value of N becomes zero only when  /2
Sol.

32. Two springs have their force constant as k1 and


k2(k1 > k2). When they are stretched by the same
force
(A) No work is done by this force in case of both the
springs
(B) Equal work is done by this force in case of both
the springs
(C) More work is done by this force in case of second
spring
(D) More work is done by this force in case of first
spring

Corporate Head Office : Motion Education Pvt. Ltd., 394 - Rajeev Gandhi Nagar, Kota-5 (Raj.)
CIRCULAR & WORK, POWER & ENERGY Page # 177

Sol. 35. In the figure shown all the surfaces are


frictionless, and mass of the block, m = 1kg. The
block and wedge are held initially at rest. Now wedge
is given a horizontal acceleration of 10 m/s2 by
applying a force on the wedge, so that the block
does not slip on the wedge. Then work done by the
normal force in ground frame on the block in 3
seconds is

10m/s2
m
M

(A) 30 J (B) 60 J
(C) 150 J (D) 100 3J
Sol.

33. A force F  k[ yi  xj ] where k is a positive
constant acts on a particle moving in x-y plane
starting from the point (3, 5), the particle is taken
along a straight line to (5, 7). The work done by the
force is :
(A) zero (B) 35 K
(C) 20 K (D) 15 K
Sol.

34. A 10 kg block is pulled in the vertical plane Question No. 36 to 37 (2 questions)


along a frictionless surface in the form of an arc of A spring block system is placed on a rough horizontal
a circle of radius 10 m. The applied force is 200 N floor. The block is pulled towards right to give spring
as shown in the figure. If the block started from 2mg mg
rest at A, the velocity at B would be : an elongation less than but more than
K K
and released.
60° 36. Which of the following laws/principles of physics
F can be applied on the spring block system
B

A
(A) 1.732 m/s (B) 17.32 m/s
(C) 173.2 m/s (D) None of these (A) conservation of mechanical energy
Sol. (B) conservation of momentum
(C) work energy principle
(D) None
Sol.

: 0744-2209671, 08003899588 | url : www.motioniitjee.com, : info@motioniitjee.com


Page # 178 CIRCULAR & WORK, POWER & ENERGY

37. The correct statement is 39. A wedge of mass M fitted with a spring of stiffness
(A) The block will cross the mean position 'k' is kept on a smooth horizontal surface. A rod of
(B) The block come to rest when the forces acting mass m is kept on the wedge as shown in the figure.
on it are exactly balanced System is in equilibrium. Assuming that all surfaces
(C) The block will come to rest when the work done are smooth, the potential energy stored in the spring
is :
by friction becomes equal to the change in energy
stored in spring.
(D) None m
Sol. k
M

mg2 tan 2  m 2 g tan2 


(A) (B)
2K 2K

m 2 g2 tan2  m 2 g2 tan2 
(C) (D)
2K K
Sol.

38. A toy car of mass 5 kg moves up a ramp under


the influence of force F plotted against displacement
x. The maximum height attained is given by
100
F
80
ymax 60
40 40. A block of mass m is hung vertically from an elastic
x=0 x=11m 20 thread of force constant mg/a. Initially the thread
0 2 4 6 8 10 12 x was at its natural length and the block is allowed to
(A) ymax = 20 m (B) ymax = 15 m fall freely. The kinetic energy of the block when it
(C) ymax = 11 m (D) ymax = 5 m passes through the equilibrium position will be :
Sol. (A) mga (B) mga/2 (C) zero (D) 2mga
Sol.

Corporate Head Office : Motion Education Pvt. Ltd., 394 - Rajeev Gandhi Nagar, Kota-5 (Raj.)
CIRCULAR & WORK, POWER & ENERGY Page # 179

Question No. 41 to 46 (6 questions) 43. To an observer A, the net work done on the
A block of mass m moving with a velocity v0 on a block is
smooth horizontal surface strikes and compresses a (A) –mv 20 (B) mv 20
spring of stiffness k till mass comes to rest as shown
1 2
in the figure. This phenomenon is observed by two (C) – mv 0 (D) zero
observers: 2
V0 Sol.

K
m

A : standing on the horizontal surface


B : standing on the block
41. To an observer A, the work done by spring force
is
(A) negative but nothing can be said about its
magnitude
1 2 44. According to the observer A
(B) – mv 0 (A) the kinetic energy of the block is converted into
2
(C) positive but nothing can be said about its the potential energy of the spring
magnitude (B) the mechanical energy of the spring-mass system
1 2
is conserved
(D)  mv 0 (C) the block loses its kinetic energy because of the
2
negative work done the conservative force of spring
Sol. (D) all the above
Sol.

42. To an observer A, the work done by the normal


reaction N between the block and the spring on the 45. To an observer B, when the block is compressing
block is the spring
1 2 (A) velocity of the block is decreasing
(A) zero (B) – mv 0 (B) retardation of the block is increasing
2
1 2
(C) kinetic energy of the block is zero
(C)  mv 0 (D) none of these (D) all the above
2
Sol. Sol.

: 0744-2209671, 08003899588 | url : www.motioniitjee.com, : info@motioniitjee.com


Page # 180 CIRCULAR & WORK, POWER & ENERGY

46. According to observer B, the potential energy Sol.


of the spring increases
(A) due to the positive work done by pseudo force
(B) due to the positive work done by normal reaction
between spring & wall
(C) due to the decrease in the kinetic energy of the
block
(D) all the above
Sol.


49. The potential energy for a force field F is given
by U(x, y) = sin (x + y). The force acting on the
 
particle of mass m at  0,  is
4
1
(A) 1 (B) 2 (C) (D) 0
2
Sol.

47. Assume the aerodynamic drag force on a car is


proportional to its speed. If the power output from
the engine is doubled, then the maximum speed of
the car.
(A) is unchanged
(B) increases by a factor of 2
(C) is also doubled
(D) increases by a factor of four.
Sol.

50. A particle originally at rest the highest point of


a smooth vertical circle is slightly displaced. It will
leave the circle at a vertical distance h below the
highest point, such that
(A) h = R (B) h = R/3
(C) h = R/2 (D) h = 2R
Sol.

48. A body of mass 1 kg starts moving from rest at


t = 0, in a circular path of radius 8 m. Its kinetic
energy varies as a function of time as : K.E. = 2t2
Joules, where t is in seconds. Then
(A) tangential acceleration = 4m/s2
(B) power of all forces at t = 2 sec is 8 watt
(C) first round is completed in 2 sec.
(D) tangential force at t = 2 sec is 4 newton.

Corporate Head Office : Motion Education Pvt. Ltd., 394 - Rajeev Gandhi Nagar, Kota-5 (Raj.)
CIRCULAR & WORK, POWER & ENERGY Page # 181

51. A ball whose size is slightly smaller than width of 53. A small cube with mass M starts at rest point 1
the tube of radius 2.5 m is projected from bottommost at a height 4R, where R is the radius of the circular
point of a smooth tube fixed in a vertical plane with part of the track. The cube slides down the
velocity of 10 m/s. If N1 and N2 are the normal frictionless track and around the loop. The force
reactions exerted by inner side and outer side of that the track exerts on the cube at point 2 is nearly
the tube on the ball _________ times the cube’s weight Mg.
1
D

A C
1R 2
B
O 10 m/s R
(A) N1 > 0 for motion in ABC, N2 > 0 for motion in CDA
(B) N1 > 0 for motion in CDA, N2 > 0 for motion in ABC (A) 1 (B) 2 (C) 3 (D) 4
(C) N2 > 0 for motion in ABC & part of CDA Sol.
(D) N1 is always zero.
Sol.

52. A bob attached to a string is held horizontal and


released. The tension and vertical distance from
point of suspension can be represented by.

T T
(A) (B)
h h
T T
(C) (D)
h h
Sol.

: 0744-2209671, 08003899588 | url : www.motioniitjee.com, : info@motioniitjee.com


Page # 182 CIRCULAR & WORK, POWER & ENERGY

Exercise - III (JEE ADVANCED)

(A) CIRCULAR MOTION 4. A point moves along a circle having a radius 20


1. The 10 kg block is in equilibrium. cm with a constant tangential acceleration 5 cm/s2.
How much time is needed after motion begins for
(A) 10 kg the normal acceleration of the point to be equal to
tangential acceleration?
Sol.
(B)
(i) Find the tension in string A.
(ii) Find the tension in string A just after the string B
is cut?
Sol.

5. A ring rotates about z axis as shown in figure.


The plane of rotation is xy. At a certain instant the
acceleration of a particle P (shown in figure) on the
2. A particle moves in the x-y plane with the velocity
ring is (6 i – 8 j ) m/s2. find the angular acceleration

v  ai  bt j . At the instant t  a 3 / b the magnitude of the ring & the angular velocity at that instant.
of tangential, normal and total acceleration are Radius of the ring is 2m. y
_________________, _______ & __________. P
Sol.

O x

Sol.

3. A particle moves clockwise in a circle of radius 1


m with centre at (x, y) = (1m, 0). It starts at rest
at the origin at time t = 0. Its speed increases at
 
the constant rate of   m/s2. (a) How long does it
2
take to travel halfway around the circle ? (b) What 6. A particle is revolving in a circle of radius 1m with
is the speed at that time ? an angular speed of 12 rad/s. At t = 0, it was
Sol. subjected to a constant angular acceleration  and
its angular speed increased to (480/) rpm in 2 sec.
Particle then continues to move with attained speed.
Calculate
(a) angular acceleration of the particle,
(b) tangential velocity of the particle as a function
of time.
(c) acceleration of the particle at t = 0.5 second
and at t = 3 second
(d) angular displacement at t = 3 second.

Corporate Head Office : Motion Education Pvt. Ltd., 394 - Rajeev Gandhi Nagar, Kota-5 (Raj.)
CIRCULAR & WORK, POWER & ENERGY Page # 183

Sol. 9. A particle moves in circle of radius R with a


constant speed v. Then, find the magnitude of
R
average acceleration during a time interval .
2v
Sol.

7. Figure shows the total acceleration and velocity


of a particle moving clockwise in a circle of radius 2.5
m at a given instant of time. At this instant, find :
2
a=25 m/s
5m
2.

a v
°
30

10. A 4 kg block is attached to a vertical rod by


means of two strings of equal length. When the
(a) the radial acceleration, system rotaes about the axis of the rod, the
(b) the speed of the particle and strings are extended as shown in figure.
(c) its tangential acceleration (a) How many revolutions per minute must the system
Sol. make in order for the tension in the upper chord to
be 20 kgf?
(b) What is the tension in the lower chord?

5m

8m 
A

Sol.

8. A stone is launched upward at 45° with speed v0.


A bee follows the trajectory of the stone at a
constant speed equal to the initial speed of the
stone.
(a) Find the radius of curvature at the top point of
the trajectory.
(b) What is the acceleration of the bee at the top
point of the trajectory? For the stone, neglect the
air resistance.
Sol.

11. A rod of length 1 m is being rotated about its


end in a gravity free space with a constant angular
acceleration of 5 rad/s2 starting from rest. A sleeve
is fitted on the rod at a distance of 0.5 m from the
centre. The coefficient of friction between the rod
and the sleeve is 0.05. Find the time after which
sleeve will start slipping on the rod.

: 0744-2209671, 08003899588 | url : www.motioniitjee.com, : info@motioniitjee.com


Page # 184 CIRCULAR & WORK, POWER & ENERGY

Sol. 14. The blocks are of mass 2 kg shown is in


equilibrium. At t = 0 right spring in fig. (i) and right
string in fig. (ii) breaks. Find the ratio of instantaneous
acceleration of blocks ?

37° 37° 37°


37°

12. A mass m rotating freely in a horizontal circle of


radius 1 m on a frictionless smooth table supports a 2 kg
2 kg figure (ii)
stationary mass 2m, attached to the other end of
the string passing through smooth hole O in table, Sol.
hanging vertically. Find the angular velocity of
rotation.

O m

2m
Sol.

(B) WORK, POWER AND ENERGY


15. A block of mass m is pulled on a rough horizontal
surface which has a friction coefficient . A force F
isapplied which is capable of moving the body
uniformly with speed v. Find the work done on the
block in time t by (a) weight of the block, (b) Normal
reaction by surface on the block, (c) friction, (d) F.
13. A beam of mass m is attached to one end of a Sol.
spring of natural length 3 R and spring constant

( 3  1)mg
k . The other end of the spring is fixed
R
at point A on a smooth fixed vertical ring of radius R
as shown in the figure. What is the normal reaction
at B just after the bead is released?
B

A 60°
16. Calculate the work done against gravity by a
coolie in carrying a load of mass 10 kg on his head
Sol. when he walks uniformly a distance of 5 m in the (i)
horizontal direction (ii) vertical direction. (Take g =
10 m/s2)
Sol.

Corporate Head Office : Motion Education Pvt. Ltd., 394 - Rajeev Gandhi Nagar, Kota-5 (Raj.)
CIRCULAR & WORK, POWER & ENERGY Page # 185

17. A body is constrained to move in the y-direction. 20. A rigid body of mass 2 kg initially at rest moves
under the action of an applied horizontal force 7 N
It is subjected to a force (–2i  15 j  6k ) newton.
on a table with coefficient of kinetic friction = 0.1.
What is the work done by this force in moving the Calculate the
body through a distance of 10 m ? (a) work done by the applied force on the body in 10 s.
Sol. (b) work done by friction on the body in 10 s.
(c) work done by the net force on the body in 10 s.
(d) change in kinetic energy of the body is 10 s.
Sol.

18. The relationship between force and position is


shown in the figure given (in one dimensional case).
What will be the work done by the force in displacing
a body from x = 1 cm to x = 5 cm. 21. A rigid body of mass 0.3 kg is taken slowly up an
inclined plane of length 10 m and height 5 m, and
20
(Force dyne)

then allowed to slide down to the bottom again.


10 The co-efficient of friction between the body and
0 the plane is 0.15. Using g = 9.8 m/s2 find the
1 2 3 4 5 6 (a) work done by the gravitational force over the round trip.
10 x(cm) (b) work done by the applied force (assuming it to
20 be parallel to the inclined plane) over the upward
journey
Sol.
(c) work done by frictional force over the round
trip.
(d) kinetic energy of the body at the end of the
trip?
Sol.

19. It is well known that a raindrop or a small pebble


fal l s under the i nfl uenc e of t he dow nward
gravitational force and the opposing resistive force.
The latter is known to be proportional to the speed
of the drop but is otherwise undetermined. Consider
22. A block of mass m sits at rest on a frictionless
a drop or small pebble of 1 g falling from a cliff of
table in a rail car that is moving with speed vc along
height 1.00 km. It hits the ground with a speed of
a straight horizontal track (fig.) A person riding in
50.0 ms–1. What is the work done by the unknown
the car pushes on the block with a net horizontal
resistive force ?
force F for a time t in the direction of the car's
Sol.
motion.
Ground Train

m F

s1
s
(a) What is the final speed of the block according
to a person in the car ?

: 0744-2209671, 08003899588 | url : www.motioniitjee.com, : info@motioniitjee.com


Page # 186 CIRCULAR & WORK, POWER & ENERGY

Sol. (h) Compare the work done to the K gain according


to each person.
Sol.

(b) According to a person standing on the ground


outside the train?
Sol. (i) What can your conclude from this computation?
Sol.

(c) How much did K of the block change according


to the person in the car ?
Sol. 23. In the figure shown, pulley and
k
spring are ideal. Find the potential
energy stored in the spring
(m1 > m2) m1 m2

Sol.
(d) According to the person on the ground ?
Sol.

(e) In terms of F, m, & t, how far did the force


displace the object according to the person in car ?
Sol.

24. A block of mass m placed on a smooth horizontal


surface is attached to a spring and is held at rest
by a force P as shown. Suddenly the force P changes
its direction opposite to the previous one. How many
(f) According to the person on the ground ?
times is the maximum extension l2 of the spring longer
Sol.
compared to its initial compression l2 ?
l1
P

(g) How much work does each say the force did ? Sol.
Sol.

Corporate Head Office : Motion Education Pvt. Ltd., 394 - Rajeev Gandhi Nagar, Kota-5 (Raj.)
CIRCULAR & WORK, POWER & ENERGY Page # 187

25. (a) Power applied to a particle varies with time 28. Two trains of equal masses are drawn along
as P = (3t2 – 2t + 1) watt, where t is in second. Find smooth level lines by engines; one of then X exerts
the change in its kinetic energy between time t = 2 a constant force while the other Y works at a
s and t = 4 s. constant rate. Both start from rest & after a time t
Sol. both again have the same velocity v. Find the ratio
of travelled distance during the interval.
Sol.

29. Water is pumped from a depth of 10m and


delivered through a pipe of cross section 10–2m2 upto
(b) The potential function for a conservative force a height of 10m. If it is needed to deliver a volume
is given by U = k(x + y). Find the work done by the 0.2 m3 per second, find the power required. [Use g
conservative force in moving a particle from the point = 10 m/s2]
A(1, 1) to point B (2, 3). Sol.
Sol.

26. A labourer lifts 100 stones to a height of 6 metre


in two minute. If mass of each stone be one kilogram,
calculate the average power. Given : g = 10 ms–2.
Sol.

30. A particle moves along a straight line. A force


acts on the particle which produces a constant
power. It starts with initial velocity 3 m/s and after
moving a distance 252 m its velocity is 6 m/s. Find
the time taken.
Sol.

27. An engine develops 10 kW of power. How much


time will it take to lift a mass of 200 kg through a
height of 40 m? Given : g = 10 ms–2
Sol.

: 0744-2209671, 08003899588 | url : www.motioniitjee.com, : info@motioniitjee.com


Page # 188 CIRCULAR & WORK, POWER & ENERGY

31. A force F = x2y2i + x2y2j (N) acts on a particle Sol.


which moves in the XY plane.

Y a C
D

A X
B
Find the work done by F as it moves the particle
from A to C (fig.) along each of the paths ABC, ADC,
and AC. 34. A person rolls a small ball with speed u along the
Sol. floor from point A. If x = 3R, determine the required
speed u so that the ball returns to A after rolling on
the circular surface in the vertical plane from B to C
and becoming a projectile at C. What is the minimum
value of x for which the game could be played if
contact must be maintained to point C ? Neglect
friction.

32. Calculate the forces F(y) associated with the


following one-dimensional potential energies :
(a) U = –y (b) U = ay3 – by2 Sol.
(c) U = U0 sin y
Sol.

35. A toy rocket of mass 1 kg has a small fuel of


mass 0.02 kg which it burns out in 3 s. Starting from
rest on a horizontal smooth track, it gets a speed of
20 ms–1 after the fuel is burnt out. What is the
33. Consider the shown arrangement when a is bob average thrust of the rocket? What is the energy
of mass ‘m’ is suspended by means of a string content per unit mass of the fuel? (Ignore the small
connected to peg P. If the bob is given a horizontal mass variation of the rocket during fuel burning).
velocity u having magnitude 3gl , find the minimum Sol.
speed of the bob in subsequent motion.
P

Corporate Head Office : Motion Education Pvt. Ltd., 394 - Rajeev Gandhi Nagar, Kota-5 (Raj.)
CIRCULAR & WORK, POWER & ENERGY Page # 189

1. A particle which moves along the curved path 5. A particle of mass 5 kg is free to slide on a smooth
shown passes point O with a speed of 12 m/s and ring of radius r = 20 cm fixed in a vertical plane. The
slows down to 5m/s at point A in a distance of 18 m particle is attached to one end of a spring whose
measured along the curve from O. The deceleration
other end is fixed to the top point O of the ring.
measured along the curve it proportional to distance
Initially the particle is at rest at a point A of the ring
from O. If the total acceleration of the particle is 10
m/s2 on it passes A. Find the radius of curvature of A. such that OCA = 60º, C being the centre of the
ring. The natural length of the spring is also equal to
A r = 20 cm. After the particle is released and slides
O down the ring the contact force between the particle
& the ring becomes zero when it reaches the lowest
position B. Determine the force constant of the
2. A ball of mass 1 kg is released from position A inside a
spring. O
wedge with a hemispherical cut of radius 0.5 m as shown
in the figure. Find the force exerted by the vertical wall A
OM on wedge, when the ball is in position B. (neglect 60°
friction everywhere) Take (g = 10m/s2) C
M
A C
B
60°

6. Two blocks of mass m1 = 10kg and m2 = 5kg


O N
connected to each other by a massless inextensible
3. A small is block can move in a straight horizontal string of length 0.3 m are placed along a diameter of
line a along AB. Flash lights from one side projects a turn table. The coefficient of friction between the
its shadow on a vertical wall which has horizontal table and m1 is 0.5 while there is no friction between
cross section as a circle. Find tangential & normal m2 and the table. The table is rotating with an
acceleration of shadow of the block on the wall as a angular velocity of 10 rad/sec about a vertical axis
function of time if the velocity of the block is
passing through its centre. The masses are placed
constant (v).
along the diameter of the table on either side of the
B
R centre O such that m1 is at a distance of 0.124 m
from O. The masses are observed to be at rest with
v=const respect to an observer on the turn table.
(i) Calculate the frictional force on m1
A
Top View (ii) What should be the minimum angular speed of
the turn table so that the masses will slip from this
4. A particle is confined to move along the +x axis position
under the ac tion of a force F(x) that is derivable (iii) How should the masses be placed with the string
from thePotential U(x) = ax3 – bx. remaining taut, so that there is no frictional force
U
acting on the mass m1.

x 7. A ring of mass m can slide over a smooth vertical


O x1
x0 rod. The ring is connected to a spring of force
(a) Find the expression for F(x) 4mg
constant K  where 2R is the natural length of
(b) When the total energy of the particle is zero, R
the particle can be trapped with in the interval x = the spring. The other end of the spring is fixed to
0 to x = x1. For this case find the values of x1. the ground at a horizontal distance 2R from the base
(c) Determine the maximum kinetic energy that the of the rod. The mass is released at a height of 1.5 R
trapped particle has in its motion. Express all answers from ground
in terms a and b.

: 0744-2209671, 08003899588 | url : www.motioniitjee.com, : info@motioniitjee.com


Page # 190 CIRCULAR & WORK, POWER & ENERGY

(c) If the velocity is double the minimum velocity as


calculated in part (a) then what distance does its
front end A would have travelled in rough region
3R/2 before rod comes to rest.
L
A v0
B M A
2R m
(a) calculate the work done by the spring
(b) calculate the velocity of the ring as it reaches 11. Find the velocity with which a block of mass 1 kg
the ground. must be horizontally projected on a conveyer belt
moving uniformly at a velocity of 3 m/s so that maximum
8. Two identical beads of mass 1 kg each are heat is liberated. Take coefficient of friction of 0.1.
connected by an inextensible massless string & they Also find the corresponding amount of heat liberated.
can slide along the two arms AB and BC of a rigid What happens when belt velocity is 5 m/s ?
smooth wire frame in vertical plane. If the system is 1kg
released from rest, find the speeds of the particles
v=3m/s
when they have moved by a dis tance of 0.1 m.
Also find tension in the string.
0.4m
B 8m
A
0.3m 12. A small bead of mass m is free to slide on a fixed
smooth vertical wire, as indicated in the diagram.
One end of a light elastic string, of unstretched
C length a and force constant 2 mg/a is attached to
B. The string passes through a smooth fixed ring R
and the other end of the string is attached to the
9. The ends of spring are attached to blocks of fixed point A, AR being horizontal. The point O on
mass 3 kg and 3 kg. The 3 kg block rests on a the wire is at same horizontal level as R, and AR =
horizontal surface and the 2 kg block which is RO = a.
vertically above it is in equilibrium producing a (i) In the equilibrium, find OB
compression of 1cm of the spring. The 2kg mass (ii) The bead B is raised to a point C of the wire
must be compressed further above O, where OC = a, and is released from rest.
by at least ________, so that when it is released, Find the speed of the bead as it passes O, and find
the greatest depth below O of the bead in the
the 3 kg block may be lifted off the ground.
subsequent motion.
2kg
a R A
O

3 kg 13. A small block of mass m is projected horizontally


from the top the smooth hemisphere of radius r with
speed u as shown. For values of u  u0, it does not
10. A uniform rod of mass m length L is sliding along slide on the hemisphere (i.e. leaves the surface at
its length on a horizontal table whose top is partly the top itself)
smooth & rest rough with friction coefficient . If (a) For u = 2u0 it lands at point P on ground Find OP.
the rod after moving through smooth part, enters (b) For u = u0/3, Find the height from the ground at
the rough with velocity v0. which it leaves the hemisphere.
(a) What will be the magnitude of the friction force (c) Find its net acceleration at the instant it leaves
when its x length (< L) lies in the rough part during the hemisphere.
sliding.
u
(b) Determine the minimum velocity v0 with which it r
must enter so that it lies completely in rough region
before coming to rest. o

Corporate Head Office : Motion Education Pvt. Ltd., 394 - Rajeev Gandhi Nagar, Kota-5 (Raj.)
CIRCULAR & WORK, POWER & ENERGY Page # 191

Exercise - IV PREVIOUS YEAR QUESTIONS

LEVEL - I JEE MAIN


(Circular Motion) 3. An angular ring with inner and outer radii R1 and
1. The minimum velocity (in ms–1) with which a car R2 is rolling without slipping with a uniform angular
driver must traverse a flat curve of radius 150 m speed. The ratio of the forces experienced by the
and coefficient of friction 0.6 to avoid skidding is : two particles situated on the inner and outer parts
[AIEEE 2002] F1
(A) 60 (B) 30 (C) 15 (D) 25 of the ring,
Sol. F2 is [AIEEE 2005]

2
R2  R1  R1
(A) (B)  (C) 1 (D)
R1  R 2
R2
Sol.

2. Which of the following statements is false for a 4. A point P moves in counter-clockwise direction
particle moving in a circle with a constant angular on a circular path as shown in the figure. The
speed ? [AIEEE 2004] movement of P is such that it sweeps out a length
(A) The velocity vector is tangent to the circle. s = t3 + 5, where s is in metre and t is in second.
(B) The acceleration vector is tangent to the circle. The radius of the path is 20 m. The acceleration of
(C) The acceleration vector points to the centre of P when t = 2s is nearly [AIEEE 2010]
the circle. y
(D) The velocity and acceleration vectors are B
perpendicular to each other. P (x, y)
Sol.
m
20

x
O A
(A) 13 ms–2 (B) 12 ms–2 (C) 7.2 ms–2 (D) 14 ms–2
Sol.

: 0744-2209671, 08003899588 | url : www.motioniitjee.com, : info@motioniitjee.com


Page # 192 CIRCULAR & WORK, POWER & ENERGY

5. For a particle in uniform circular motion the (Work Power, Energy)



acceleration a at a point P(R,  ) on the circle of 7. A spring of force constant 800 N/m has an
extension of 5 cm. The work done in extending it
radius R is (here  is measured from the x-axis) from 5 cm to 15 cm is [AIEEE 2002]
[AIEEE 2010] (A) 16 J (B) 8 J (C) 32 J (D) 24 J
Sol.
v2 v2
(A)  cos  iˆ  sin  ˆj
R R
v2 v2
(B)  sin  iˆ  cos  ˆj
R R
v2 v2
(C)  cos  iˆ  sin  ˆj
R R
v2 ˆ v2 ˆ
(D) i j
R R
Sol.

8. A body is moved along a straight line by a machine


delivering a constant power. The distance moved by
the body in time t is proportional to
[AIEEE 2003]
(A) t3/4 (B) t3/2 (C) t1/4 (D) t1/2
Sol.

6. Two cars of masses m1 and m2 are moving in


circles of radii r1 and r2, respectively. Their speeds
are such that they make complete circles in the
same ti me t. The rati o of thei r centri petal
acceleration is [AIEEE 2012]
(A) m1r1 : m2r2 (B) m1 : m2
(C) r1 : r2 (D) 1 :1
Sol.
9. A spring of spring constant 5 × 103 N/m is
stretched initally by 5 cm from the unstreched
position. Then the work required to stretch it further
by another 5 cm is [AIEEE 2003]
(A) 12.50 N-m (B) 18.75 N-m
(C) 25.00 N-m (D) 6.25 N-m
Sol.

Corporate Head Office : Motion Education Pvt. Ltd., 394 - Rajeev Gandhi Nagar, Kota-5 (Raj.)
CIRCULAR & WORK, POWER & ENERGY Page # 193

 Sol.
10. A force F  (5iˆ  3iˆ  2k)N
ˆ is applied over a
particle which displaces it from its origin to the point

r  (2iˆ  ˆj) m. The work done on the particle in
joules in [AIEEE 2004]
(A) –7 (B) +7 (C) +10 (D) +13
Sol.

13.The block of mass M moving on the frictionless


horizontal surface collides with the spring of spring
constant k and compresses it by length L. The
maximum momentum of the block after collision is
[AIEEE 2005]

11.A uniform chain of length 2 m is kept on a table


such that a length of 60 cm hangs freely from the
edge of the table. The total mass of the chain is 4
kg. What is the work done in pulling the entire chain
M
on the table? [AIEEE 2004]
(A) 7.2 J (B) 3.6 J (C) 120 J (D) 1200 J
Sol.

kL2 ML2
(A) Mk L (B) (C) zero (D)
2M k
Sol.

14.A spherical ball of mass 20 kg is stationary at the


top of a hill of height 100 m. It rools down a smooth
surface to the ground, then climbs up another hill of
12.A particle moves in a straight line with retardation height 30 m and finally rolls down to a horizontal
proportional to its displacement. Its loss of kinetic base at a height of 20 m above the ground. The
energy for any displacement x is proportional to velocity attained by the ball is
[AIEEE 2004] [AIEEE 2005]
(A) x2 (B) ex (C) x (D) logex (A) 40 m/s (B) 20 m/s
(C) 10 m/s (D) 10 30 m/s

: 0744-2209671, 08003899588 | url : www.motioniitjee.com, : info@motioniitjee.com


Page # 194 CIRCULAR & WORK, POWER & ENERGY

Sol. 17. A mass of M kg is suspended by a weightless


string. The horizontal force that is required to
displace it until the string makes an angle of 45°
with the initial vertical direction is [AIEEE 2006]
(A) Mg ( 2  1) (B) Mg 2

Mg
(C) (D) Mg ( 2  1)
2
Sol.

15.A body of mass m is accelerated uniformly from


rest to a speed V in a time T. The instantaneous
power delivered to the body as function of time, is
given by [AIEEE 2005]

mv 2 mv 2 2 1 mv 2 1 mv 2 2
(A) t (B) t (C) t (D) t
T2 T2 2 T2 2 T2 18.A ball of mass 0.2 kg is thrown vertically upwards
Sol. by applying a force by hand. If the hand moves 0.2
m while applying the force and the ball goes upto 2
m height further, find the magnitude of the force.
Consider g = 10 m/s2. [AIEEE 2006]
(A) 4 N (B) 16 N (C) 20 N (D) 22 N
Sol.

16.A bullet fired into a fixed target loses half of its


velocity after penetrating 3 cm. How much further
it will penetrate before coming to rest, assuming
that it faces constant resistance to motion ?
[AIEEE 2005]
(A) 3.0 cm (B) 2.0 cm (C) 1.5 cm (D) 1.0 cm
Sol.

19.The potential energy of a 1 kg particle free to


move along the x-axis is given by [AIEEE 2006]

 x4 x2 
V ( x)     J
 4 2
The total mechanical energy of the particle is 2 J.
Then, the maximum speed (in ms–1) is
3 1
(A) (B) 2 (C) (D) 2
2 2
Corporate Head Office : Motion Education Pvt. Ltd., 394 - Rajeev Gandhi Nagar, Kota-5 (Raj.)
CIRCULAR & WORK, POWER & ENERGY Page # 195

Sol. 22.At time t = 0 s particle starts moving along the


x-axis. If its kinetic energy increase uniformly with
time t, the net force acting on it must be proportional
to [AIEEE 2011]
1
(A) t (B) constant (C) t (D)
t
Sol.

20.A 2 kg block slides on a horizontal floor with a


speed of 4 m/s. It strikes a uncompressed spring,
and compresses it till the block is motionless. The
kinetic friction force is 15 N and spring constant is
10000 N/m. The spring compresses by
[AIEEE 2007]
(A) 5.5 cm (B) 2.5 cm
(C) 11.0 cm (D) 8.5 cm
Sol.

21.An athlete in the olympic games covers a distance


of 100 m in 10 s. His kinetic energy can be estimated
to be in the range [AIEEE 2008]
(A) 200J–500J (B) 2×105J–3×105 J
(C) 20000J–50000J (D) 2000J – 5000J
Sol.

: 0744-2209671, 08003899588 | url : www.motioniitjee.com, : info@motioniitjee.com


Page # 196 CIRCULAR & WORK, POWER & ENERGY

LEVEL - II JEE ADVANCED


1. An ideal spring with spring-constant k is hung 3. A particle, which is constrained to move along x-
from the ceiling and a block of mass M is attached axis, is subjected to a force in the same direction
to its lower end. The mass is released with the spring which varies with the distance x of the particle from
initially unstretched. Then the maximum extension the origin as F (x) =-kx + ax3, Here, k and a are
in the spring is [JEE(Scr.)-2002] positive constant. Fro x  0, the functional form of
(A) 4 Mg/k (B) 2 Mg/k (C) Mg/k (D) Mg/2k the potential energy U (x) of the particle is :
Sol. [JEE 2002]
U(x) U(x)

x x
(A) (B)

U(x) U(x)

x x
(C) (D)

2. A spherical ball of mass m is kept at the highest Sol.


point in the space between two fixed, concentric
spheres A and B (see figure.) The smaller sphere A
has a radius R and the space between the two
spheres has a width d. The ball has a diameter very
slightly less than d. All surfaces are frictionless. The
ball is given a gentle push (towards the right in the
figure). The angle made by the radius vector of the
ball with the upward vertical is denoted by  (shown
in the figure) [JEE-2002]
(a) Express the total normal reaction force exerted 4. In a region of only gravitational field of mass ‘M’ a
by the spheres on the ball as a function of angle . particle is shifted from A to B via three different
Sphere B paths in the figure. The work done in different paths
are W1, W2, W3 respectively then [JEE(Scr.)-2003]

d O (3)
R B C
M
Sphere A
(2)
(b) Let NA and NB denote the magnitudes of the
normal reaction force on the ball exerted by the
sp here s A and B, resp ect i vel y. Sket ch t he (1)
variations of NA and NB as functions of cos in the A
range 0  by drawing two separate graphs in
your answer book, taking cos on the horizontal axes. (A) W1 = W2 = W3 (B) W1 = W2 > W3
Sol. (C) W1 > W2 > W3 (D) W1 < W2 < W3
Sol.

Corporate Head Office : Motion Education Pvt. Ltd., 394 - Rajeev Gandhi Nagar, Kota-5 (Raj.)
CIRCULAR & WORK, POWER & ENERGY Page # 197

5. A particle is placed at the origin and a force F =   


kx is acting on it (where k is a positive constant). If (A)   (B) 
4 4 2
U(0) = 0, the graph of U(x) versus x will be (where U
 3 3
is the potential energy function) [JEE(Scr.)-2004] (C)  (D) 
U(x) 2 4 4
Sol.
U(x)
x x
(A) (B)

U(x) U(x)

x
(C) (D)
x

Sol.

8. A light inextensible string that goes over a smooth


fixed pulley as shown in the figure connects two
blocks of masses 0.36 kg and 0.72 kg. Taking g = 10
m/s2, find the work done (in joules) by the string on
6. Statement I A block of mass m starts moving on
the block of mass 0.36 kg during the first second
a rough horizontal surface with a velocity v. It stops
after the sysem is released from rest. [JEE 2009]
due to friction between the block and the surface
after moving through a certain distance. The sur-
face is now tilted to an angle of 30° with the hori-
zontal and the same block is made to go up on the
surface with the same initial velocity v. The de-
crease in the mechanical energy in the second situ-
ation is smaller than that is the first situation.
Statement II The coefficient of friction between Sol.
the block and the surface decreases with the
increase in the angle of inclination. [JEE 2007]
(A) Statement-1 is True, Statement-2 is True;
State me nt -2 i s a correc t ex pl anat i on for
Statement-1
(B) Statement-1 is True, Statement-2 is True; 9. A block of mass 2 kg is free to move along the
Statement-2 is NOT a correct explanation for x-axis. It is at rest and from t = 0 onwards it is
Statement - 1 subjected to a time-dependent force F(t) in the x
(C) Statement-1 is True, Statement-2 is False -direction. The force F(t) varies with t as shown in
(D) Statement-1 is False, Statement-2 is True the figure. The kinetic energy of the block after 4.5
Sol. s is
F(t)

4N

[JEE 2010]
7. A bob of mass M is suspended by a massless 4.5s
t
string of length L. The horizontal velocity V at O 3s
position A is just sufficient to make it reach the
point B. The angle at which the speed of the bob
(A) 4.50 J (B) 7.50 J (C) 5.06 J (D) 14.06 J
is half of that at A, satisfies [JEE 2008]
Sol.
B

V
A

: 0744-2209671, 08003899588 | url : www.motioniitjee.com, : info@motioniitjee.com


Page # 198 CIRCULAR & WORK, POWER & ENERGY

10. A ball of mass (m) 0.5 kg is attached to the end 12. Two identical discs of same radius R are rotating
of a string having length (L) 0.5 m. The ball is about their axes in opposite directions with the same
rotated on a horizontal circular path about vertical constant angular speed . The discs are in the same
axis. The maximum tension that the string can bear horizontal plane At time t = 0, the points P and Q
is 324 N. the maximum possible value of angular are facing each other as shown in the figure. The
velocity of ball (in radian/s) is [JEE-2011] relative speed between the two points P and Q is
r.In one time period (T) of rotation of the discs, r
as a function of time is best represented by
[JEE-2012]
L
P Q
R R
m
(A) 9 (B) 18 (C) 27 (D) 36
Sol.

(A)

0 t
T

(B)

0 t
T

11. A block of mass 0.18 kg is attached to a spring


of force-constant 2 N/m. The coefficient of friction
between the block and the floor is 0.1. Initially the
block is at rest and the spring is un-stretched. An
impulse is given to the block as shown in the figure. (C)
The block slides a distance of 0.06 m and comes to t
rest for the first time. The initial velocity of the 0 T
block in m/s is V = N/10. Then N is [JEE-2011]

(D)
0 t
T
Sol.

Sol.

Corporate Head Office : Motion Education Pvt. Ltd., 394 - Rajeev Gandhi Nagar, Kota-5 (Raj.)
CIRCULAR & WORK, POWER & ENERGY Page # 199

Exercise - I OBJECTIVE PROBLEMS (JEE MAIN)


(A) CIRCULAR MOTION
1. C 2. C 3. D 4. A 5. D 6. C 7. A
8. B 9. C 10. B 11. B 12. B 13. A 14. C
15. A 16. C 17. (i)A,(ii) A 18. C 19. C 20. C
21. D 22. C 23. C 24. A 25. C 26. C 27. D
28. B 29. A 30. C 31. A 32. B 33. A 34. D
35. C 36. C 37. B
(B) WORK, POWER AND ENERGY
38. B 39. C 40. B 41. B 42. C 43. B 44. A
45. D 46. A 47. B 48. A 49. B 50. D 51. D
52. A 53. D 54. C 55. C 56. C 57. A 58. A
59. C 60. A 61. A 62. B

Exercise - II
(A) CIRCULAR MOTION
1. BD 2. BCDE 3. BC 4. A 5. D 6. B 7. B
8. D 9. C 10. D 11. D 12. A 13. B 14. B
15. C 16. D 17. B
(B) WORK, POWER AND ENERGY
18. ACD 19. AB 20. ABC 21. BD 22. A 23. A 24. D
25. BCD 26. BD 27. ABC 28. BD 29. ABD 30. C 31. C
32. C 33. C 34. B 35. C 36. C 37. C 38. C
39. C 40. B 41. B 42. B 43. C 44. D 45. C
46. B 47. B 48. B 49. A 50. B 51. C 52. A
53. C

Exercise - III (JEE ADVANCED)

1. (i) 125 N, (ii) 80 N2. 3b / 2 , b/2, b 3. (a) t = 2s, (b) 3.14 m/s 4. 2 sec

5. 3 k rad / s 2 ,  2 k rad / s 6. (a) 2 rad/s2, (b) 12 + 2t for t  2s, 16 for t  2s, (c) 28565  169 , 256
1/ 2
3  3
m/s (d) 44 rad
2
7. (a) 25 2
m / s (b)  125  m/s (c) 25/2 m/s2
2  4 
v2 30 35
8. (a) v02/2g, (b) 2g 9. 2 2 10. (a) per min., (b) 150 N 11. 0.1 sec.
R  2
25
12. 2 g rad/s 13. (1 – 3 / 2)mg 14.
24
(B) WORK, POWER AND ENERGY
15. (a) Zero, (b) Zero, (c) –mgvt (d) mgvt 16. (i) Zero, (ii) 500 J 17. 150 J

18. 20 Ergs 19. –8.75 J 20. (a) 875 Joule (b) –250 joule (c) 625 joule.

(d) Change in kinetic energy of the body is equal to the work done by the net force in 10 second.
This is in accordance with work-energy theorem
21. (a) Since the gravitational force is a conservative force therefore the work done in round
trip is zero. (b) 18.5 J (c) –7.6 J (d) 10.9 J

: 0744-2209671, 08003899588 | url : www.motioniitjee.com, : info@motioniitjee.com


Page # 200 CIRCULAR & WORK, POWER & ENERGY

22. (a) a1 = F/m, so v1 = a1t = Ft/m (b) Since velocities and, v = vc + v1 = vc + Ft/m
2 2 2
m( v1 ) F t m( v c  v1 ) 2 mv 2c a t 2 Ft 2
(c) K1 = = (d) K  – (e) s1 is 1  (f) s1 + vct
2 2m 2 2 2 2m
(h) Compare W and W1 and K and K1, they are respectively equal.
(i) The work-energy theorem holds for moving observers.
2m12 g2
23. 24. 3 25. (a) 46 J, (b) –3k 26. 50 W 27. 8 s 28. 3/4 29. 80 KW
k
5 5
a 2a
30. 54 sec. 31. WABC = WADC = (J), WAC = (J)
3 5
dU dU dU
32. (a) F = –  (b) F = – = –3ay2 + 2by (c) F = – = – U0 cos y
dy dy dy
1 gl 5
33. 34. gR, xmin  2R 35. 20/3 N, 10000 J kg–1
3 3 2

15 3 vR R( vt  R)v1/ 2 b
1. 3.3 m 2. N 3. aN  2
, a t  2 3/ 2 4. F = –3ax 2 + b , x  ,
2 ( 2Rt  vt ) (2Rt  vt ) a
2b b
KEmax  5. 500 N/m 6. (i) 36 N, (ii) 11.66 rad/sec, (iii) 0.1 m, 0.2m 7. mgR / 2, 2 gR
3 3 a
4 2 3 2 m 5
8. , 3a1 = 4a2 + 20, T = 12 N 9. 2.5 cm 10. (a) f  – xg ; (b) g ; (c)
5 5  2
a
11. 4 m/s, 24.5J, 40 J 12. (i) , (ii) 2 ag, 2a 13. (a) 2 2 r (b) h  19 r , (c) g
2 27

Exercise - IV PREVIOUS YEAR QUESTIONS

LEVEL - I JEE MAIN


(A) CIRCULAR MOTION
1. B 2. B 3. D 4. D 5. C 6. C
(B) WORK, POWER AND ENERGY
7. B 8. B 9. B 10. B 11. B 12. A 13. A
14. A 15. A 16. D 17. D 18. D 19. A 20. A
21. D 22. D

LEVEL - II JEE ADVANCED


NB
NA
5mg
mg

1. B 2. (a) N = 3mg cos  – 2mg, (b) cos


cos 

cos   2 / 3 cos  1 cos   2 / 3


cos   –1
3. D 4. A 5. A 6. C 7. D
8. Work = T.S. = 4.8 × 10/6 = 8N 9. C 10. D 11. 4 12. A

Corporate Head Office : Motion Education Pvt. Ltd., 394 - Rajeev Gandhi Nagar, Kota-5 (Raj.)

Vous aimerez peut-être aussi